Sei sulla pagina 1di 172

PROGRAMA

NACIONAL
OLIMPADAS
DE QUMICA
Q
U

M
I
C
A
2
0
1
2
XVIII
Olimpada
Norte/Nordeste
de Qumica
V Olimpada
Brasileira de
Qumica Jnior
Olimpada
Brasileira
de Qumica
44
th
International
Chemistry
Olympiad
17 Olimpada
Iberoamericana
de Qumica
Prof. Jesualdo Pereira Farias
Reitor da Universidade Federal do Cear
Prof. Jos Arimatia Dantas Lopes
Reitor da Universidade Federal do Piau
Prof
a
. Mrcia Maria Tavares Machado
Pr-Reitora de Extenso da UFC
Profa. Maria da Glria Carvalho Moura
Pr-Reitora de Extenso da UFPI
Newton Mrio Battastini
Presidente da Associao Brasileira de Qumica

Prof. Srgio Maia Melo
Coordenador do Programa Nacional Olimpadas de Qumica
Prof. Jos Arimatia Dantas Lopes
Vice-coordenador do Programa Nacional Olimpadas de Qumica e
Coordenador da Equipe Pedaggica (docentes do Estado do Piau)
Cristiano Marcelino Jr.
Antonio Carlos Pavo
Flvia Christiane Guinhos
Ktia Cristina de Freitas Silva
Natrcia Maria Miranda Bezerra
Marlene Barbosa Vieira
Vanessa Oliveira Cavalcanti
Preparao dos exames experimentais e edio de ftas de vdeo

ISSN: 1809-2012
Imprensa Universitria
Universidade Federal do Cear
Organizao de originais:
Prof. Srgio Melo

Capa:
Maherle
Editorao e Projeto Grfco:
Maherle/Srgio Melo
2012 Programa Nacional Olimpadas de Qumica
Lanamento em 30.11.2012 por ocasio da solenidade de encerramento dos eventos: XVIII Olimpada Nor-
te/Nordeste de Qumica, Olimpada Brasileira de Qumica Jnior e Olimpada Brasileira de Qumica - 2012
Tiragem: 15.000 exemplares.
Distribuio gratuita.
QUMICA

N
D
I
C
E
2
0
1
2Parte 1 - Olimpadas Nacionais de Qumica
Calendrio . . . . . . . . . . . . . . . . . . . . . . . . . . . . . . . . 04
Mensagem . . . . . . . . . . . . . . . . . . . . . . . . . . . . . . . . 05
Opiniao . . . . . . . . . . . . . . . . . . . . . . . . . . . . . . . . . . 07
XVIII Olimpada Norte/Nordeste
Exames . . . . . . . . . . . . . . . . . . . . . . . . . . . . . 09
Soluoes escolhidas . . . . . . . . . . . . . . . . . . . . . . . 13
Resultados . . . . . . . . . . . . . . . . . . . . . . . . . . . 18
V OBQ Junior
Exames Fase I . . . . . . . . . . . . . . . . . . . . . . . . . . 25
Exames Fase II . . . . . . . . . . . . . . . . . . . . . . . . . . 33
Soluoes escolhidas . . . . . . . . . . . . . . . . . . . . . . . 00
Resultados . . . . . . . . . . . . . . . . . . . . . . . . . . . 39
OBQ 2012
Fase III Modalidade A. . . . . . . . . . . . . . . . . . . . . . . 42
Fase III Modalidade B . . . . . . . . . . . . . . . . . . . . . . 50
Soluoes escolhidas . . . . . . . . . . . . . . . . . . . . . . . 59
Resutados . . . . . . . . . . . . . . . . . . . . . . . . . . . . 70
Parte 2 - Processo seletivo para compor a equipe nas olimpadas internacionais de 2012
PROCESSO SELETIVO . . . . . . . . . . . . . . . . . . . . . . . . . . . . 78
OBQ Fase IV . . . . . . . . . . . . . . . . . . . . . . . . . . . . . . . . 81
OBQ FASE V . . . . . . . . . . . . . . . . . . . . . . . . . . . . . . . . 82
OBQ FASE VI . . . . . . . . . . . . . . . . . . . . . . . . . . . . . . . . 84
Resultado Final . . . . . . . . . . . . . . . . . . . . . . . . . 91
44
th
International Chemistry Olympiad . . . . . . . . . . . . . . . . . . 92
Exame Terico . . . . . . . . . . . . . . . . . . . . . . . . . . 92
Exame Prtico . . . . . . . . . . . . . . . . . . . . . . . . . 117
17 Olimpada Iberoamericana . . . . . . . . . . . . . . . . . . . . . 132
Exame Terico . . . . . . . . . . . . . . . . . . . . . . . . . 132
Exame Prtico . . . . . . . . . . . . . . . . . . . . . . . . . 151
Destaques Olmpicos . . . . . . . . . . . . . . . . . . . . . . . . . . . 158
Depoimentos . . . . . . . . . . . . . . . . . . . . . . . .161
Consideracoes Finais . . . . . . . . . . . . . . . . . . . . . . . . . . . 165
Enderecos dos Cooordenadores . . . . . . . . . . . . . . . . . . . . . . 166
Programa Nacional Olimpadas de Qumica <<
4
Calendrio 2012
Data Hora Atividade
14/01/2012 14:00h Fase IV da OBQ-2011 (Exame sobre tcnicas laboratoriais com o objetivo
de selecionar a equipe que representar o Brasil nos Estados Unidos).
06 a 17/02/2012 Curso de Aprofundamento e Excelncia (Fase V) para os 15 estudantes selecionados
no exame de conhecimentos de laboratrio. Ser ministrado pela UFMG. Estudantes
se deslocaro at a cidade-sede.
17/03/2012 9:00h Exames da Olimpada Brasileira de Qumica - 2011 Fase VI - Questes de
baseadas na lista sugerida pelos organizadores da 44th IChO.
07/04/2012 Divulgao dos nomes dos quatro estudantes que representaro o Brasil nas com-
peties internacionais
03 a 21/05/2012 Inscries para a XVIII Olimpada Norte/Nordeste de Qumica - XVIII ONNeQ. Quarenta
estudantes por estado.
02/06/2012 Exames da Olimpada Norte/Nordeste de Qumica - XVIII ONNeQ.
03/07/2012 Divulgao de resultados da Olimpada Norte/Nordeste de Qumica - XVIII ONNeQ. A
partir de 03/07/2012
06/06 a
15/08/2012
Inscries para a V Olimpada Brasileira de Qumica Jnior. Escolas inscrevem seus alu-
nos de 8 e 9 anos do ensino fundamental. On line.
21 a 30/07/2012 44 Olimpada Internacional de Qumica, Washington - USA.
01 a 15/08/2012 Inscries para a Olimpada Brasileira de Qumica - 2012. Vinte e cinco estudantes por
estado na modalidade A (estudantes da penltima srie do ensino mdio ou srie
anterior, vinte e cinco estudantes por estado na modalidade B (estudantes da ltima
srie do ensino mdio). Inscries restritas aos coordenadores-estaduais.
25/08/2012 14:00h Exames da Olimpada Brasileira de Qumica - 2012. Fase III - Modali-
dades A e B. Questes analtico-expositivas.
25/08/2012 Exames da V Olimpada Brasileira de Qumica Jnior - V OBQjr,
para estudantes de 8 e 9 anos. (Fase I).
22 a 30/09/2012 17 Olimpada Ibero-americana de Qumica, Santa F de la Veracruz - Argentina.
20/10/2012 Exames da V Olimpada Brasileira de Qumica Jnior, V OBQjr. (Fase II). Postagem dos
exames at 26/10/2012.
25/10/2012 Divulgao dos resultados da OBQ-2012.
05/11/2012 Divulgao de resultados da V Olimpada Brasileira de Qumica Jnior, V OBQjr, a partir
de 05/11/2012.
29 e 30/11/2012 9h30min Reunio do Conselho de coordenadores, em Fortaleza.
30/11/2012 19:00h Solenidade de encerramento e premiao, em Fortaleza.
>> Olimpada Brasileira de Qumica - 2012
5
Mensagem
M
a
Mensagem do Reitor da Universidade Federal do
Piau
aos participantes das Olimpadas de
Qumica
Programa Nacional Olimpadas de Qumica <<
6
Mensagem
AGUARDANDO
O ARQUIVO
>> Olimpada Brasileira de Qumica - 2012
7
Pr-sal, cincia, tecnologia e educao
Srgio Mascarenhas
Enquanto governadores, deputados e senadores brasileiros se engalfnham num
cabo de guerra poltico pela partilha federativa dos recursos a serem gerados com a
explorao do petrleo da camada pr-sal, vai passando quase despercebida e mais
uma vez negligenciada a oportunidade histrica de o Pas garantir o uso desses re-
cursos para dar um salto indito e h muito necessrio nas reas de cincia, tecnolo-
gia e educao.
Se no bastasse atentar para a dvida humana e social que representa o atraso
brasileiro nos indicadores de desempenho educacional e nos rankings internacio-
nais de pesquisa e desenvolvimento, vale notar que nenhum pas poder ter um real
desenvolvimento, neste sculo, sem um programa robusto de impulso inovao
que passe, tambm, pela incluso intelectual das novas geraes.
Um verdadeiro plano de desenvolvimento da cincia, tecnologia e educao no
Brasil no poder ser feito s com protocolos de intenes, reduo da burocracia e
fomento pontual a programas e instituies de excelncia. Nosso problema no a
falta de instrumentos, de recursos. Numa palavra, preciso destinar mais dinheiro,
muito mais dinheiro, para que o Pas possa irrigar essa cadeia de capital humano que
comea nas creches, passa pelo ensino fundamental e mdio at chegar nas univer-
sidades, programas de ps-graduao e centros de inovao associados a empresas
capazes de aplicar tecnologias inovadoras na gerao de riqueza.
A prpria competncia para explorar o petrleo do pr-sal e os servios dele de-
rivados, com autonomia e inteligncia, em mdio e longo prazos, depende da cons-
tante renovao e evoluo dessa cadeia intelectual. Nunca o Pas precisou tanto
de engenheiros, gelogos, fsicos, qumicos, cientistas da computao, matemticos,
entre tantos outros profssionais cujo talento e formao se empregam em todas as
etapas de gerao e aplicao do conhecimento.
E no basta dar a esses futuros profssionais um diploma de nvel superior. Ser
preciso, sim, investir na qualidade dessa formao, de modo que sejam dadas as con-
dies para que possam inovar, gerar novas tcnicas, processos e produtos intelectu-
almente apropriveis e sustentar uma continuada e acirrada competio tecnolgica
com seus colegas norte-americanos, europeus e asiticos.
Nas ltimas dcadas, os ciclos de produo e aplicao do conhecimento se
encurtaram, levando a uma convergncia temporal entre cincia e tecnologia. En-
quanto foram necessrios 40 anos desde o estabelecimento das leis da eletricidade
Opinio
Programa Nacional Olimpadas de Qumica <<
8
e magnetismo at o funcionamento do motor eltrico, a tecnologia mais recente da
luz laser, por exemplo, j encontrou utilidade no mesmo ano de seu invento. Cada
vez mais, produtos, processos e servios tecnolgicos tm vida curta, pressionados
por um novo ciclo de inovao dentro de uma economia globalizada e com compe-
titividade acelerada.
Vale ento perguntar: em que, exatamente, o Brasil tem se mostrado inovador?
At hoje, no temos um nico Prmio Nobel, nem cientfco nem literrio. Se quiser-
mos comparar, basta lembrar que a Universidade Rockefeller, de Nova York, sozinha,
j recebeu 26 deles, e o mais recente na rea da medicina. O fato que nossos jo-
vens nunca foram devidamente educados para uma cultura baseada em cincia e
tecnologia. Basta olhar em torno. Campos de pelada h em todo lugar, do centro
s periferias. Carnaval fora de poca, quase todo fm de semana. J museus de cin-
cia, planetrios e bibliotecas so raros, e parecem cada vez menos procurados, assim
como a prpria carreira de professor, como mostrou estudo recente da Fundao
Carlos Chagas.
Para mudar isso, cincia e tecnologia precisam impregnar o sistema educacional.
Nossa inovao deveria comear pelos mtodos e processos de ensino. Ainda esta-
mos longe da escola parque sonhada por Ansio Teixeira. Nossos redutos de educa-
o ainda respiram um ar cartorial, com estruturas engessadas, onde a tecnologia
mal-empregada e crescem os impulsos ao bullying e violncia. Nesse ambiente, os
professores vivem uma espcie de sndrome de quatro medos: o medo do aluno, o
medo do seu prprio desamparo pedaggico, o medo do conhecimento avassalador
que jorra pela internet e o medo do futuro de sua carreira, desprezada no s pelo
Estado, mas tambm pelos sistemas privados, com salrios irrisrios e cargas didti-
cas intensas.
Mas no estamos perdidos. H diversos bons exemplos de como virar esse jogo
espalhados pelo Brasil. Um deles a Empresa Brasileira de Pesquisa Agropecuria
(Embrapa), que aproximou a cincia e tecnologia do agronegcio nacional. Ainda
falta popularizar a banda larga, informatizar escolas, disseminar o uso de computa-
dores pessoais, criar centenas de museus e centros de cincia, promover o uso cvico
das redes sociais e a produo de contedos educativos por agncias multimdia, en-
tre outras propostas que tive a oportunidade de apresentar durante a 4. Conferncia
Nacional de Cincia, Tecnologia e Inovao para o Desenvolvimento Sustentvel, em
2010, em Braslia.
Agora, nossas esperanas repousam na angustiada solicitao da Academia Bra-
sileira de Cincias (ABC) e da Sociedade Brasileira para o Progresso da Cincia (SBPC)
para que parte dos recursos do petrleo do pr-sal seja destinada, em lei, para esses
urgentes investimentos em cincia, tecnologia e inovao, formando o alicerce do
Opinio
>> Olimpada Brasileira de Qumica - 2012
9
desenvolvimento futuro do Pas. A meta deve ser, no mnimo, duplicar o volume de
recursos investidos ao ano nessas reas. Esse, sim, seria um verdadeiro salto de de-
senvolvimento do Brasil.
Srgio Mascarenhas, fsico, presidente honorrio da SBPC, membro
titular da Academia Brasileira de Cincias, professor emrito do Insti-
tuto de Fsica de So Carlos, da USP
Opinio
"Projetar Braslia para os Polticos que vocs colocaram l,
foi como criar um lindo vaso de fores pr vocs usarem
como pinico. Hoje eu vejo, tristemente, que Braslia nunca
deveria ter sido projetada em forma de avio, mas sim de
Camburo"
Oscar Niemayer
Programa Nacional Olimpadas de Qumica <<
10
XVIII Olimpada Norte/Nordeste de Qumica
Questo1
O citocromo, molcula que representaremos como CyFe
+2
, reage com o ar que
respiramos para fornecer energia necessria para sintetizar adenosina trifosfato
(ATP). O corpo usa ATP como fonte de energia para outras reaes. Com base nos
potenciais de reduo:
E
o
O
2(g)
+ 4H
+
(aq)
+ 4e
-
2H
2
O
(l)
+0,82 V
CyFe
+3
(aq)
+ e
-
CyFe
+2
(aq)
+0,22 V
Responda as questes abaixo:
a) Considerando que esta reao ocorra a em uma clula eletroqumica, indique
os eletrodos (catodo e anodo) onde cada semi-reao deve ocorrer.
b) Escreva a equao geral balanceada, e determine o potencial DE
o
gerado nessa
reao.
c) Explique se esse processo exotrmico ou endotrmico.
d) Explique porque essa transformao do tipo oxido-reduo, e identifque os
agentes oxidante e redutor.
Questo2
O agrnomo de uma usina de cana de acar solicitou uma avaliao do teor de
sacarose contida em determinado lote de cana-de-acar. Para isso, foi entregue
ao qumico uma amostra de 2,0 litros de caldo de cana para determinar a concen-
trao de sacarose (C
12
H
22
O
11
).
a) Ao receber o resultado da anlise, expresso em quantidade de matria (0,25
mol.L
-1
), o auxiliar de escritrio, no conseguindo decifr-lo, recorreu ao
agrnomo que calculou a massa (g) de sacarose contida nos 2,0 litros de caldo
de cana 0,25 mol.L
-1
. Que massa (g) de sacarose est contida nesses 2 litros de
soluo?
XVIII ONNeQ
>> Olimpada Brasileira de Qumica - 2012
11
XVIII ONNeQ
b) Caso toda esta massa de sacarose fosse utilizada para adoar um suco de limo
para que este fcasse na concentrao 0,15 mol.L
-1
de sacarose, quantos mL de
suco poderiam ser preparados?
Dados: Massas molares C=12 g, O=16g, H=1g
Questo3
Os raios atmico e inico (2+) para o clcio e para o zinco esto relacionados a
seguir:
Elemento Raio () 1
a
energia de ionizao
Ca 1,74 6,1 eV
Zn 1,31 9,4 eV
Ca
+2
0,99
Zn
+2
0,74
a) Explique a razo de o raio inico em cada caso ser menor que o raio atmico
b) Por que o raio atmico do clcio maior que o do zinco?
c) Por que a energia de ionizao do zinco maior que a do clcio?
d) Para a formao dos ctions Ca
+2
e Zn
+2
ser necessria uma segunda energia
de ionizao. Em relao primeira, essa energia ser maior ou menor? ex-
plique.
Questo4
Devido os atentados terroristas ocorridos em Nova Iorque, Madri e Londres, os
Estados Unidos e pases da Europa tm aumentando o controle quanto venda e
produo de compostos explosivos que possam ser usados na confeco de bom-
bas. Dentre os compostos qumicos explosivos, a nitroglicerina um dos mais
conhecidos. um lquido temperatura ambiente, altamente sensvel a qualquer
vibrao, decompondo-se de acordo com a equao:
2C
3
H
5
(NO
3
)
3
(l) 3N
2
(g) + 1/
2
O
2
(g) + 6 CO
2
(g) + 5 H
2
O(g)
Programa Nacional Olimpadas de Qumica <<
12
XVIII ONNeQ
Considere-se uma amostra de 4,45 g de nitroglicerina, massa molar 227 g/mol,
contida em um frasco fechado com volume total de 100,0 mL .
a) Calcule a entalpia envolvida na exploso.
Dados: entalpia de formao das substncias (kJ/mol): C
3
H
5
(NO
3
)
3
(l) = -364;
CO
2
(g)=-394; H
2
O(g) =-242.
b) Calcule a presso mxima no interior do frasco antes de seu rompimento, con-
siderando-se que a temperatura atinge 127C.
Dado: R = 0,082 atm.L.K
-1
.mol
-1
.
Questo5
Um composto orgnico A, opticamente ativo, contm 6 tomos de carbono por
molcula, ao ser desidratado produz o composto B, tambm opticamente ativo.
O mesmo composto A ao ser oxidado com permanganato de potssio, forma o
composto C, tambm opticamente ativo, que reage com metanol, em meio cido,
formando o ster D. O composto B ao ser reduzido com H
2
na presena de Pd
forma o composto E, opticamente inativo.
Escreva as estruturas e os nomes dos compostos A, B, C, D e E.
>> Olimpada Brasileira de Qumica - 2012
13
Questo1
Desenvolvida por Ksia Priscila Omena Cardoso, IFAL Macei, AL
a) A semi - reao:
RESPOSTAS ESCOLHIDAS

QUESTO 01
Desenvolvida por Ksia Priscila Omena Cardoso, IFAL Macei, AL

a) A semi - reao:



Tem o maior potencial de reduo (E), logo atua como ctodo.
Enquanto que a semi-reao:


Possui o menor potencial de reduo, atuando como nodo.

b)



(Equao Geral)
O potencial gerado :
E= E
(recebe e-)
- E
(perde e-)

E= + 0,82 (+0,22)
E= 0,60 V

c) Tratando-se de uma reao na qual o O
2
atua como agente oxidante, o processo
exotrmico.


d) Pela equao geral:



No CyFe
2+
, o Nox do ferro vai de +2 a +3 nos produtos, indicando assim perda
de eltrons, logo o CyFe
2+
o agente redutor. Enquanto o O
2
o agente oxidante,
Tem o maior potencial de reduo (E), logo atua como ctodo.
Enquanto que a semi-reao:
RESPOSTAS ESCOLHIDAS

QUESTO 01
Desenvolvida por Ksia Priscila Omena Cardoso, IFAL Macei, AL

a) A semi - reao:



Tem o maior potencial de reduo (E), logo atua como ctodo.
Enquanto que a semi-reao:


Possui o menor potencial de reduo, atuando como nodo.

b)



(Equao Geral)
O potencial gerado :
E= E
(recebe e-)
- E
(perde e-)

E= + 0,82 (+0,22)
E= 0,60 V

c) Tratando-se de uma reao na qual o O
2
atua como agente oxidante, o processo
exotrmico.


d) Pela equao geral:



No CyFe
2+
, o Nox do ferro vai de +2 a +3 nos produtos, indicando assim perda
de eltrons, logo o CyFe
2+
o agente redutor. Enquanto o O
2
o agente oxidante,
Possui o menor potencial de reduo, atuando como nodo.
b)
RESPOSTAS ESCOLHIDAS

QUESTO 01
Desenvolvida por Ksia Priscila Omena Cardoso, IFAL Macei, AL

a) A semi - reao:



Tem o maior potencial de reduo (E), logo atua como ctodo.
Enquanto que a semi-reao:


Possui o menor potencial de reduo, atuando como nodo.

b)



(Equao Geral)
O potencial gerado :
E= E
(recebe e-)
- E
(perde e-)

E= + 0,82 (+0,22)
E= 0,60 V

c) Tratando-se de uma reao na qual o O
2
atua como agente oxidante, o processo
exotrmico.


d) Pela equao geral:



No CyFe
2+
, o Nox do ferro vai de +2 a +3 nos produtos, indicando assim perda
de eltrons, logo o CyFe
2+
o agente redutor. Enquanto o O
2
o agente oxidante,
O potencial gerado :
E= E(recebe e-) - E(perde e-)
E= + 0,82 (+0,22)
E= 0,60 V
c) Tratando-se de uma reao na qual o O
2
atua como agente oxidante, o processo
exotrmico.
d) Pela equao geral:

RESPOSTAS ESCOLHIDAS

QUESTO 01
Desenvolvida por Ksia Priscila Omena Cardoso, IFAL Macei, AL

a) A semi - reao:



Tem o maior potencial de reduo (E), logo atua como ctodo.
Enquanto que a semi-reao:


Possui o menor potencial de reduo, atuando como nodo.

b)



(Equao Geral)
O potencial gerado :
E= E
(recebe e-)
- E
(perde e-)

E= + 0,82 (+0,22)
E= 0,60 V

c) Tratando-se de uma reao na qual o O
2
atua como agente oxidante, o processo
exotrmico.


d) Pela equao geral:



No CyFe
2+
, o Nox do ferro vai de +2 a +3 nos produtos, indicando assim perda
de eltrons, logo o CyFe
2+
o agente redutor. Enquanto o O
2
o agente oxidante,

No CyFe
2+
, o Nox do ferro vai de +2 a +3 nos produtos, indicando assim perda de
eltrons, logo o CyFe
2+
o agente redutor. Enquanto o O
2
o agente oxidante, pois
seu Nox vai de zero a -2 nos produtos. Essa variao do Nox indica perda e ganho
de eltrons, sendo assim uma reao de xido-reduo.
Solues Escolhidas
Programa Nacional Olimpadas de Qumica <<
14
Questo2
Desenvolvida por Wederson Santos Silva, Colgio Motiva - Campina Grande, PB
a) A massa de sacarose (massa molar de 342 g/mol) contida nesses 2 litros de
soluo de caldo de cana 0,25mol L
-1
de 171 gramas. Sabemos disso, pois:
sabendo que cada litro de caldo de cana possui 0,25 mol de sacarose e que
temos 2 litros, encontraremos 0,5 mol de sacarose (2 x 0,25) nos 2 litros de
caldo. Alm disso, constatamos que a massa molar desse soluto de 342 g/
mol, sendo assim, na quantidade de 0,5 mol de sacarose temos:
342 g - 1 mol de sacarose
X - 0, 5 mol de sacarose
Temos, pois, pela regra de trs acima, a quantidade de 171 gramas de sacarose nos
2 litros de caldo de cana.
b) Enquanto 1 mol de sacarose possui 342 g, 0,15 mol possui:
1 mol - 342 g
0,15 mol - X X = 51,3 g
Logo, 0,15 mol de sacarose possui 51,3 g. Como temos 171 gramas de sacarose
sendo usados para adocicar o suco de limo a uma concentrao de 0,15 mol L
-1
,
teremos:
51,3 g de sacarose - 1 litro de suco
171 g de sacarose - X
Assim sendo, poderemos preparar com 171 g de sacarose 3,33 litros de suco de
limo ou 3333 mL do suco em concentrao de 0,15 mol L
-1
de sacarose.
Questo3
Desenvolvida por Flvio Lus Schneider Junior, Colgio Militar Manaus, AM

a) Quando um tomo sofre ionizao e torna-se um ction, haver um aumento
na atrao entre prtons e eltrons, essa atrao ser maior nos ons do que
nos tomos no estado neutro devido conservao do nmero de prtons
e a diminuio do nmero de eltrons e consequentemente, o aumento da
atrao entre prtons e eltrons, com o aumento dessa atrao ocorrer uma
XVIII ONNeQ
>> Olimpada Brasileira de Qumica - 2012
15
diminuio da distncia entre os prtons e os eltrons, diminuindo assim o
raio. Logo, o raio ser maior em entidades neutras do que em ctions.
b) O zinco possui uma carga nuclear, nmero de prtons, maior que a carga nu-
clear do clcio, como os dois esto em um mesmo perodo, possuem o mesmo
nmero de camadas, o raio ir depender da carga nuclear efetiva, e a carga
nuclear efetiva aumenta quando o nmero de prtons aumenta como o zinco
possui um maior nmero de prtons ele ter uma maior carga nuclear efetiva e
consequentemente seus prtons atrairo seus eltrons com mais intensidade
do que os do clcio, com o aumento da atrao ocorrer uma diminuio da
distncia entre prtons e assim ter um menor raio que o do clcio. Logo, pelo
fato do clcio ter uma carga nuclear efetiva menor, ele tem um raio maior que
o do zinco.
c) A energia de ionizao a energia necessria para se retirar um eltron do
tomo no estado fundamental e gasoso. Como o zinco possui um raio menor
que o do clcio e possui uma carga nuclear efetiva maior que a do clcio, seus
prtons atraem os eltrons com mais intensidade que os do clcio, ento, para
se retirar um eltron que est fortemente atrado pelo ncleo necessrio uma
maior quantidade de energia, e como essa atrao maior no zinco que no
clcio, a energia de ionizao de zinco fca sendo maior que a do clcio.
d) Maior. Quando for retirado o primeiro eltron, ocorrer um aumento na atrao
entre os prtons e eltrons, devido a conservao do nmero de prtons e a
diminuio do nmero de eltrons, como o aumento dessa atrao maior vai
ser a energia necessria para a retirada do eltron, por essa a segunda energia
de ionizao bem maior que a primeira.
Questo4
Desenvolvida por Pablo talo do Nascimento Santos, Colgio Santa Rita Picos, PI
a) 2C
3
H
5
(NO
3
)
3(l)
3N
2(g)
+ O
2(g)
+ 6CO
2(g)
+ 5H
2
O
(g)
H = Hp - Hr
H = 0 + 0 + 6(- 394) + 5(-242) 2(-364)
H = -2364 1210 + 728
H = - 3574 + 728
H = - 2846 KJ / 2 mols de 2C
3
H
5
(NO
3
)
3
Solues Escolhidas
Programa Nacional Olimpadas de Qumica <<
16
Descobrindo o nmero de mols de nitroglicerina envolvidos na reao:
n = m/MM
n = 4,45g / 227g/mol
n = 0,02 mols de nitroglicerina (aproximadamente)
Logo, a entalpia envolvida na exploso pode ser assim calculada:
H= - 2846 KJ 2 mols de 2C
3
H
5
(NO
3
)
3
x 0,02 mols de O
3
H
5
(NO
3
)
3
H = - 2846 KJ x 0,01KJ
H = - 28,46 KJ
b) A presso mxima no interior do frasco pode ser calculada tomando como
referncia a temperatura mxima atingida pelo sistema:
P.V = n.R.T
P x 0,1 = 0,145 x 0,082 x 400
P = 0,145 x 10 x 32,8
P =1,45 x 32,8
P = 47,56 atm
A presso mxima no interior do frasco antes de seu rompimento de 47,56 atm.
Questo5
Desenvolvida por Gabriel Mathews Viana Pinheiro, Colgio Master Fortaleza, CE
Estrutura e nome dos compostos A, B, C, D e E.
a)

Estrutura e nome dos compostos A, B, C, D e E.
a)

H
3
C
OH
CH
3
3-metil pentan-1-ol
b)

CH
2
H
3
C
CH
3
3-metil pent-1-eno
c)

H
3
C
OH
CH
3
O
cido 3-metilpentanico
d)

H
3
C
O
CH
3
CH
3
O
3-metilpentanoato de metila
e)

CH
3
H
3
C
CH
3
3-metil pentano

Raciocnio utilizado para desvendar as estruturas de A, B, C, D e E. A foi
desidratado, o que nos leva a crer que ele um lcool. A oxidao de um lcool
pode fornecer aldedo ou cido carboxlico (se for primrio) ou cetona (se
secundrio).
A reao de C para produzir D foi uma esterificao, logo, no pode ser
aldedo ou cetona. C um cido carboxlico, ento A um lcool primrio.
b)

Estrutura e nome dos compostos A, B, C, D e E.
a)

H
3
C
OH
CH
3
3-metil pentan-1-ol
b)

CH
2
H
3
C
CH
3
3-metil pent-1-eno
c)

H
3
C
OH
CH
3
O
cido 3-metilpentanico
d)

H
3
C
O
CH
3
CH
3
O
3-metilpentanoato de metila
e)

CH
3
H
3
C
CH
3
3-metil pentano

Raciocnio utilizado para desvendar as estruturas de A, B, C, D e E. A foi
desidratado, o que nos leva a crer que ele um lcool. A oxidao de um lcool
pode fornecer aldedo ou cido carboxlico (se for primrio) ou cetona (se
secundrio).
A reao de C para produzir D foi uma esterificao, logo, no pode ser
aldedo ou cetona. C um cido carboxlico, ento A um lcool primrio.
XVIII ONNeQ
>> Olimpada Brasileira de Qumica - 2012
17
c)

Estrutura e nome dos compostos A, B, C, D e E.
a)

H
3
C
OH
CH
3
3-metil pentan-1-ol
b)

CH
2
H
3
C
CH
3
3-metil pent-1-eno
c)

H
3
C
OH
CH
3
O
cido 3-metilpentanico
d)

H
3
C
O
CH
3
CH
3
O
3-metilpentanoato de metila
e)

CH
3
H
3
C
CH
3
3-metil pentano

Raciocnio utilizado para desvendar as estruturas de A, B, C, D e E. A foi
desidratado, o que nos leva a crer que ele um lcool. A oxidao de um lcool
pode fornecer aldedo ou cido carboxlico (se for primrio) ou cetona (se
secundrio).
A reao de C para produzir D foi uma esterificao, logo, no pode ser
aldedo ou cetona. C um cido carboxlico, ento A um lcool primrio.
d)

Estrutura e nome dos compostos A, B, C, D e E.
a)

H
3
C
OH
CH
3
3-metil pentan-1-ol
b)

CH
2
H
3
C
CH
3
3-metil pent-1-eno
c)

H
3
C
OH
CH
3
O
cido 3-metilpentanico
d)

H
3
C
O
CH
3
CH
3
O
3-metilpentanoato de metila
e)

CH
3
H
3
C
CH
3
3-metil pentano

Raciocnio utilizado para desvendar as estruturas de A, B, C, D e E. A foi
desidratado, o que nos leva a crer que ele um lcool. A oxidao de um lcool
pode fornecer aldedo ou cido carboxlico (se for primrio) ou cetona (se
secundrio).
A reao de C para produzir D foi uma esterificao, logo, no pode ser
aldedo ou cetona. C um cido carboxlico, ento A um lcool primrio.
e)

Estrutura e nome dos compostos A, B, C, D e E.
a)

H
3
C
OH
CH
3
3-metil pentan-1-ol
b)

CH
2
H
3
C
CH
3
3-metil pent-1-eno
c)

H
3
C
OH
CH
3
O
cido 3-metilpentanico
d)

H
3
C
O
CH
3
CH
3
O
3-metilpentanoato de metila
e)

CH
3
H
3
C
CH
3
3-metil pentano

Raciocnio utilizado para desvendar as estruturas de A, B, C, D e E. A foi
desidratado, o que nos leva a crer que ele um lcool. A oxidao de um lcool
pode fornecer aldedo ou cido carboxlico (se for primrio) ou cetona (se
secundrio).
A reao de C para produzir D foi uma esterificao, logo, no pode ser
aldedo ou cetona. C um cido carboxlico, ento A um lcool primrio.
Raciocnio utilizado para desvendar as estruturas de A, B, C, D e E. A foi de-
sidratado, o que nos leva a crer que ele um lcool. A oxidao de um lcool pode
fornecer aldedo ou cido carboxlico (se for primrio) ou cetona (se secundrio).
A reao de C para produzir D foi uma esterifcao, logo, no pode ser aldedo ou
cetona. C um cido carboxlico, ento A um lcool primrio.
A tem 6 tomos de carbono, sendo um deles quiral. Ao ser desidratado, forma o
alceno B, tambm quiral. A possibilidade vivel :
A tem 6 tomos de carbono, sendo um deles quiral. Ao ser desidratado,
forma o alceno B, tambm quiral. A possibilidade vivel :

Sabendo disso, as reaes que temos so:
H
3
C
OH
CH
3
H
2
SO
4
CH
2
H
3
C
CH
3
H
2
Pd
CH
3
H
3
C
CH
3
KMnO
4
H
3
C
CH
3
OH
O
CH
3
OH
H
+
H
3
C
O
CH
3
CH
3
O


H
3
C
OH
CH
3
H
2
SO
4
CH
2
H
3
C
CH
3
H
2
Pd
CH
3
H
3
C
CH
3

Solues Escolhidas
Programa Nacional Olimpadas de Qumica <<
18
XVIII ONNeQ
XVIII OLIMPADA NORTE/NORDESTE
de QUMICA 2012
O U R O
Nome Cidade UF Escore
Pablo talo do Nascimento Santos Picos PI 100,0
Flvio Lus Schneider Junior Manaus AM 99,1
Ramon Santos Gonalves da Silva Belm PA 96,0
Rafael Ribeiro Alves Areia PB 96,0
Joaquim Ivo Vasques Dantas Landim Fortaleza CE 95,0
Nathrcia Castro Mota Fortaleza CE 95,0
Vitria Nunes Medeiros Fortaleza CE 95,0
P R A T A
Nome Cidade UF Escore
merson Holanda Marinho Fortaleza CE 94,0
Daniel de Siqueira Carvalho Recife PE 92,0
Pedro Henrique Rodrigues da Mota Natal RN 92,0
Francisco Davi Barbosa dos Santos Fortaleza CE 90,0
Gabriel Mathews Viana Pinheiro Fortaleza CE 90,0
Wederson Santos Silva Massaranduba PB 90,0
Wendrey Lustosa Cardoso Manaus AM 85,9
Eleodrio Sales Bonfm Neto Areia PB 84,9
Ana Raquel Ferras Rameiro Recife PE 84,9
Ksia Priscila Omena Cardoso Macei AL 83,9
Vtor Juc Policarpo Fortaleza CE 83,9
Wildson Santos C. Rosa Teresina PI 83,9
B R O N Z E
Nome Cidade UF Escore
Juliana Sales do Nascimento Fortaleza CE 81,9
Matheus Salmito Rodrigues Ponte Fortaleza CE 81,9
Pedro Lucas Porto Almeida Fortaleza CE 81,9
Emanoela Bitencourt Varjao Aracaj SE 81,9
Rhosane Silva dos Santos Manaus AM 80,9
Jssica Silva Lopes Fortaleza CE 80,9
Gilmar Leite Pessoa Filho Joo Pessoa PB 80,9
Romullo Randell Macedo Carvalho Teresina PI 80,9
>> Olimpada Brasileira de Qumica - 2012
19
Resultados
Luana Ayane Rodrigues Santos Macei AL 80,4
Lucas Soares Rodrigues Gomes Joo Pessoa PB 80,4
Thiago Buarque de Gusmo Lafayette Macei AL 79,9
Alex Mendes Leonel Freire Macei AL 78,9
Johnatan Santiago Sto. Antnio de Jesus BA 78,9
Joana Lourenco de Carvalho Joo Pessoa PB 78,9
Gustavo Henrique P. C. de Albuquerque Recife PE 78,9
Flavia Vanessa Carvalho S. Esteves Teresina PI 78,9
Jonadab dos Santos Silva Macei AL 78,4
Camila Frade Oliveira Natal RN 78,4
Dimitri Reis de Matos Salvador BA 77,8
Diego Rodrigues Carvalho Salvador BA 76,8
Livio Moreira Rios So Lus MA 76,8
Jssica Matias Lewinter Fortaleza CE 76,8
Leandro Favaro Macap AP 76,8
Gabriel Castro Tavares Manaus AM 75,8
Hugo de Albuquerque Meira Recife PE 75,8
M E N O H O N R O S A
Nome Cidade UF Escore
Tssio Perotti Arruda Fortaleza CE 74,8
Felipe de Souza Souto Fortaleza CE 72,8
Lvia Rodrigues de Arajo Fortaleza CE 72,8
Fabrcio Maciel Soares So Lus MA 72,8
Heitor Augusto Carneiro de Almeida Joo Pessoa PB 72,8
Joao Vistor Porto Jaaske Joo Pessoa PB 71,8
Joo Guilherme N. de Carvalho Teresina PI 71,8
Roger Leite Lucena Imperatriz MA 71,3
Abdon de Andrade Moutinho Neto Recife PE 70,8
Igor Barbosa Cunha Aracaj SE 70,8
Marcos Antonio S Failanche Filho Macap AP 69,8
Pedro Henrique Gonalves Carvalho Fortaleza CE 69,8
Luckeciano Carvalho Melo Teresina PI 69,8
Dbora Barreto Ornelas Salvador BA 68,7
Breno Negreiros Pinto Fortaleza CE 68,7
Drvylla de Sousa Lima Fortaleza CE 68,7
Wessley Roberto Batista da S. Guirino Teresina PI 68,7
Programa Nacional Olimpadas de Qumica <<
20
Marcus Di Fabianni Ferreira Lopes Filho Manaus AM 67,7
Felipe Eduardo de Paula Santos Fortaleza CE 67,7
Haroldo Nogueira Victoriano Neto Fortaleza CE 67,7
Pedro talo Oliveira Gomes Fortaleza CE 67,7
Clisostenes Arruda Barbosa Campina Grande PB 67,7
Luiz Henrique Ferreira Campina Grande PB 67,7
Heron Ferreira Arago Santa Cruz do Capibaribe PE 67,7
Natlia Nobrega de Lima Natal RN 67,7
Liang Wei Dong Salvador BA 67,2
Bianca Davi Pereira de Almeida Campina Grande PB 67,2
Rodrigo Dias Garcia Salvador BA 66,7
Igor Chiasso Marques Jaboato dos Guararapes PE 66,7
Marcelo Henrique Assuno Barros Santa Cruz do Capibaribe PE 66,7
Gustavo Maximiano Alves Colinas do Tocantins TO 66,7
Daniel Meira Nobrega de Lima Joo Pessoa PB 65,7
Anglica de Brito Sousa Cocal dos Alves PI 65,7
Raphael Souza de Almeida Frei Paulo SE 65,7
Larissa Ferreira Motavaz Palmas TO 65,7
Felipe Castro Vergasta Salvador BA 65,2
Lucas da Silva Ferreira Macei AL 64,7
Jlio Csar Freitas Silva Fortaleza CE 64,7
Mateus Juc Pinheiro Fortaleza CE 64,7
Mauroclio Rocha Pontes Filho Fortaleza CE 64,7
Naiara de Oliveira Baptista Fortaleza CE 64,7
Joo Gabriel Mansano So Lus MA 64,7
Lvia Teresa Soars Stevenin Natal RN 64,7
Joo Ppedro O. Freitas Aracaj SE 64,7
Alberto Jos Pinto Salvador BA 64,2
Rebeca Beck Salvador BA 64,2
Kaynan Bezerra de Lima Fortaleza CE 64,2
Erinaldo da Costa Quintino Jnior Macei AL 63,7
Bianca Gomes Wanderley Manaus AM 63,7
Juliany Pires Figueiredo Manaus AM 63,7
Maria Isabel Assis Viasus Manaus AM 63,7
Victor Sousa Silva Fortaleza CE 63,7
Joo Guilherme N. de Carvalho Teresina PI 63,7
XVIII ONNeQ
>> Olimpada Brasileira de Qumica - 2012
21
Resultados
Felipe Guedes de Oliveira Kucera Manaus AM 63,2
Lo-Amy David de O. Silva Campina Grande PB 63,2
Gabriel Jos Moreira da Costa Silva Macei AL 62,7
Fbio Malheiros Lacerda Nascimento Salvador BA 62,7
Gabriel Dias Amrico Patos PB 62,7
Guilherme Lobo F. Ziani Recife PE 62,7
caro Vicente da Silva Rodrigues Teresina PI 62,7
Francisco Rebouas de Azevedo jnior Mossor RN 62,7
Isaac Csar Coelho Argolo Salvador BA 62,2
Mateus Caracas Veras Caucaia CE 62,2
Matheus Wenzel S Gonalves Aracaj SE 62,2
Letcia Ges Gita Fernandes Macei AL 61,7
Rayanne Arajo Bezerra Manaus AM 61,7
Thiago de Oliveira Guedes Salvador BA 61,7
Artur Souto Martins Fortaleza CE 61,7
Francisco Maxsuwel Nunes de Oliveira Caucaia CE 61,7
Heitor Sales de Castro Fortaleza CE 61,7
Jeferson de Queiroz Gomes Fortaleza CE 61,7
Joo Pedro Nunes Aquime Belm PA 61,7
Barbara de Alvarenga Borges Fonseca Joo Pessoa PB 61,7
Emilly Rennale Freitas de Melo Campina Grande PB 61,7
Jordan de Maria Barros Teresina PI 61,7
Felipe Pereira de Lima Manaus AM 61,2
Gilmara da Rocha Brito Cocal dos Alves PI 61,2
Mrcio Pinheiro Lima Manaus AM 60,7
Daniel Rocha de Sena Salvador BA 60,7
Caio Vidal Bezerra Fortaleza CE 60,7
Maria Beatriz B. Ferreira Campina Grande PB 60,7
Iago Guimares Sentges Salvador BA 60,2
Marcio Santos Souza Cardoso Salvador BA 60,2
Cayo Felliphe Costa Parente Manaus AM 59,6
Millena de Albuquerque Silva Manaus AM 59,6
Matheus veras Guterres mendes So Lus MA 59,6
Jose Alberto de Araujo Gomes Filho Campina Grande PB 59,6
Lauro Augusto J. Oliveira Aracaju SE 59,6
Roberto Vieira dos Santos Macei AL 59,1
Programa Nacional Olimpadas de Qumica <<
22
Ana Ceclia Carvalho Torres Recife PE 59,1
Elaine Costa Paulista PE 59,1
Ana Cristina Barreto S. de Arajo Itapissuma PE 58,6
Renato Frankiley da Silva Lima Recife PE 58,6
Rodrigo Castiel Reis de Souza Recife PE 58,6
Pamplona de Sousa Xavier Teresina PI 58,6
Thiago Oliveira Rodrigues Picos PI 58,6
Letcia Bito Maynart Aracaj SE 58,6
Vitor do Bomfm A. Carvalho Aracaj SE 58,6
Irma Csasznik Manaus AM 58,1
Ana Paula Carvalho Simes Salvador BA 58,1
Orlando Verosa Aceto Recife PE 58,1
Marina Albuquerque Berreza de Almeida Mossor RN 58,1
Thallyta Ktia Feij Arapiraca AL 57,6
Gabriel Saboia Manaus AM 57,6
Vitor Candeia Manaus AM 57,6
Mariane Greice Pereira Ventura Salvador BA 57,6
Emanuel Pereira de Queiroz Fortaleza CE 57,6
Felipe Brando Forte Fortaleza CE 57,6
Agda Morgana Alves Alves Belm PA 57,6
Gabriela Deus Correia Lima Teresina PI 57,6
Lucas Smith Manaus AM 57,1
Lian Chiamentti Joo Pessoa PB 57,1
Alexandre Andrade Cavalcanti de Almeida Recife PE 57,1
Gabriela Wanderley da N. Farias de Barros Mossor RN 57,1
Pablo Jango Lima Marques Manaus AM 56,6
Pedro Pereira de Almeida Salvador BA 56,6
Mariana Andressa luna pinheiro So Lus MA 56,6
Chelsea Magalhes Tavares da Silva So Lus MA 56,6
Emerson Rodrigues Vero Filho Recife PE 56,6
Milena Gomes de Andrade Macaparana PE 56,6
Lucas Leal Varjo Aracaj SE 56,6
Victor Yuri Santos Ramos Aracaj SE 56,6
Petrus Fay Dias da Fonsca Macei AL 56,1
Rodrigo Lins S. de Lima Salvador BA 56,1
Sofa Barretto Pereira Salvador BA 56,1
XVIII ONNeQ
>> Olimpada Brasileira de Qumica - 2012
23
Resultados
Larissa Victria Barra de Moura Belm PA 56,1
Lucas David Noveline Belm PA 56,1
Iuri Wagner Gomes Ferreira Natal RN 56,1
Jacobus Laurens de Jager Manaus AM 55,6
Luciana Disraeli Macedo Oliveira Macap AP 55,6
Matheus Bartz Noy Campina Grande PB 55,6
Matheus Rodrigues Marques de Lima Campina Grande PB 55,6
Tain Soares de Lucena Catingueira PB 55,6
Lucas Philipovsky Macap AP 55,1
Raissa Damasceno Barreto da Silva Salvador BA 55,1
Nilo Parente Pessoa Dias Fortaleza CE 55,1
Bianca da Silva Arajo Patos PB 55,1
Rayssa Daiana Silveira Okoro So Lus MA 54,6
Rodrigo Alarcio de Carvalho Belm PA 54,6
Gleyber Paixo Pinto Palmas TO 54,6
Tarcsio Freire de Oliveira e Silva Salvador BA 54,1
Renato Albuquerque Heluy So Lus MA 54,1
Daniel de Oliveira Sampaio Vasconcelos e S Recife PE 54,1
der Leandro da Silva Dantas Natal RN 54,1
Lais Maria Gaspar Coelho So Lus MA 53,6
Gabriel Lima Jurema Teresina PI 53,6
Renanna Najara Veras Rodrigues Teresina PI 53,6
Pedro Henrique Almeida Fraiman Natal RN 53,6
Aaron Moscoso Castro Rgo Aracaj SE 53,6
Luanna Karoline L Nascimento Aracaj SE 53,6
Natally Suzy Anny N. Siqueira Itabaiana SE 53,6
Tiago da Silva Nunes Aracaj SE 53,6
Raphael Medeiros Lima Palmas TO 53,6
Andrews Oliveira Duyprath de Andrade Salvador BA 53,1
Rodrigo dos Santos Macei AL 52,6
ngela Gabriele Gomes Lira Fortaleza CE 52,6
Mariana Camyla Duarte Pontes Fortaleza CE 52,6
Renato Baiochi Alves Veronese Palmas TO 52,6
Lucas Luan Ildefonso da Silva Macei AL 52,1
Gean Carlos Costa e Costa Manaus AM 52,1
Victor de Arajo Rocha Salvador BA 52,1
Programa Nacional Olimpadas de Qumica <<
24
Jos Ivan Ferreira de O. Neto Belm PA 52,1
Lara Caldeira Guedes de Souza Belm PA 52,1
Caroline dos Santos Marques Palmas TO 51,6
Gabriel Lucas A. de Sousa Manaus AM 51,1
Jailson Almeida Pinheiro Jnior Salvador BA 51,1
Victor Hill Saraiva Lima Fortaleza CE 51,1
Kaio Guilherme Andrade Santos So Lus MA 51,1
Safre Torres Campina Grande PB 50,6
Higor Brenner Lima Teresina PI 50,6
Victor Machado Teresina PI 50,6
Gabriel Antnio da Silva Serra Talhada PE 50,6
Ana Carine N. das Chagas Porto Velho RO 50,6
Daniel Ezequiel Francelino Joe Melo Macei AL 50,0
Eduardo Marinho dos Santos So Lus MA 50,0
Joel Gustavo Pinto Oliveira Belm PA 50,0
Matheus Gomes Martins Recife PE 50,0
Carina de Oliveira Gregrio Teresina PI 50,0
XVIII ONNeQ
Una disciplina ao compromisso e ela se transforma em
uma deciso irreversvel de que voc far hoje o que a
maioria no far (mesmo sabendo que deve fazer!), para
que amanh voc possa ter o que a maioria no pode ter!
Zig Ziglar
>> Olimpada Brasileira de Qumica - 2012
25
V OBQ Jr Fase I
V OLIMPADA BRASILEIRA DE QUMICA JNIOR
8 e 9 anos do Ensino Fundamental
Fase I
I NS T R UE S FA S E I
1. A prova consta de 20 questes objetivas, cada uma com quatro alternativas, das quais apenas uma deve
ser assinalada.
2. A prova tem durao de 3 horas.
3. Voc receber o gabarito aps 1 hora do incio da prova, para registrar as suas opes de respostas.
4. A prova tem durao de 3 horas.
5. Voc receber a Folha de Respostas aps 1 hora do incio da prova, para registrar as suas opes.
QUESTESDEMLTIPLAESCOLHA
01.O conceito de elemento qumico mudou ao longo da histria. Atualmente, o
seu signifcado passou a ser sinnimo de
A) substncias simples.
B) qualquer substncia que no sofra decomposio.
C) um conjunto de todos os tomos que possuem o mesmo nmero de prtons.
D) todo conjunto dos elementos bsicos da natureza: ar, gua, fogo e terra.
02. A ilustrao indicada abaixo representa um dos modelos atmicos.
So feitas trs afrmativas em relao a esse modelo.
I Foi proposto por Dalton.
II Considera a existncia de uma partcula menor do que o
tomo e dotada de carga eltrica negativa.
III o modelo aceito atualmente pela comunidade cient-
fca.
Est correto o que se afrma em
A) I, apenas. B) II, apenas,
C) III, apenas. D) I, II e III.
Programa Nacional Olimpadas de Qumica <<
26
V OBQ Jr Fase I
03. Complete o sentido correto para a sentena abaixo.
Eles fcaram surpresos com o que viram: a substncia passou direto do estado slido
para o gasoso! Tio, da eu expliquei... Gente, houve uma
A) Destilao. B) Evaporao. C) Recristalizao. D) Sublimao.
04. Observe a ilustrao indicada a seguir.
Disponvel em: http://www.smartkids.com.br/especiais/
(Acessado em: 11/07/2012)
Qual o ttulo que melhor representa o principal processo mostrado na imagem
acima?
A) Esquema da fotossntese. B) Esquema do efeito estufa.
C) Esquema do aquecimento global. D) Esquema da poluio do ar e da gua
05. Um grupo de estudantes realizou um experimento em uma Feira de Conheci-
mentos utilizando dois lquidos incolores (A e B). Em uma proveta, uma vidraria
volumtrica muito usada em laboratrios de qumica, eles adicionaram um vol-
ume do lquido A. Depois, transferiram para a proveta um mesmo volume do
lquido B. Observando-se a proveta, verifcou-se que os lquidos incolores eram
imiscveis.
Em relao a essa atividade experimental correto afrmar que, considerando-se as
fases observadas, os dois lquidos formaram um(a):
A) soluo saturada. B) mistura heterognea.
C) mistura homognea transparente. D) sistema contendo uma nica fase
>> Olimpada Brasileira de Qumica - 2012
27
V OBQ Jr Fase I
06. O oxignio possui ao nvel do mar ponto de fuso e ponto de ebulio, respec-
tivamente, de -219
o
C e de -183
o
C. Quando um cilindro que contm oxignio
liquefeito aberto em qualquer local do Brasil, em qualquer poca do ano,
temperatura ambiente, ocorre uma
A) destilao do lquido B) fuso para liberao do gs
C) liquefao D) vaporizao
07. Observe o esquema mostrado ao lado. Considerando que
as esferas de tonalidades e de tamanhos diferentes represen-
tam tomos de elementos qumicos distintos, entre as opes
abaixo, esse modelo representa os componentes de um:
A) pedao de cobre B) lquido de um termmetro de mercrio
C) anel de ouro e prata D) gs exalado pelo motor de um automvel
08. O quadro apresentado abaixo indica algumas das funes, no organismo hu-
mano, de elementos qumicos presentes em sais minerais diferentes (I, II, III e IV)
e os alimentos onde eles so normalmente encontrados.
Sais
minerais
Funes Alimentos onde so encontrados
I Componente da hemoglobina e atua na respirao
celular. A sua falta causa anemia.
Fgado, gema de ovo, legumes e vegetais
verdes.
II Componente das molculas de DNA e RNA e tam-
bm participa da formao dos ossos e dos dentes.
Carnes, peixes, feijo, ervilha, cereais, leite
e produtos lcteos.
III Componente dos ossos e dos dentes; participa da
contrao dos msculos e da coagulao do sangue.
Vegetais (brcolis, espinafre, rcula etc.),
leite e produtos lcteos.
IV Muito importante na formao dos ossos e dos
dentes.
gua tratada para consumo humano.
A sequncia que contm os nomes elementos qumicos presentes nesses sais min-
erais e que indica corretamente as suas respectivas suas funes e alimentos onde
so encontrados dada por
A) I Fsforo; II Ferro; III Clcio; IV Flor
B) I Fsforo; II Clcio; III Flor; IV Ferro
C) I Ferro; II Fsforo; III Clcio; IV Flor
D) I Ferro; II Clcio; III Flor; IV Fsforo.
Programa Nacional Olimpadas de Qumica <<
28
09. No laboratrio de cincias de uma escola destilaram-se separadamente quatro
amostras lquidas coletadas: I - da chuva, II - de um rio contaminado, III - de uma
nascente e IV - da torneira desse prprio local. O destilado obtido em todos os
casos era um lquido incolor com PF = 0
o
C, PE = 100
o
C e d = 1,0 g/cm
3
(a 25
o
C).
correto afrmar que nessas condies
A) os destilados obtidos de todas essas amostras se referem a uma mesma sub-
stncia pura composta.
B) a gua da nascente to pura quanto os destilados obtidos das amostras da
chuva, do rio e da torneira.
C) as amostras da gua da torneira e da chuva produzem destilados mais puros do
que as demais amostras.
D) o destilado da amostra do rio contaminado uma substncia mais impura do
que a dos outros destilados.
10. A ilustrao mostrada ao lado traz algumas caracters-
ticas do elemento qumico glio. Com base nos dados
fornecidos por esta imagem, so feitas trs afrmaes.
I. O argnio possui Z = 18.
II. O glio possui 69,723 nutrons.
III. A confgurao eletrnica [Ar] 3d
10
4s
2
4p
1
permite indicar o perodo e o grupo
nos quais se localiza o glio na tabela peridica.
De acordo com os dados fornecidos pela imagem, est correto o que afrmado em
A) I e II B) I e III C) II e III D) I, II e III
11. tomos de sdio (Z=11; [Ne] 3s
1
), potssio (Z = 19; [Ar] 4s
1
) e rubdio (Z=37;
[Kr] 5s
1
) reagem explosivamente quando entram em contato com a gua. Alm
disso, eles combinam-se com o cloro e com o oxignio formando, respectiva-
mente, substncias de frmulas ECl e E
2
O (onde E representa o elemento). Por
sua vez, tomos de magnsio (Z= 12; [Ne ] 3s
2
), clcio (Z=20; [Ar] 4s
2
) e estrncio
(z= 38; [Kr] 5s
2
) tambm reagem com gua, mas no to violentamente quanto
esses outros elementos, e tambm se combinam com o cloro (Z=17; [Ne] 3s
2

3p
5
) e com o oxignio (Z=8; 1s
2
2s
2
2p
4
), porm, formando substncias de fr-
mulas ECl
2
e EO, respectivamente.
V OBQ Jr Fase I
>> Olimpada Brasileira de Qumica - 2012
29
De acordo com os dados apresentados e com as caractersticas da atual tabela dos
elementos qumicos, correto afrmar que
A) as substncias compostas exemplifcadas so slidos covalentes.
B) o magnsio e o cloro so elementos constitudos pelo nenio (Ne).
C) o sdio, o potssio e o rubdio pertencem ao mesmo grupo da tabela peridica.
D) os elementos metlicos citados no texto so do mesmo perodo da tabela peridica.
12. Uma grande indstria de reciclagem resolveu destinar trs depsitos grandes
(I, II e III) para armazenagem de materiais separados em uma etapa preliminar
de catao. Algumas caractersticas das substncias desses materiais a serem
depositadas em cada um dos depsitos esto descritas a seguir.
I. Substncias condutoras de eletricidade tanto no estado slido quanto no lquido.
II. Substncias condutoras de eletricidade no estado lquido, mas no no estado
slido.
III. Substncias com pontos de fuso razoavelmente mais baixos do que as indica-
das para os outros contineres e que no conduzem a corrente eltrica nem no
estado slido nem no lquido.
Considerando tais especifcaes, em qual(is) desses contineres devem ser arma-
zenados, respectivamente, componentes base de ouro retirados de equipamen-
tos de informtica e solventes orgnicos, como a acetona?
A) I, apenas B) II, apenas C) I e III, respectivamente D) II e III, respectivamente.
13. Realizou-se um experimento com uma bebida de colorao laranja apreendida
em uma operao policial. Os seus constituintes eram: essncia, corante e gua.
O objetivo principal da anlise era conseguir obter, de forma isolada, o compo-
nente de maior percentual nesse produto.
Nas condies estabelecidas, qual o processo mais indicado para a obteno dessa
substncia em laboratrio?
A) Centrifugao B) Decantao C) Destilao simples D) Destilao fracionada
14. A moeda de R$ 1 possui um disco interno formado por uma liga de Cu e Ni e
um disco externo composto por alpaca, uma liga metlica de Cu, Ni, Sn e Ag.
V OBQ Jr Fase I
Programa Nacional Olimpadas de Qumica <<
30
Uma moeda de R$ 1 pode ser considerada como um exemplo de
A) mistura metlica slida.
B) objeto com baixa condutividade eltrica.
C) um tipo de material semelhante ao utilizado na fabricao de pregos para usos
comuns.
D) um conjunto de ligas que possui as mesmas propriedades fsicas e qumicas
dos seus elementos qumicos constituintes.

15. Um estudante enumerou trs diferentes iniciativas que ele considerava como
exemplos de qumica verde, conforme mostrado abaixo.
I. Biocombustvel a partir do bagao de cana.
II. Biodiesel a partir de leo usado em lanchonete.
III. Aditivo para massas de pes e biscoitos a partir de solues de baterias.
So exemplos adequados aos princpios da qumica verde o que se afrma em
A) I e II. B) I e III. C) II e III. D) I, II e III.
16. Em um recipiente contendo acetona foram imersos quatro copos: um de
alumnio, um de isopor, um de papel e um de vidro. Verifcou-se uma modif-
cao em apenas um deles, acompanhada pela liberao de gs. Ao fnal, resul-
tou-se em uma pasta, que se tornou slida aps o seu resfriamento.
De acordo com as caractersticas dos materiais desses objetos, qual deles teve o
seu formato alterado pela ao do solvente?
A) Alumnio. B) Isopor. C) Papel. D) Vidro.
17. Observe a imagem abaixo. O
principal aspecto abordado
nessa ilustrao se refere s:
A) misturas heterogneas
B) reaes qumicas.
C) substncias simples.
D) fontes naturais de calorias.
V OBQ Jr Fase I
>> Olimpada Brasileira de Qumica - 2012
31
18. Um pedao de enxofre (S
8
) foi comprado em um armazm e colocado dentro
de uma cisterna. Essa cisterna abastecida com gua fornecida pela compan-
hia de guas e esgoto do municpio. Depois de ser transferido para a cisterna,
esse material pde ser observado dentro do reservatrio por vrios meses.
So feitas trs afrmativas referentes situao acima descrita.
I. O pedao de enxofre constitudo por uma substncia covalente, por isso no
se dissolveu.
II. O material comprado no armazm uma substncia simples, ainda que pu-
desse conter algumas impurezas, e se dissolve lentamente na gua da cisterna.
III. O sistema existente dentro da cisterna, aps um ms da transferncia do enxo-
fre, considerado uma mistura homognea.
Est correto o que se afrma em
A) I, apenas. B) II, apenas. C) I e III. D) II e III.
19. Observe a fgura indicada abaixo.
Disponvel em: http://007blog.net/ (Acessado em: 11/07/2012)
Esta imagem refora um dos alertas ambientais que vm sendo realizado, princi-
palmente quanto aos
A) riscos de reas polares serem destrudas por causa dos biocombustveis.
B) efeitos da chuva cida como uma ameaa para a extino dos ursos polares.
C) perigos dos ursos migrarem para as regies tropicais em busca de mais alimentos.
D) impactos causados nas camadas polares pela infuncia do homem na maior
produo e no maior lanamento de CO
2
na natureza.
20. Uma professora de cincias montou quatro sistemas, como o indicado abaixo
V OBQ Jr Fase I
Programa Nacional Olimpadas de Qumica <<
32
V OBQ Jr Fase I
na fgura abaixo, para realizar um experimento de condutividade eltrica na
sala de aula. A lmpada usada era muito pequena.
Ela testou a condutividade em quatro copos distintos contendo em cada um deles
um lquido diferente: um leo vegetal, uma soluo de acar, uma soluo de
sal de cozinha e gua da torneira. A lmpada fcou acesa em apenas dois desses
sistemas.
De acordo com a composio desses lquidos, a pequena lmpada foi acesa nos
copos contendo:
A) gua da torneira e soluo de sal de cozinha.
B) gua da torneira e leo vegetal.
C) leo vegetal e soluo de acar.
D) soluo de acar e soluo de sal de cozinha.
GABARITO
1 2 3 4 5 6 7 8 9 10
C B D A B D D C A B
11 12 13 14 15 16 17 18 19 20
C C D A A B A B D A
>> Olimpada Brasileira de Qumica - 2012
33
V OLIMPADA BRASILEIRA DE QUMICA JNIOR
8 e 9 anos do Ensino Fundamental
Fase II
I NS T R UE S
1. A prova consta de 10 (dez) questes do tipo mltipla escolha (mximo 40 pontos) e 3 (trs) questes
analtico-expositivas (mximo 60 pontos).
2. Para responder as questes de mltipla escolha, identifque APENAS UMA NICA alternativa correta e
marque a letra correspondente no gabarito existente na Folha de Respostas.
3. Para responder as questes analtico-expositivas, utilize APENAS o espao destinado para cada uma das
trs questes na Folha de Respostas.
4. A prova tem durao de 3 horas.
5. Voc receber a Folha de Respostas aps 1 hora do incio da prova, para registrar as suas opes.
QUESTESDEMLTIPLAESCOLHA
01 Em 2011, o elemento qumico de nmero atmico 112 foi aceito ofcialmente
e passou a integrar a tabela peridica. O reconhecimento veio das unies in-
ternacionais de qumica e fsica puras e aplicadas (respectivamente IUPAC e
IUPAP, siglas em ingls). Alguns tomos desse elemento foram produzidos em
experincias desenvolvidas por cientistas dos laboratrios de Dubna, na Rs-
sia, e Lawrence Livermore, nos EUA. Os tomos do elemento 112 so altamente
instveis e existem por apenas alguns milionsimos de segundo, antes de se
dividirem em tomos de outros elementos.
Adaptado de JC e-mail 3782, de 12 de Junho de 2009
De acordo com as suas caractersticas e propriedades, na tabela peridica, o el-
emento qumico com o nmero atmico 112 se localiza
A) antes do hidrognio.
B) vizinho ao hidrognio.
C) no grupo dos gases nobres.
D) aps o elemento que possui 111 prtons.
V OBQ Jr Fase II
Programa Nacional Olimpadas de Qumica <<
34
02 Os produtores apcolas levam os
quadros contendo favos de mel para
um estabelecimento chamado de
casa do mel. Nesse local, realizada
a extrao, o processamento, o envase
e a estocagem do produto. Aps a re-
tirada da camada de cera que cobre os
favos, os quadros so fxados dentro
de um equipamento, como o mostra-
do na fgura ao lado, no qual ocorre
a etapa I desse processo. Depois de
extrado, o mel passa pela etapa II, na
qual se recomenda o uso de vrias pe-
neiras, com gramaturas diferentes, seguindo a ordem da maior para a menor.
Em nova separao, etapa III, o produto fca em repouso em um tanque, por
aproximadamente 48 h, para que as bolhas, produzidas ao longo do processo,
e as partculas no retiradas na etapa anterior possam ser eliminadas. Em se-
guida, o mel envasado e, depois, rotulado.
Para um processamento adequado do mel, as etapas I, II e III, correspondem re-
spectivamente
A) centrifugao, fltrao e decantao. B) centrifugao, peneirao e aerao.
C) fltrao, peneirao e centrifugao D) fltrao, peneirao e aerao.
03 A maior parte do carbono (Z=6) presente na Terra composta por cerca de
98,9% de carbono-12, algo prximo a 1,1% de carbono-13 e uma quantidade
muito pequena (traos) de carbono-14, uma espcie radioativa. O carbono-14
a espcie investigada em uma tcnica, chamada de tcnica de radiocarbono,
que permite determinar a idade aproximada de diferentes artefatos.
O texto informa que h
A) diferentes tipos de elementos qumicos de carbono na Terra.
B) uma espcie isotpica de carbono com aplicao arqueolgica.
C) diferentes espcies de carbono com diferentes nmeros atmicos.
D) trs formas alotrpicas para o carbono: carbono-12, carbono-13
e carbono-14.
04 Uma fbrica resolveu utilizar gelo seco (CO
2
slido a -78,5 C), armazenado
dentro de sacos plsticos perfurados, em seus carrinhos de vendas de picols e
V OBQ Jr Fase II
>> Olimpada Brasileira de Qumica - 2012
35
sorvetes. A deciso aconteceu depois de se confrmar que a baixa temperatura
do gs liberado pelo gelo seco mantm os alimentos congelados dentro dos
carrinhos, ao longo do perodo indicado pela empresa para a comercializao
desses produtos na rua.
Em relao s informaes contidas no texto acima, CORRETO afrmar que
A o gs liberado dos sacos plsticos constitudo basicamente por vapor dgua.
B) o gelo seco sofre sublimao e o gs frio de CO
2
retarda o degelo dos produtos.
C) o gs produzido no carrinho o resultado de um fenmeno qumico, uma
reao, que ocorre entre o gelo seco e o oxignio.
D) o gelo seco formado por uma mistura contendo duas substncias diferentes,
uma lquida e outra gasosa, que mantm a refrigerao nos carrinhos.
05 Mineradores utilizam mercrio
para separar o ouro. Nesse
processo, forma-se uma liga
slida, com o teor de ouro nor-
malmente de 30 a 50%. Para
separar o ouro do mercrio,
comumente feita a pirlise
do mercrio. Um processo in-
dicado para essa pirlise a
retortagem do amlgama, uti-
lizando-se uma retorta, como
o equipamento ilustrado ao
lado, que foi desenvolvido no
CETEM/MCT. Essa retorta proporciona uma recuperao total do ouro amal-
gamado e do mercrio utilizado no processo.
Adaptado de: http://www.cetem.gov.br/publicacao/CTs/CT2007-071-00.pdf (Acesso em: 01/10/2012)
Na retortagem de amlgama de ouro e mercrio ocorre o processo de
A) cristalizao do mercrio. B) destilao do mercrio.
C) reao do mercrio. D) sublimao do mercrio.
V OBQ Jr Fase II
Programa Nacional Olimpadas de Qumica <<
36
06 Em temperatura ambiente e presso atmosfrica normal (uma atmosfera), o
hidrognio (H
2
) se encontra em estado gasoso, mas caso seja resfriado se liq-
uefaz. Em presso atmosfrica, isso ocorre a -253 C.
Sabendo-se que o hidrognio tem Z=1, CORRETO afrma que
A) a ligao covalente de cada molcula de H
2
rompida quando essa substncia
muda de estado lquido para gasoso.
B) as ligaes covalentes das molculas de hidrognio so fracas porque essa
substncia apresenta baixo ponto de ebulio.
C) as ligaes covalentes do H
2
envolvem uma transferncia total de eltrons, fa-
zendo que esse gs tenha uma temperatura de ebulio muito baixa.
D) a ligao covalente de cada molcula de H
2
apresenta um compartilhamento
de dois eltrons entre os dois tomos de hidrognio e assim se mantm quan-
do a substncia liquefeita.
07 Uma forma de se testar a veracidade de uma pedra de diamante tentando
risc-lo com uma barra de metal contendo um pequeno cristal de diamante
verdadeiro fxo na sua ponta. Caso o diamante seja verdadeiro, no ser pos-
svel observar o aparecimento de um risco sobre o mesmo. Nesse caso, embora
possa conter um pequeno teor de impurezas, basicamente, a pedra constitu-
da por tomos de
A) Ag B) Au C) C D) Ca
08 Realizou-se um experimento, no qual um frasco de vidro contendo cubos de
gelo, retirado do freezer, sofreu um aquecimento, sob presso constante de 1
atm, conforme ilustrado a seguir.
-10
o
C 0
o
C 0
o
C 100
o
C 100
o
C 105
o
C

V OBQ Jr Fase II
>> Olimpada Brasileira de Qumica - 2012
37
Qual dos grfcos abaixo apresenta uma curva de aquecimento adequada para
esse processo?
A B C D
09 Muitos trabalhadores percorrem diariamente vrios quilmetros at a cratera de
um vulco indonsio para buscar pedras amarelas, constitudas basicamente por
uma nica substncia simples. Eles vendem esse material para fbricas locais, que
purifcam essa substncia e a utilizam para refnar acar e produzir fsforos e
remdios. Coletar esse ouro amarelo tem o seu preo. Apesar de a remunerao
diria de U$ 8,0 (oito dlares) ser muito maior do que a de agricultores, a profsso
muito mais perigosa. Os operrios trabalham no entorno de um enorme lago no
interior da cratera, que considerado um dos mais cidos do mundo, e onde h
emisses de gases txicos, que j mataram muitos catadores dessas pedras.
Adaptado de: http://www.dailymotion.com/video/xht63x_ouro-amarelo_news (Acesso em: 01/10/2012)
Em relao principal substncia abordada no texto acima, CORRETO afrmar
que ela
A) um sal. B) constituda pelo elemento qumico Au.
C) o enxofre. D) formada por dois elementos qumicos.
10 Baseando-se em experimentos com radioatividade, o cientista Ernest Ruther-
ford props um modelo atmico.
Qual das imagens apresentadas a seguir corresponde a esse modelo?
A B C D
V OBQ Jr Fase II
Programa Nacional Olimpadas de Qumica <<
38
QUESTESANALTICO-EXPOSITIVAS
11 A queima de um pedao de fta de magnsio ocorre segundo a equao da
reao qumica mostrada abaixo. Nesse processo, ocorre liberao de energia
na forma de calor e de uma intensa luminosidade.
2 Mg + O
2
2 MgO
Utilizando-se essa reao, qual a massa de magnsio necessria para se produzir
2,0 g de xido magnsio?
Dados de massas (molares): C = 12 g/mol; O = 16 g/mol; Mg = 24 g/mol.
12 Com base nos princpios da qumica verde, avalie as vantagens da utilizao do
equipamento ilustrado na questo 05 em um garimpo de ouro.
13 Discuta se o ar uma soluo e se possvel separar alguns dos seus principais
componentes.
GABARITO-QUESTESDEMLTIPLAESCOLHA
01 02 03 04 05 06 07 08 09 10
D A B B B D C A C B
POSSVEISRESPOSTAS-QUESTESANALTICOEXPOSITIVAS
11 1,2 g de magnsio.
12 Essa tcnica reduz ou elimina a gerao de produtos e sub-produtos txicos,
que so nocivos sade humana ou ao ambiente, pois os constituintes da
amlgama so totalmente recuperados.
13 O ar uma soluo, uma mistura homognea. Os seus constituintes podem ser
liquefeitos e, depois, separados por destilao fracionada.
V OBQ Jr Fase II
>> Olimpada Brasileira de Qumica - 2012
39
V OLIMPADA BRASILEIRA DE QUMICA JNIOR
RESULTADO
Aluno Escola Cidade UF ANO NOTA
O U R O
Rodrigo Silva Ferreira Militar de Salvador Salvador BA 9o 100
Maria Eduarda Belota Moreno Olimpo DF Braslia DF 9o 100
Andr Saugo Mazzari Notre Dame de Campinas Campinas SP 8o 99
Joo Martins Cortez Filho Dom Barreto Teresina PI 9o 96
Pedro Henrique R. de Freitas Militar de Braslia Braslia DF 9o 95
Pedro Oscar Souza Livera Esc. Internacional de Aldeia Camaragibe PE 9o 95
Joo Pedro Ramos Milhome Dom Barreto Teresina PI 9o 95
PRATA
Leonardo Oliveira Reis Anchieta Salvador BA 9o 93
Letcia Pereira de Souza Objetivo Junior Taubat SP 9o 93
Fernanda Gonalves Fidncio EEEB Gov. Andr F. Montoro Valinhos SP 9o 92
Victria Moreira Reis Cogo Dom Barreto Teresina PI 9o 91
Larissa Narumi Takeda Militar de Porto Alegre Porto Alegre RS 9o 91
Vincius de Camargo Fund. Univer. Vida Crist Mococa SP 9o 91
Gustavo Francisco de Oliveira Fund. Univer. Vida Crist Mococa SP 9o 91
Thiago Ferreira Kalife Esc. Interamrica Goinia GO 9o 90
Larissa Malanski Chimin Marista de Cascavel Cascavel PR 9o 90
Izadora Bandoch Militar de Curitiba Curitiba PR 9o 90
Francisco Anderson S. de Moura Dom Barreto Teresina PI 9o 90
Hugo Concolato de O. Freitas Sistema Elite de Ensino Rio de Janeiro RJ 9o 90
Aridne Garcia Leite Militar de Porto Alegre Porto Alegre RS 9o 90
Laura Jardim Chiarelli EMEF Prof. Arlinda R. Negri Dumont SP 9o 89
Wesley Marques Gonalves Esc. de Aplicao do Recife Recife PE 9o 88,5
Igor Radel Ribeiro Antonio Vieira Salvador BA 9o 88
V OBQJr Resultado
Programa Nacional Olimpadas de Qumica <<
40
Joo Victor Resende Carvalho E M Mario Faustino Teresina PI 9o 88
Arthur Augusto Siqueira Carvalho Dom Barreto Teresina PI 9o 88
Daniel Souza de Almeida EEEM Demtrio Ribeiro Alegrete RS 9o 88
Richard William R. A. dos Santos EMEF Prof. Arlinda R. Negri Dumont SP 9o 88
Gabriel Pereira Penna Andrade FIDE Itabira MG 9o 87,5
Gabriel Bertoni Lugtenburg Classe A Porto Velho RO 9o 87
Elcio Koodiro Yoshida Etapa So Paulo SP 9o 87
BRONZE
Nayara Lima Rocha da Cruz Anchieta Salvador BA 9o 86
Lia de Freitas Arajo Alves Farias Brito Fortaleza CE 9o 86
Vitria Barros Gomes Dom Barreto Teresina PI 9o 86
Giuliano Pantarotto Semente Inst. Educ. Parque Ecolgico Indaiatuba SP 9o 86
Andr Gandelman Bovolini Bandeirantes So Paulo SP 9o 86
Marcio Vinicius D. Ross EE Prof Azarias Mendes Votorantim SP 9o 86
Gabriel M. de Oliveira Santos EE Prof Azarias Mendes Votorantim SP 9o 86
Samuel Domingues Beato EE Prof Azarias Mendes Votorantim SP 9o 86
Deivid Dda Arajo Nunes Amadeus Aracaju SE 9o 86
Giovanna Soares Nutels Contato Macei AL 9o 85
Seon Augusto de S. Ferreira Militar de Salvador Salvador BA 9o 85
Leonardo Patio Gonalves Esc. Interamrica Goinia GO 9o 85
Jos Henrique Carvalho Militar de Curitiba Curitiba PR 9o 85
Marcrio Barcellos Gessinger Militar de Porto Alegre Porto Alegre RS 9o 85
Bento Bruno Pereira Militar de Porto Alegre Porto Alegre RS 9o 85
Las Yoshie Morikawa Muta Mater Amabilis Guarulhos SP 9o 85
Rafael da Costa Araripe Petrpolis So B. do Campo SP 9o 85
Lucas Jun Koba Sato Bandeirantes So Paulo SP 9o 85
Susanna Tawata Tamachiro Exatuse So Paulo SP 9o 85
Gabriel Moura Brana Ari de S Cavalcante Fortaleza CE 8o 85
Gustavo Pereira de Souza So Francisco Xavier Ipatinga MG 9o 84,5
V OBQJr Resultado
>> Olimpada Brasileira de Qumica - 2012
41
Thalita Micaelle Lira da Luz Patricia Costa Olinda PE 8o 84,5
Lucas Tadeu Rocha Santos Farias Brito Sobralense Sobral CE 9o 84
Valentina de M. Cezar de Arajo Diocesano Leo XIII Paranagu PR 9o 84
Gustavo Guedes Faria Poliedro So Jos dos
Campos
SP 9o 84
Gabriela do Couto Fernandes Olimpo DF Braslia DF 9o 83,5
Gabriela F. Borges da Costa Profa. Julieta M. dos Santos Coxim MS 9o 83,5
Hlio Luigi Crspo Neto Esc. de Aplic. do Recife Recife PE 9o 83,5
Bruno Felipe Santos de Oliveira Esc. Batista Biblica Salvador BA 9o 83
Lauro de Arajo Costa Moura Farias Brito Fortaleza CE 9o 83
Eliabe Bastos Dias Sete de Setembro Fortaleza CE 9o 83
Heron Almeida Medeiros Filho Dom Barreto Teresina PI 9o 83
Ralph Biaggi Neto Anchieta - Objetivo Ribeiro Preto SP 9o 83
Cludio Ado da Cunha EMEF Profa. Mercedes Klein So Jos dos
Campos
SP 9o 83
Leonardo Henrique M. Florentino Objetivo Vergueiro So Paulo SP 9o 83

A lista de alunos agraciados com MENO HONROSA e os DEMAIS CLASSIFICADOS na
V Olimpada Brasileira de Qumica Jnior encontra-se disponvel em www.obquimica.org/
V OBQJr Resultado
Programa Nacional Olimpadas de Qumica <<
42
Olimpada Brasileira
de Qumica - 2012
MODALIDADE A ( 1 e 2 anos )
PARTE A - QUESTES MLTIPLA ESCOLHA
Texto para as questes 1, 2 e 3
Certo xido foi dissolvido em gua dando origem a uma soluo incolor. Borbu-
lhou-se gs carbnico atravs da soluo sendo observada a formao de preci-
pitado branco. A mistura foi levada a uma centrfuga e separou-se o slido do fl-
trado. Ao slido foi acrescentado 5,0 mL de uma soluo de cido clordrico 10%
(m/v). Observou-se a liberao de gs e, ao fnal do processo, o slido foi inteira-
mente consumido.
QUESTO 1
Dentre os elementos abaixo, o nico que forma um xido com as caractersticas
descritas no experimento o
a) alumnio b) brio
c) enxofre d) fsforo e) potssio.
QUESTO 2
A frmula qumica do gs liberado ao acrescentar cido clordrico ao slido
a) H
2
b) Cl
2
c) O
2
d) CO
2
e) H
2
O
QUESTO 3
A quantidade de matria, em mol, de cido clordrico adicionada foi de
a) 1,4 x 10
2
b) 4,2 x 10
2

c) 1,8 x 10
1
d) 2,7 x 10
1
e) 5,0 x 10
1
A
OBQ-2012 Fase III
>> Olimpada Brasileira de Qumica - 2012
43
QUESTO 4
A queima de 1,6163 g uma substncia lquida formada apenas por C, H e O em
um laboratrio de Qumica formou 1,895 g de H
2
O e 3,089 g de CO
2
. Com base nas
informaes, podemos concluir que a formula emprica da substncia queimada :
a) CH
4
O b) C
3
H
6
O
2
c)C
2
H
4
O
2
d) C
2
H
6
O e) C
2
H
4
O
QUESTO 5
O Cloreto de Hidrognio, HCl(g), um gs incolor, irritante, corrosivo e altamente
txico temperatura ambiente. Suponha que o Cloreto de Hidrognio seja um gs
ideal, qual o tipo de interao ocorreria entre as molculas de Cloreto de Hidrognio?
a) Dipolo- Dipolo b) Dipolo- Induzido
c) Ligao de Hidrognio d) Foras de London
e) Nenhuma das alternativas
QUESTO 6
Um acidente em um laboratrio provocou a intoxicao de um grupo de pessoas
por inalao de um gs. Um analista coletou uma amostra desse gs e a introduziu
em um recipiente inelstico de 1 dm
3
, temperatura de 27 C. A amostra de gs
contida no recipiente pesou 1,14 g e a presso medida no recipiente foi de 1 atm.
Assim, pode-se afrmar que este gs :
a)CO b)C
2
H
2
c)H
2
S d)NO e)NO
2

Dados: R = 0,082 atm.l.K
-1
.mol
-1
.
QUESTO 7
Na reao de Landolt, uma soluo de iodato de potssio adicionada a uma solu-
o acidifcada de bissulfto de sdio contendo amido. Um dos produtos gerados
continua a reagir com o bissulfto. Quando este totalmente consumido, forma-se
um on. Por fm, um complexo azul ser formado com o amido e a espcie inica.
Sobre este on, assinale a alternativa que indica a hibridizao do tomo central e
a sua geometria molecular.
a) sp; linear b) sp
2
; angular c) sp
3
d; bipirmide trigonal.
d) sp
3
d; linear e) sp
2
; linear.
Modalidade A
Programa Nacional Olimpadas de Qumica <<
44
QUESTO 8
Um metal M dissolvido em cido clordrico concentrado e observa-se a liberao
de um gs A. soluo obtida foi adicionada tioacetamida formando um precipi-
tado amarelo. A soluo foi centrifugada e o sobrenadante separado. Ao sobrena-
dante foi adicionada uma soluo de nitrato de prata e percebeu-se a precipitao
de um slido B branco. As espcies M, A e B so, respectivamente:
a) Cu, gs hidrognio e cloreto de prata.
b) Cd, gs oxignio e clorato de prata.
c) Co, gs hidrognio e cloreto de prata
d) Cd, gs hidrognio e cloreto de prata.
e) Cu, gs oxignio de cloreto de prata.
QUESTO 9
Um estudante, a pedido de seu professor, precisa preparar 400 mL de uma soluo
de amnia 5 mol/L. No rtulo do frasco de amnia, lacrado, que utilizar para pre-
parar sua soluo, o estudante observou as seguintes informaes:
Concentrao (m/m): 29,0%
Densidade: 0,9 g.cm
-3
Massa molar: 17,02 g.mol
-1
A partir dessas informaes, deduz-se que o volume de soluo concentrada, me-
dida pelo estudante, para preparar a soluo solicitada pelo professor foi de:
a) 86,00 mL b) 94,15 mL c)112,03 mL d) 130,42 mL e) 145,31 mL
QUESTO 10
A decomposio do N
2
O
4
em NO
2
dada pela seguinte reao:
N
2
O
4
2NO
2
Coloca-se n mols de N
2
O
4
em um recipiente de presso p e temperatura T e espera-
-se o equilbrio ser atingido. Sabendo que o grau de decomposio , a constante
de equilbrio Kc pode ser expressa como:
a)
Kc =
2a
pRT (n-a)
2
b)
Kc =
2a
pRT (n-a)
2
c)
Kc =
a
4pRT (n
2
+a
2
)
d)
Kc =
4pa
[(RT) (c+a)]
2
e)Kc =
4apRT
(n
2
-a
2
)
OBQ-2012 Fase III
>> Olimpada Brasileira de Qumica - 2012
45
Modalidade A
PARTE B - QUESTES ANALTICO-EXPOSITIVAS
QUESTO 11
Em qumica analtica, possvel determinar satisfatoriamente a concentrao de me-
tais em soluo usando agentes complexantes. O agente quelante mais utilizado o
EDTA, visto que ele reage com os ctions metlicos em uma proporo bem defnida
de 1:1. O EDTA um cido poliprtico com quatro prtons ionizveis, cuja estrutura
pode ser simplifcada pela frmula H
4
Y. Titulaes com EDTA so feitas comumente
em solues tamponadas de pH = 10, para que no haja competio entre os ons
metlicos e os ons H
+
, garantindo a formao de um complexo estvel. Uma das gran-
des utilidades do uso de EDTA para a determinao de clcio e magnsio. As reaes
de complexao e suas respectivas constantes de equilbrio so apresentadas abaixo.
Mg
2+
(aq) + Y
4-
(aq) MgY
2-
(aq) Kf = 4,9 x 10
8
Ca
2+
(aq) + Y
4-
(aq) CaY
2-
(aq) Kf = 5,0 x 10
10
No processo de titulao utilizado o indicador negro de Ericromo T (H
3
In em sua
forma protonada), que forma complexos de cor vermelho-vinho com os metais em
soluo. Em pH = 10, quando deslocado pelo EDTA, o indicador encontra-se na for-
ma HIn- de cor azul. Logo, o fnal da titulao tido quando a soluo assume uma
colorao azul indicando o excesso do indicador livre. O grfco para a titulao
complexomtrica de clcio e magnsio, em pH = 10, apresentado a seguir.
Programa Nacional Olimpadas de Qumica <<
46
Para se determinar a concentrao de uma soluo de Ca
2+
, foi preparada uma so-
luo de EDTA dissolvendo Na
2
H
2
Y . H2O em gua e completando o volume do
balo at 250 mL. Como a concentrao de EDTA era desconhecida, foi usada uma
soluo de Mg
2+
de concentrao 0,0050 M para padronizao. O volume gasto na
padronizao de 25,0 mL da soluo de EDTA foi de 18,5 mL da soluo de Mg
2+
.
Antes de iniciar a titulao da soluo clcio, 50,0 mL dessa soluo foram mistu-
rados com 50,0 mL da soluo de magnsio utilizada na padronizao do EDTA. A
nova soluo foi diluda em balo volumtrica at o volume de 500 mL. Uma al-
quota de 50 mL foi ento tamponada em pH = 10 e titulada com a soluo de EDTA
gastando 9,7 mL para que a soluo fcasse azul.
I) Explique analiticamente o porqu da adio de magnsio soluo de clcio
antes da titulao.
II) Calcule a concentrao de Ca
2+
da soluo inicial. Expresse o resultado em
mol.L
-1
e em ppm.
QUESTO 12 - Equilbrio cido-base.

O grfco abaixo representa o progresso da titulao do aminocido histidina com
equivalentes de NaOH.
Histidina
I) Apresente todos os equilbrios de ionizao relevantes para a histidina, indican-
do para cada um deles o pK relacionado. Indique, tambm, as zonas de maior
capacidade tamponante para este aminocido.
OBQ-2012 Fase III
>> Olimpada Brasileira de Qumica - 2012
47
Modalidade A
II) Para reproduzir o meio intracelular em laboratrios de bioqumica, tampes de
fosfato so utilizados. Considerando que o pH intracelular seja igual a 7,4 e que
a soluo utilizada para o preparo do tampo tenha [PO
4
3-
] = 0,01 mol.L
-1
, calcu-
le o volume de HCl 6,00 mol.L
-1
que deve ser adicionado a 500 mL dessa soluo,
para obteno da soluo desejada.
Dados: pK
a
(H
3
PO
4
) = 2,15 pK
a
(H
2
PO
4
-) = 7,1 pK
a
(HPO
4
2-
) = 12,4
QUESTO 13
Uma fbrica que produz cal ( CaO ) necessita reduzir o custo de produo para se
manter no mercado com preo competitivo para o produto. A direo da fbrica
solicitou ao departamento tcnico o estudo da viabilidade de reduzir a tempera-
tura do forno de calcinao de Carbonato de Clcio dos atuais 1500K para 800K.
I) Considerando apenas o efeito termodinmico, pergunta-se: O departamento
tcnico pode aceitar a nova temperatura de calcinao?
II) Em caso afrmativo, o departamento tcnico pode fornecer outra temperatura
de operao que proporcione maior economia?
III) Em caso negativo, qual a temperatura mais econmica para se operar o forno
de calcinao?
Dados a 25C
Substncia DS / J . mol
-1
. K
-1
DH
0
/ kJ . mol
-1
CaCO3(s) 92,9 - 1 206,9
CaO(s) 39,8 -635,1
CO2(g) 213,6 -393,5
OBS: desconsidere a variao das propriedades com a temperatura

QUESTO 14
Uma grande diferena entre os elementos do segundo perodo para os demais a
falta de capacidade de formar um grande nmero de ligaes qumicas. So obser-
vados molculas ou ons como o SiF
6
2
, PF
6
-
e SF
6
, mas nenhum anlogo observa-
do para carbono, nitrognio ou oxignio.
I) Utilizando de conceitos da Teoria da Ligao de Valncia, explique por que os
elementos silcio, fsforo e enxofre podem fazer um maior nmero de ligaes
que o mximo possvel para carbono, nitrognio ou oxignio.
Programa Nacional Olimpadas de Qumica <<
48
Alm do SF
6
, o enxofre forma uma vasta srie de compostos com o for: S
2
F
2
, SOF
2
,
SF
4
, SOF
4
e o S
2
F
10
.
II) Existem dois compostos com frmula qumica S
2
F
2
, um dos exemplos de iso-
meria mais simples da qumica inorgnica. Escreva a estrutura de Lewis para os
dois ismeros.
As molculas SF
4
e SOF
4
possuem igual nmero pares de eltrons ao redor do to-
mo central, para esses pares est prevista uma geometria de bipirmide trigonal.
III) Represente espacialmente o arranjo bipirmide de base trigonal e identifque as
posies axiais (ax) e equatorais (eq) em sua representao. Defna os ngulos te-
ricos formados entre as posies equatoriais e entre uma axial e uma equatorial.
IV) O SOF
4
tem, obviamente, uma ligao diferente das demais. Represente essa
molcula considerando o seu arranjo espacial e explique a sua escolha para a
posio dessa ligao.
V) Entre as molculas SF
4
e XeF
4
, qual apresentar o menor ngulo entre as liga-
es? Justifque sua resposta.
QUESTO 15
A amnia, nas condies ambientes, um composto gasoso, usado como matria-
-prima para diversas substncias, por exemplo, na fabricao de fertilizantes agr-
colas, explosivos para fns militares, gs de refrigerao, etc.
preparada atravs de sntese direta com gs hidrognio (processo Haber-Bosch).
Na fabricao de fertilizantes e de explosivos, usa-se um sal, obtido a partir da sua
reao com cido ntrico.
Com relao amnia:
I) Qual a sua geometria molecular?
II) Escreva a equao de ionizao que ocorre, quando dissolvida em gua, ci-
tando o nome comercial da soluo obtida.
III) Ao entrar em contato com gs clordrico, produz um determinado sal. Qual a
cor que a soluo aquosa desse sal desenvolver, na presena de fenolftalena?
Explique.
IV) Escreva as reaes de sntese da amnia e da formao de seu sal, conforme
texto acima.
OBQ-2012 Fase III
>> Olimpada Brasileira de Qumica - 2012
49
Modalidade A
QUESTO 16
Em uma estao padro de tratamento de gua para consumo humano, a gua,
aps sua captao de um rio ou represa, passa pela seguinte sequencia de proces-
sos:
1. Adio de sulfato de alumnio para reagir com a alcalinidade da gua e agregar
as impurezas dissolvidas e em suspenso na gua.
2. Processo de agitao lenta (mistura lenta) da gua para aumentar o tamanho
das partculas formadas no processo anterior.
3. Processo de separao por sedimentao das partculas formadas nos proces-
sos anteriores fcando a gua superfcial lmpida.
4. Processo destinado a remover partculas em suspenso em meio fltrante cons-
titudo de areia.
5. Processo no qual utilizado cloro para matar os microorganismos patognicos.
6. Processo em que adicionado cido fuorsilcico.
7. Adio de uma suspenso de cal hidratada para eliminar a acidez da gua.
I) Identifque cada um desses processos
II) Escreva a frmula do cido fuorsilcico (cido hexafuorossilcico)
III) Segundo norma do Ministrio da Sade, o valor mximo permitido de fuore-
to em gua para consumo humano de 1,5 mg/L. Considerando que o cido
furossilsico, adiconado gua, utilizado na forma de uma soluo aquosa a
23%, com densidade igual a 1,19 g/mL, e que todo o for presente disponibi-
lizado na forma de fuoreto, calcule o volume mximo dessa soluo que pode
ser adicionado a cada m
3
de gua para consumo humano.
Programa Nacional Olimpadas de Qumica <<
50
B
Olimpada Brasileira
de Qumica - 2012
MODALIDADE B ( 3 ano )
PARTE A - QUESTES MLTIPLA ESCOLHA
QUESTO 1
A queima de 1,6163 g uma substncia lquida formada apenas por C, H e O em
um laboratrio de Qumica formou 1,895 g de H
2
O e 3,089 g de CO
2
. Com base nas
informaes, podemos concluir que a formula emprica da substncia queimada :
a) CH
4
O b) C
3
H
6
O
2
c) C
2
H
4
O
2
d) C
2
H
6
O e) C
2
H
4
O
QUESTO 2
O Cloreto de Hidrognio, HCl(g), um gs incolor, irritante, corrosivo e altamente
txico temperatura ambiente. Suponha que o Cloreto de Hidrognio seja um gs
ideal, qual o tipo de interao ocorreria entre as molculas de Cloreto de Hidrognio?
a) Dipolo- Dipolo b) Dipolo- Induzido
c) Ligao de Hidrognio d) Foras de London
e) Nenhuma das alternativas
QUESTO 3
Na reao de Landolt, uma soluo de iodato de potssio adicionada a uma solu-
o acidifcada de bissulfto de sdio contendo amido. Um dos produtos gerados
continua a reagir com o bissulfto. Quando este totalmente consumido, forma-se
um on. Por fm, um complexo azul ser formado com o amido e a espcie inica.
Sobre este on, assinale a alternativa que indica a hibridizao do tomo central e
a sua geometria molecular.
a) sp; linear b) sp
2
; angular. c) sp
3
d; bipirmide trigonal.
d) sp
3
d; linear e) sp
2
; linear.
OBQ-2012 Fase III
>> Olimpada Brasileira de Qumica - 2012
51
Modalidade B
QUESTO 4
Um metal M dissolvido em cido clordrico concentrado e observa-se a liberao
de um gs A. soluo obtida foi adicionada tioacetamida formando um precipi-
tado amarelo. A soluo foi centrifugada e o sobrenadante separado. Ao sobrena-
dante foi adicionada uma soluo de nitrato de prata e percebeu-se a precipitao
de um slido B branco. As espcies M, A e B so, respectivamente:
a) Cu, gs hidrognio e cloreto de prata b) Cd, gs oxignio e clorato de prata.
c) Co, gs hidrognio e cloreto de prata d) Cd, gs hidrognio e cloreto de prata.
e) Cu, gs oxignio de cloreto de prata.

QUESTO 5
1) Um estudante, a pedido de seu professor, precisa preparar 400 mL de uma solu-
o de amnia 5 mol/L. Ao olhar frasco lacrado de amnia que utilizar para produ-
zir sua soluo, o estudante observou as seguintes informaes:
Concentrao (m/m): 29,0%
Densidade: 0,9 g.cm
-3
Massa molar: 17,02 g.mol
-1
A partir das informaes, o volume de soluo concentrada medida pelo estudante
para preparar a soluo solicitada pelo professor foi de:
a) 86,00 mL b) 94,15 mL c)112,03 mL d) 130,46 mL e) 145,31 mL
QUESTO 6
A decomposio do N
2
O
4
em NO
2
dada pela seguinte reao:
N
2
O
4
2NO
2
Coloca-se n mols de N
2
O
4
em um recipiente de presso p e temperatura T e espera-
-se o equilbrio ser atingido. Sabendo que o grau de decomposio , a constante
de equilbrio Kc pode ser expressa como:
a)
Kc =
2a
pRT (n-a)
2
b)
Kc =
4pa
2
RT (n
2
-a
2
)
c)
Kc =
a
4pRT (n
2
+a
2
)
d)
Kc =
4pa
[(RT) (n+a)]
2
e)Kc =
4apRT
(n
2
-a
2
)
Programa Nacional Olimpadas de Qumica <<
52
QUESTO 7
O actnio
228
88
Ac possui um tempo de meia-vida igual a 6,13 horas bem prximo ao
tecncio-99m considerado um radioistopo ideal para o uso de diagnstico para a
determinao de doenas . Sua equao de decaimento a seguinte:
228
88
Ac
0
-1
+
228
90
Th. Com relao a essas informaes e aos fenmenos radioativos, so
feitas as afrmaes a seguir:
I) o tempo necessrio para que uma massa de m
0
de actnio se reduza para 3/8
de sua massa inicial (m
0
) so cinco horas e quarenta e sete segundos. Considere
log 2 = 0,301 e log 3 = 0,477.
II) alm das desintegraes radioativas envolvendo partculas e radiao eletro-
magntica, foi observado outro fenmeno nuclear denominado de captura
eletrnica. Este fenmeno consiste na captura de um eltron extranuclear. O
eltron capturado reage com um prton formando um nutron. Assim, a carga
do istopo diminui em uma unidade e a massa aumenta em uma unidade, j
que o nutron tem massa;
III) so necessrias a emisso de oito partculas alfa e 3 partculas beta para que o
decaimento do actnio promova o aparecimento do istopo de platina;
IV) a primeira lei de Soddy aborda que ao emitir uma partcula alfa o istopo ra-
dioativo ir desintegrar um nutron para que haja a perda de quatro unidades
de massa e duas unidades de carga do istopo;
V) diferentemente das partculas alfa e beta a radiao gama de natureza eletro-
magntica e seu poder de penetrao alto de modo a ser nocivo para o ser
humano;
VI) ao emitir radiao gama o
228
88
Ac se desintegra em outro elemento diferente.
Dessas afrmaes, so corretas:
a) I, IV e VI; b) III e VI; c) I e V; d) II e V; e) I, III e IV;
QUESTO 8
Abaixo so apresentados os potenciais padro para as redues dos seguintes xi-
dos metlicos:
N
2
O
3
(s) NiO(s) E=0,4 V
FeO(s) Fe(s) E=-0,87V
OBQ-2012 Fase III
>> Olimpada Brasileira de Qumica - 2012
53
Modalidade B
O potencial de uma pilha pode ser relacionado com potencial padro das semi-
-reaes pela Equao de Nerst:
E = E
0
- lnQ
RT
nF
Com base nos dados, a nica opo correta :
a) A reao depende do pH.
b) Dissolvendo FeO em excesso de cido ntrico concentrado e basifcando a solu-
o, posteriormente, obtm-se Fe(OH)
2
.
c) Sabendo que NiO possui a mesma estrutura cristalina que NaCl, pode-se afr-
mar que o nmero de coordenao do Ni 8.
d) O xido de Ni(III) uma espcie diamagntica.
e) A adio de alumnio metlico ao sistema forma uma pilha de potencial E =
2,06V. E(Al
3+
/Al
0
) = -1,66V.
QUESTO 9
Um mecanismo de reao bastante estudado em qumica orgnica a substitui-
o nucleoflica de 1 ordem (SN
1
). Esse um mecanismo de reao que envolve
a formao de carboctions na etapa determinante da reao. Esse tipo de reao
caracterizado como SN
1
porque a etapa de formao do carboction, que deter-
mina a velocidade da reao, unimolecular, pois, s envolve uma espcie em sua
formao. Dada a reao abaixo, responda:
H
3
C-C-Br + OH
-
H
3
C-C-OH+Br
-
C
3
CH
3
CH
3
CH
3

a) A lei de velocidades para esse o consumo do brometo de terc-butila dado por
V = k[(CH
3
)
3
CBr][OH-].
b) Como a formao do carboction a etapa determinante da reao, podese
afrma que V = k[(CH
3
)
3
C+][OH-].
c) A velocidade de reao dada por V = k[(CH3)3CBr]. Correta, pois, como a etapa
determinante s depende da formao do carboction, a velocidade da reao
Programa Nacional Olimpadas de Qumica <<
54
que determinada ela etapa lenta depende apenas da concentrao do bro-
meto de terc-butila.
d) Um grfco da concentrao de brometo de terc-butila em funo da velocida-
de uma reta com coefciente angular igual k[OH-].
e) O oxignio na molcula de lcool terc-butilico tem hibridizao sp pois faz ape-
nas duas ligaes, uma com o carbono e outra com o hidrognio.
QUESTO 10
cido benzico reage com o composto A produzindo um cloreto de acila que, por
sua vez, reage com etilamina formando o composto orgnico B. Os compostos A e
B so, respectivamente:
a) HCl e N-etilbenzamida.
b) PCl
3
e benzoato de etila.
c) SOCl
2
e anidrido benzoico.
d) SOCl
2
e N-etilbenzamida.
e) PCl
3
e anidrido p-etilbenzamida.
PARTE B - QUESTES ANALTICO-EXPOSITIVAS
QUESTO 11
Determinao de clcio por titulao complexomtrica usando EDTA.
Em qumica analtica, possvel determinar satisfatoriamente a concentrao de
metais em soluo usando agentes complexantes. O agente quelante mais utiliza-
do o EDTA, visto que ele reage com os ctions metlicos em uma proporo bem
defnida de 1:1. O EDTA um cido poliprtico com quatro prtons ionizveis, cuja
estrutura pode ser simplifcada pela frmula H
4
Y. Titulaes com EDTA so feitas
comumente em solues tamponadas de pH = 10, para que no haja competio
entre os ons metlicos e os ons H+, garantindo a formao de um complexo est-
vel. Uma das grandes utilidades do uso de EDTA para a determinao de clcio e
magnsio. As reaes de complexao e suas respectivas constantes de equilbrio
so apresentadas abaixo.
Mg
2+
(aq) + Y
4-
(aq) MgY
2-
(aq) Kf = 4,9 x 10
8
Ca
2+
(aq) + Y
4-
(aq) CaY
2-
(aq) Kf = 5,0 x 10
10
OBQ-2012 Fase III
>> Olimpada Brasileira de Qumica - 2012
55
Modalidade B
No processo de titulao utilizado o indicador negro de Ericromo T (H3In em sua
forma protonada), que forma complexos de cor vermelho-vinho com os metais em
soluo. Em pH = 10, quando deslocado pelo EDTA, o indicador encontra-se na for-
ma HIn- de cor azul. Logo, o fnal da titulao tido quando a soluo assume uma
colorao azul indicando o excesso do indicador livre. O grfco para a titulao
complexomtrica de clcio e magnsio, em pH = 10, apresentado a seguir.

Para se determinar a concentrao de uma soluo de Ca
2+
, foi preparada uma
soluo de EDTA dissolvendo NaH
2
Y . H
2
O em gua e completando o volume do
balo at 250 mL. Como a concentrao de EDTA era desconhecida, foi usada uma
soluo de Mg
2+
de concentrao 0,0050 M para padronizao. O volume gasto na
padronizao de 25,0 mL da soluo de EDTA foi de 18,5 mL da soluo de Mg
2+
.
Antes de iniciar a titulao da soluo clcio, 50,0 mL dessa soluo foram mistu-
rados com 50,0 mL da soluo de magnsio utilizada na padronizao do EDTA. A
nova soluo foi diluda em balo volumtrica at o volume de 500 mL. Uma al-
quota de 50 mL foi ento tamponada em pH = 10 e titulada com a soluo de EDTA
gastando 9,7 mL para que a soluo fcasse azul.
I) Explique analiticamente o porqu da adio de magnsio soluo de clcio
antes da titulao.
II) Calcule a concentrao de Ca
2+
da soluo inicial. Expresse o resultado em
mol.L
-1
e em ppm.
Programa Nacional Olimpadas de Qumica <<
56
QUESTO 12
Equilbrio cido-base.
O grfco abaixo representa o progresso da titulao do aminocido histidina com
equivalentes de NaOH.
Histidina

I) Apresente todos os equilbrios de ionizao relevantes para a histidina, indican-
do para cada um deles o pK relacionado. Indique, tambm, as zonas de maior
capacidade tamponante para este aminocido.
Para reproduzir o meio intracelular em laboratrios de bioqumica, tampes de fos-
fato so utilizados.
II) Considerando que o pH intracelular seja igual a 7,4 e que a soluo utilizada
para o preparo do tampo tenha [PO
4
3-
] = 0,01 mol.L
-1
, calcule o volume de HCl
6,00 mol.L
-1
que deve ser adicionado a 500 mL dessa soluo, para obteno da
soluo desejada.
Dados: pK
a
(H
3
PO
4
) = 2,15 pK
a
(H
2
PO
4-
) = 7,1 pK
a
(HPO
4
2-
) = 12,4
QUESTO 12
Uma fbrica que produz cal ( CaO ) necessita reduzir o custo de produo para se
manter no mercado com preo competitivo para o produto. A direo da fbrica
solicitou ao departamento tcnico o estudo da viabilidade de reduzir a tempera-
tura do forno de calcinao de Carbonato de Clcio dos atuais 1500K para 800K.
OBQ-2012 Fase III
>> Olimpada Brasileira de Qumica - 2012
57
Modalidade B
I) Considerando apenas o efeito termodinmico, pergunta-se: O departamento tc-
nico pode aceitar a nova temperatura de calcinao?
II) Em caso afrmativo, o departamento tcnico pode fornecer outra temperatura
de operao que proporcione maior economia?
III) Em caso negativo, qual a temperatura mais econmica para se operar o forno
de calcinao?
Dados a 25C
Substncia DS / J . mol
-1
. K
-1
DH
0
/ kJ . mol
-1
CaCO3(s) 92,9 - 1 206,9
CaO(s) 39,8 -635,1
CO2(g) 213,6 -393,5
OBS: desconsidere a variao das propriedades com a temperatura

QUESTO 13
Uma grande diferena entre os elementos do segundo perodo para os demais a
falta de capacidade de formar um grande nmero de ligaes qumicas. So obser-
vados molculas ou ons como o SiF
6
2
, PF
6

e SF
6
, mas nenhum anlogo observa-
do para carbono, nitrognio ou oxignio.
I) Utilizando de conceitos da Teoria da Ligao de Valncia, explique por que os
elementos silcio, fsforo e enxofre podem fazer um maior nmero de ligaes
que o mximo possvel para carbono, nitrognio ou oxignio.
Alm do SF
6
, o enxofre forma uma vasta srie de compostos com o for: S
2
F
2
, SOF
2
,
SF
4
, SOF
4
e o S
2
F
10
.
II Existem dois compostos com frmula qumica S
2
F
2
, um dos exemplos de iso-
meria mais simples da qumica inorgnica. Escreva a estrutura de Lewis para os
dois ismeros.
As molculas SF
4
e SOF
4
possuem igual nmero pares de eltrons ao redor do to-
mo central, para esses pares est prevista uma geometria de bipirmide trigonal.
III) Represente espacialmente o arranjo bipirmide de base trigonal e identifque
as posies axiais (ax) e equatorais (eq) em sua representao. Defna os ngu-
los tericos formados entre as posies equatoriais e entre uma axial e uma
equatorial.
Programa Nacional Olimpadas de Qumica <<
58
IV) O SOF
4
tem, obviamente, uma ligao diferente das demais. Represente essa
molcula considerando o seu arranjo espacial e explique a sua escolha para a
posio dessa ligao.
V) Entre as molculas SF
4
e XeF
4
, qual apresentar o menor ngulo entre as liga-
es? Justifque sua resposta.
QUESTO 14
Na segunda metade do sculo XIX, Vant Hof e Le Bel estabeleceram o incio da es-
tereoqumica analisando o nmero de ismeros de algumas substncias orgnicas.
As observaes experimentais s poderiam ser explicadas se o carbono ocupasse
o centro de um tetraedro, com as ligaes apontando para os vrtices desse polie-
dro. Assim, descartou-se a possibilidade de uma geometria plana, com o carbono
no centro de um quadrado. Um exemplo do conhecimento do nmero de isme-
ros pode ser visto a seguir:
CH
2
RR = no h ismero CHRRR = dois ismeros
I) Mostre que um arranjo espacial com o carbono em uma estrutura plana o
centro de um quadrado, no poderia sustentar o nmero de ismeros observa-
do para os compostos do tipo CH
2
RR e CHRRR.
II) Represente espacialmente os dois ismeros de um composto tipo CHRRR.
QUESTO 15
Determinado composto orgnico A tem massa molecular de 70 g/mol e apresen-
ta a seguinte composio centesimal: C 85,71 % H14,28%
Este composto, ao sofrer ozonlise, seguida de hidrlise, origina acetona e um
novo composto orgnico B.
O composto B , ento, separado e tratado com soluo de KMnO
4
, em meio ci-
do sulfrico, resultando no composto orgnico C, ismero de funo, de outro
composto orgnico D.
O composto D, saponifcado com soluo aquosa de KOH, originando os com-
postos E e F.
Faa todas as reaes envolvidas no processo descrito acima e nomeie pelas nor-
mas IUPAC os compostos A, B, C, D e F.
OBQ-2012 Fase III
>> Olimpada Brasileira de Qumica - 2012
59
Questes escolhidas

Respostas escolhidas OBQ
Modalidades A e B
QUESTO 11
Desenvolvida por Juliana Barbosa Echenique, IFRJ, Rio de Janeiro, RJ.
a - Atravs do grfco da titulao complexomtrica do Ca
2+
e do Mg
2+
apresenta-
do, observa-se que a reao entre Ca
2+
e o indicador ocorre muito no comeo
da curva de titulao desse metal, e no na parte central da infexo. Tendo
o complexo CaY
2-
um Kf maior que o MgY
2-
, o Mg
2+
no interfere diretamente
na interao do EDTA com o Ca
2+
, no entanto, a adio de Mg
2+
garante que a
mudana da colorao (aparecimento da cor azul) s ocorra aps o trmino
da titulao de todo metal presente. Esse procedimento foi feito para que a
reao fosse mais lenta, isto , para que a reao demorasse mais tempo para
fcar azul. Conhecendo a concentrao do EDTA e do Mg
2+
possvel, portanto,
descontar o EDTA gasto com o Mg
2+
e calcular o volume gasto na titulao do
Ca
2+
. Sendo assim, possvel encontrar a concentrao do Ca
2+
.
b - [EDTA] = 3,7. 10
-3
mol/L

(18,5. 5,0. 10
-3
/ 25,0)
V
soluo
= 50,0 mL
V
EDTA
= 9,7 mL
m
soluo
= 9,7. 3,7. 10
-3
/ 50,0

m
soluo
= 7,178
-4
mol/L.
Logicamente, temos:
7,178. 10
-4
mol ---------- 1 L
3,589. 10
-4
mol ---------- 500 mL
Como temos uma soluo de volume igual a 500 mL, analisando:
50 mL

Mg
2+
(0,005 mol/L); logo: n = 0,005.0,05

n = 2,5. 10
-4
mols de Mg
2+
.
50 mL

Ca
2+
400 mL

gua
Programa Nacional Olimpadas de Qumica <<
60
Como temos 3,589. 10
-4
mols de metal, sendo que destes, 2,5. 10
-4
mols so de Mg
2+
.
Logo:
3,589. 10
-4
mols de metal - 2,5. 10
-4
mols de Mg
2+
= 1,089. 10
-4
mols de Ca
2+
. Este
valor para um volume de 500 mL. Ento na inicial teremos:
m
Ca
2+
= 1,089. 10
-4
/ 0,05

mCa
2+
= 2,178. 10
-3
mol/L
Para transformar em ppm s multiplicar a molaridade encontrada por massa at-
mica e depois multiplica por 1000, veja:
ppm = 2,178. 10
-3
.40,1. 1000

concentrao de Ca
2+
em ppm = 87,34ppm.
QUESTO 12
Desenvolvida por Gabriel Mathews Viana Pinheiro, Colgio Master -Fortaleza,CE
Para ilustrar as espcies protnicas ser feito em grfco de topo:
400 mL gua
Como temos 3,589. 10
-4
mols de metal, sendo que destes, 2,5. 10
-4
mols so de
Mg
2+
. Logo:
3,589. 10
-4
mols de metal - 2,5. 10
-4
mols de Mg
2+
= 1,089. 10
-4
mols de Ca
2+
.
Este valor para um volume de 500 mL. Ento na inicial teremos:
m
Ca
2+
= 1,089. 10
-4
/ 0,05 m
Ca
2+
= 2,178. 10
-3
mol/L
Para transformar em ppm s multiplicar a molaridade encontrada por massa
atmica e depois multiplica por 1000, veja:
ppm = 2,178. 10
-3
.40,1. 1000 concentrao de Ca
2+
em ppm = 87,34ppm.

QUESTO 12
Desenvolvida por Gabriel Mathews Viana Pinheiro, Colgio Master -
Fortaleza,CE

I) Para ilustrar as espcies protnicas ser feito em grfico de topo:
0 PK
1
PK
r
PK
2


0,82 1,82 2,82 5,0 6,0 7,0 8,17 9,17 10,17

Vamos representar a estrutura ao lado como sendo H
3
A
2+
.
Equilbrio de ionizao relevantes para a histidina.


Vamos representar a estrutura ao lado como sendo H
3
A
2+
.
Equilbrio de ionizao relevantes para a histidina.
1 H
3
A
2+

(aq)
+ 1H
2
O
(l)
1 H
2
A
+

(aq)
+ 1H
3
O
+
(aq)
pK
1
= 1,82
1 H
2
A
+

(aq)
+ 1H
2
O
(l)
1 HA
(aq)
+ 1H
3
O
+
(aq)
pK
1
= 6,0
1 HA
-
(aq)
+ 1H2O
(l)
1 A
-

(aq)
+ 1H
3
O
+

(aq)
pK
1
= 9,17
As zonas de maior capacidade tamponante esto na faixa de pH = Pk
+
-1.
Logo, as zonas de maior capacidade tamponante so:
0,82 pH 2,82 ( tampo entre H
3
A
2+

(aq)
e H
2
A
+

(aq)
)
5,0 pH 7,0 ( tampo entre H
2
A
+

(aq)
e HA
(aq)
)
8,17 pH 10,17 ( tampo entre HA
(aq)
e A
-
(aq)
)
II ) A zona de maior capacidade tamponante de um tampo de H
2
PO
4
-
e HPO
4
2-
e
pH= pKa
2
+
- 1, no caso: 6,1 pH 8,1.
Equao de Henderson-Hasselbalch: pH = pKa = log [Base]/[cido]
Clculo da razo [HPO
4
2-
]/[ H
2
PO
4
-
] necessria para um tampo de H
2
PO
4
-
de
pH=7,4.
pH = pKa
2

+ log [HPO
4
2-
]/[ H
2
PO
4
-
]
7,4 = 7,1


+ log [HPO
4
2-
]/[ H
2
PO
4
-
]
OBQ-2012 Fase III
>> Olimpada Brasileira de Qumica - 2012
61
As zonas de maior capacidade tamponante esto na faixa de pH = Pk
+
-1. Logo, as
zonas de maior capacidade tamponante so:
0,82 pH 2,82 ( tampo entre H
3
A
2+

(aq)
e H
2
A
+

(aq)
)
5,0 pH 7,0 ( tampo entre H
2
A
+

(aq)
e HA
(aq)
)
8,17 pH 10,17 ( tampo entre HA
(aq)
e A
-
(aq)
)
II ) A zona de maior capacidade tamponante de um tampo de H
2
PO
4-
e HPO
4
2-
e
pH= pKa
2

+-
1, no caso: 6,1 pH 8,1.
Equao de Henderson-Hasselbalch: pH = pKa = log [Base]/[cido]
Clculo da razo [HPO
4
2-
]/[ H
2
PO
4-
] necessria para um tampo de H
2
PO
4-

de pH=7,4.
pH = pKa
2
+ log [HPO
4
2-
]/[ H
2
PO
4-
]
7,4 = 7,1 + log [HPO
4
2-
]/[ H
2
PO
4-
]
[HPO
4
2-
]/[ H
2
PO
4-
]= 1,995
Clculo do nmero de mols de PO
4
-3
da soluo:
NPO
4
-3
= M PO
4
-3
x V PO
4
-3
= 0,01M x 0,500 L = 5x10
-3
mol de PO
4
-3
.
No tampo com pH = 7,4 as espcies presentes que contm fsforo sero
HPO
4
-2
e H
2
PO
4-
. Equilbrio de massa : 5 x 10
-3
mol = n HPO
4
-2
= nHPO
4
-2
(I)
Clculo do nHPO
4
-2
e do nHPO
4-
no tampo:
[HPO
4
2-
]/[ H
2
PO
4
-
]= 1,995
Clculo do nmero de mols de PO
4
-3
da soluo:
NPO
4
-3
= M PO
4
-3
x V PO
4
-3
= 0,01M x 0,500 L = 5x10
-3
mol de PO
4
-3
.
No tampo com pH = 7,4 as espcies presentes que contm fsforo sero
HPO
4
-2
e H
2
PO
4
-
. Equilbrio de massa : 5 x 10
-3
mol = n HPO
4
-2
= nHPO
4
-2
(I)
Clculo do nHPO
4
-2
e do

nHPO
4
-
no tampo:
[HPO
4
-2
]/ [HPO
4
-
] =

nHPO
4
-2
/ V = 1,995 (II)
nHPO
4
-
/ V

Substituindo (II) em (I), temos:
1,995 x nH
2
PO
4
-
= nHPO
4
-
= 5 x 10
-3
nHPO
4
-
= 1,669 x 10
-3
mol
nHPO
4
-2 =
3,33 x 10
-3
mol
Quando o HCl adicionado :
1 HCl
(aq)
+ 1PO
4
-3
(aq)
Cl
-

(aq)
+ 1HPO
4
2-
(aq)
(reao completa)
Clculo do volume de HCl necessrio para converter todo o PO
4
3-
e HPO
4
2-

nPO
4
-3
= 5 x 10
-3
mol 1mol de HCl 1mol de PO
4
-3

5x 10
-3
mol de HCl 5x 10
-3
mol de PO
4
-3
VHCl = nHCl/ MHCl = 5 x 10
-3
mol / 6,00mol = 8,33 x 10
-4
L
Depois disso, quando HCl adicionado , ocorre a seguinte reao:
1 HCl
(aq)
+ 1PO
4
-2
(aq)
Cl
-

(aq)
+ 1H
2
PO
4
-
(aq)
(reao completa)
1,669 x 10-3mol de HPO42-devm ser consumidos para que possa ternH2PO4-
=1,669 x 10-3 e nHPO42- = 3,33 x 10-3mol. Logo:
1mol de HCl 1mol de HPO
4
-2
1,669 x 10
-3
mol de HCl 1mol de HPO
4
-2
VHCl = nHCl/ MHCl = 1,669 x 10
-3
mol / 6,00mol = 2,78 x 10
-4
L
Substituindo (II) em (I), temos:
[HPO
4
2-
]/[ H
2
PO
4
-
]= 1,995
Clculo do nmero de mols de PO
4
-3
da soluo:
NPO
4
-3
= M PO
4
-3
x V PO
4
-3
= 0,01M x 0,500 L = 5x10
-3
mol de PO
4
-3
.
No tampo com pH = 7,4 as espcies presentes que contm fsforo sero
HPO
4
-2
e H
2
PO
4
-
. Equilbrio de massa : 5 x 10
-3
mol = n HPO
4
-2
= nHPO
4
-2
(I)
Clculo do nHPO
4
-2
e do

nHPO
4
-
no tampo:
[HPO
4
-2
]/ [HPO
4
-
] =

nHPO
4
-2
/ V = 1,995 (II)
nHPO
4
-
/ V

Substituindo (II) em (I), temos:
1,995 x nH
2
PO
4
-
= nHPO
4
-
= 5 x 10
-3
nHPO
4
-
= 1,669 x 10
-3
mol
nHPO
4
-2 =
3,33 x 10
-3
mol
Quando o HCl adicionado :
1 HCl
(aq)
+ 1PO
4
-3
(aq)
Cl
-

(aq)
+ 1HPO
4
2-
(aq)
(reao completa)
Clculo do volume de HCl necessrio para converter todo o PO
4
3-
e HPO
4
2-

nPO
4
-3
= 5 x 10
-3
mol 1mol de HCl 1mol de PO
4
-3

5x 10
-3
mol de HCl 5x 10
-3
mol de PO
4
-3
VHCl = nHCl/ MHCl = 5 x 10
-3
mol / 6,00mol = 8,33 x 10
-4
L
Depois disso, quando HCl adicionado , ocorre a seguinte reao:
1 HCl
(aq)
+ 1PO
4
-2
(aq)
Cl
-

(aq)
+ 1H
2
PO
4
-
(aq)
(reao completa)
1,669 x 10-3mol de HPO42-devm ser consumidos para que possa ternH2PO4-
=1,669 x 10-3 e nHPO42- = 3,33 x 10-3mol. Logo:
1mol de HCl 1mol de HPO
4
-2
1,669 x 10
-3
mol de HCl 1mol de HPO
4
-2
VHCl = nHCl/ MHCl = 1,669 x 10
-3
mol / 6,00mol = 2,78 x 10
-4
L
Quando o HCl adicionado :
[HPO
4
2-
]/[ H
2
PO
4
-
]= 1,995
Clculo do nmero de mols de PO
4
-3
da soluo:
NPO
4
-3
= M PO
4
-3
x V PO
4
-3
= 0,01M x 0,500 L = 5x10
-3
mol de PO
4
-3
.
No tampo com pH = 7,4 as espcies presentes que contm fsforo sero
HPO
4
-2
e H
2
PO
4
-
. Equilbrio de massa : 5 x 10
-3
mol = n HPO
4
-2
= nHPO
4
-2
(I)
Clculo do nHPO
4
-2
e do

nHPO
4
-
no tampo:
[HPO
4
-2
]/ [HPO
4
-
] =

nHPO
4
-2
/ V = 1,995 (II)
nHPO
4
-
/ V

Substituindo (II) em (I), temos:
1,995 x nH
2
PO
4
-
= nHPO
4
-
= 5 x 10
-3
nHPO
4
-
= 1,669 x 10
-3
mol
nHPO
4
-2 =
3,33 x 10
-3
mol
Quando o HCl adicionado :
1 HCl
(aq)
+ 1PO
4
-3
(aq)
Cl
-

(aq)
+ 1HPO
4
2-
(aq)
(reao completa)
Clculo do volume de HCl necessrio para converter todo o PO
4
3-
e HPO
4
2-

nPO
4
-3
= 5 x 10
-3
mol 1mol de HCl 1mol de PO
4
-3

5x 10
-3
mol de HCl 5x 10
-3
mol de PO
4
-3
VHCl = nHCl/ MHCl = 5 x 10
-3
mol / 6,00mol = 8,33 x 10
-4
L
Depois disso, quando HCl adicionado , ocorre a seguinte reao:
1 HCl
(aq)
+ 1PO
4
-2
(aq)
Cl
-

(aq)
+ 1H
2
PO
4
-
(aq)
(reao completa)
1,669 x 10-3mol de HPO42-devm ser consumidos para que possa ternH2PO4-
=1,669 x 10-3 e nHPO42- = 3,33 x 10-3mol. Logo:
1mol de HCl 1mol de HPO
4
-2
1,669 x 10
-3
mol de HCl 1mol de HPO
4
-2
VHCl = nHCl/ MHCl = 1,669 x 10
-3
mol / 6,00mol = 2,78 x 10
-4
L
Questes escolhidas
Programa Nacional Olimpadas de Qumica <<
62
Clculo do volume de HCl necessrio para converter todo o PO
4
3-
e HPO
4
2-
[HPO
4
2-
]/[ H
2
PO
4
-
]= 1,995
Clculo do nmero de mols de PO
4
-3
da soluo:
NPO
4
-3
= M PO
4
-3
x V PO
4
-3
= 0,01M x 0,500 L = 5x10
-3
mol de PO
4
-3
.
No tampo com pH = 7,4 as espcies presentes que contm fsforo sero
HPO
4
-2
e H
2
PO
4
-
. Equilbrio de massa : 5 x 10
-3
mol = n HPO
4
-2
= nHPO
4
-2
(I)
Clculo do nHPO
4
-2
e do

nHPO
4
-
no tampo:
[HPO
4
-2
]/ [HPO
4
-
] =

nHPO
4
-2
/ V = 1,995 (II)
nHPO
4
-
/ V

Substituindo (II) em (I), temos:
1,995 x nH
2
PO
4
-
= nHPO
4
-
= 5 x 10
-3
nHPO
4
-
= 1,669 x 10
-3
mol
nHPO
4
-2 =
3,33 x 10
-3
mol
Quando o HCl adicionado :
1 HCl
(aq)
+ 1PO
4
-3
(aq)
Cl
-

(aq)
+ 1HPO
4
2-
(aq)
(reao completa)
Clculo do volume de HCl necessrio para converter todo o PO
4
3-
e HPO
4
2-

nPO
4
-3
= 5 x 10
-3
mol 1mol de HCl 1mol de PO
4
-3

5x 10
-3
mol de HCl 5x 10
-3
mol de PO
4
-3
VHCl = nHCl/ MHCl = 5 x 10
-3
mol / 6,00mol = 8,33 x 10
-4
L
Depois disso, quando HCl adicionado , ocorre a seguinte reao:
1 HCl
(aq)
+ 1PO
4
-2
(aq)
Cl
-

(aq)
+ 1H
2
PO
4
-
(aq)
(reao completa)
1,669 x 10-3mol de HPO42-devm ser consumidos para que possa ternH2PO4-
=1,669 x 10-3 e nHPO42- = 3,33 x 10-3mol. Logo:
1mol de HCl 1mol de HPO
4
-2
1,669 x 10
-3
mol de HCl 1mol de HPO
4
-2
VHCl = nHCl/ MHCl = 1,669 x 10
-3
mol / 6,00mol = 2,78 x 10
-4
L
Depois disso, quando HCl adicionado , ocorre a seguinte reao:
[HPO
4
2-
]/[ H
2
PO
4
-
]= 1,995
Clculo do nmero de mols de PO
4
-3
da soluo:
NPO
4
-3
= M PO
4
-3
x V PO
4
-3
= 0,01M x 0,500 L = 5x10
-3
mol de PO
4
-3
.
No tampo com pH = 7,4 as espcies presentes que contm fsforo sero
HPO
4
-2
e H
2
PO
4
-
. Equilbrio de massa : 5 x 10
-3
mol = n HPO
4
-2
= nHPO
4
-2
(I)
Clculo do nHPO
4
-2
e do

nHPO
4
-
no tampo:
[HPO
4
-2
]/ [HPO
4
-
] =

nHPO
4
-2
/ V = 1,995 (II)
nHPO
4
-
/ V

Substituindo (II) em (I), temos:
1,995 x nH
2
PO
4
-
= nHPO
4
-
= 5 x 10
-3
nHPO
4
-
= 1,669 x 10
-3
mol
nHPO
4
-2 =
3,33 x 10
-3
mol
Quando o HCl adicionado :
1 HCl
(aq)
+ 1PO
4
-3
(aq)
Cl
-

(aq)
+ 1HPO
4
2-
(aq)
(reao completa)
Clculo do volume de HCl necessrio para converter todo o PO
4
3-
e HPO
4
2-

nPO
4
-3
= 5 x 10
-3
mol 1mol de HCl 1mol de PO
4
-3

5x 10
-3
mol de HCl 5x 10
-3
mol de PO
4
-3
VHCl = nHCl/ MHCl = 5 x 10
-3
mol / 6,00mol = 8,33 x 10
-4
L
Depois disso, quando HCl adicionado , ocorre a seguinte reao:
1 HCl
(aq)
+ 1PO
4
-2
(aq)
Cl
-

(aq)
+ 1H
2
PO
4
-
(aq)
(reao completa)
1,669 x 10-3mol de HPO42-devm ser consumidos para que possa ternH2PO4-
=1,669 x 10-3 e nHPO42- = 3,33 x 10-3mol. Logo:
1mol de HCl 1mol de HPO
4
-2
1,669 x 10
-3
mol de HCl 1mol de HPO
4
-2
VHCl = nHCl/ MHCl = 1,669 x 10
-3
mol / 6,00mol = 2,78 x 10
-4
L
1,669 x 10
-3
mol de HPO
4
2-
devem ser consumidos para que possa ter nH
2
PO
4
-

=1,669 x 10
-3
e nHPO
4
2-
= 3,33 x 10
-3
mol. Logo:
1mol de HCl 1mol de HPO
4
-2
1,669 x 10
-3
mol de HCl 1mol de HPO
4
-2
VHCl = nHCl/ MHCl = 1,669 x 10
-3
mol / 6,00mol = 2,78 x 10
-4
L
Volume total de HCl 6,0M necessrio = (8,33 + 2,78) x 10
-4
L = 1,111 x 10
-3
L
QUESTO 13
Desenvolvida por Joo Lucas Maehara Said dos Reis, IFES, Maratazes- ES.
Para produzir cal (CaO), a fbrica faz a seguinte reao de decomposio (calcinao):
Volume total de HCl 6,0M necessrio = (8,33 + 2,78) x 10
-4
L = 1,111 x 10
-3
L

QUESTO 13
Desenvolvida por Joo Lucas Maehara Said dos Reis, IFES, Maratazes- ES.

Para produzir cal (CaO), a fbrica faz a seguinte reao de decomposio
(calcinao):
CaCO
3
(s) CaO(s) + CO
2
(g)
I) necessrio analisar os valores de G para as temperaturas de 1500K e 800K
A partir dos dados disponveis na tabela, calcula-se os valores de H
0
e S para
reao de calcinao:
H
0
= H
0
produtos - H
0
reagentes
[1 mol (- 635, 1 KJ mol
-1
) + 1 mol (- 393,5 KJ mL
-1
)] [1 mol (1206,9 KJ mol
-1
)]
= 1,783,10 KJ
S
0
= H
0
produtos - H
0
reagentes
[(213,6 J mol
-1
. K
-1
) 1 mol + 1 mol (39,8 j mol
-1
.

K
-1
)]- [1 mol (92,95 mol
-1
. K
-1
)]
= 1,605.10
-1
KJ K
-1

A partir dos valores obtidos calcula-se valor de G a parti da expresso G= H
0
-
S
0
T
Para a temperatura de 800k: G = 1,783.10
-1
KJ 1,605.10
-1
KJ. K
-1
. 1500 K
= -62,45 KJ
Para a temperatura de 800k: G = 1,783 . 10
2
KJ 1,605.10
-1
KJ. K
-1
. 800 K
= 49,9 KJ
A 800k tem que G> 0. A esta temperatura a reao no espontnea, o que
desfavorvel para a indstria. Logo, o departamento tcnico na de v aceitar
mudana da temperatura de calcinao de 1500K para 800K.

I) necessrio analisar os valores de G para as temperaturas de 1500K e 800K
A partir dos dados disponveis na tabela, calcula-se os valores de H0 e S para
reao de calcinao:
H
0
= H
0
produtos - H
0
reagentes
[1 mol (- 635, 1 KJ mol
-1
) + 1 mol (- 393,5 KJ mL
-1
)] [1 mol (1206,9 KJ mol-1)]
= 1,783,10 KJ
S
0
= H
0
produtos - H
0
reagentes
OBQ-2012 Fase III
>> Olimpada Brasileira de Qumica - 2012
63
[(213,6 J mol
-1
. K
-1
) 1 mol + 1 mol (39,8 j mol-1 . K
-1
)]- [1 mol (92,95 mol
-1
. K
-1
)]
= 1,605.10
-1
KJ K
-1
A partir dos valores obtidos calcula-se valor de G a parti da expresso G= H
0
- S
0
T
Para a temperatura de 800k: G = 1,783.10
-1
KJ 1,605.10
-1
KJ. K
-1
. 1500 K
= -62,45 KJ
Para a temperatura de 800k: G = 1,783 . 10
2
KJ 1,605.10
-1
KJ. K
-1
. 800 K
= 49,9 KJ
A 800k tem que G> 0. A esta temperatura a reao no espontnea, o que des-
favorvel para a indstria. Logo, o departamento tcnico na de v aceitar mudana
da temperatura de calcinao de 1500K para 800K.
II) De acordo com item anterior o caso negativo. A resposta do item anterior diz
que no conveniente aceitar a diminuio da temperatura, logo, no conve-
niente tentar novas medidas de economia baixando a temperatura.
III) A temperatura mais econmica aquela em que T assuma o menor valor pos-
svel para que G<0
A partir da expresso G= H
0
- S
0
T
II) De acordo com item anterior o caso negativo. A resposta do item anterior diz
que no conveniente aceitar a diminuio da temperatura, logo, no conveniente
tentar novas medidas de economia baixando a temperatura.
III) A temperatura mais econmica aquela em que T assuma o menor valor
possvel para que G<0
A partir da expresso G= H
0
- S
0
T
H
0
- S.T<0
H
0
<S.T
T>


T>





T => 1110,90 K
Logo, para qualquer valor de T maior que 111,90 K tem-se G <0 pode-se, portanto,
operar o forno de calcinao temperatura de 1111K. Portanto, a temperatura mais
econmica deve ser superior a 1110,9K lembrando que no vivel uma
temperatura igual a 1110,9 K , pois o equilbrio seria estabelecido.
Em suma:
I) No. O departamento tcnico no pode aceitar a nova temperatura
(800K). Como a reao espontnea apenas para T>1110,9K, a 800K
reao no ocorreria.
II) O caso foi negativo.
III) A temperatura mnima para se operar o forno de calcinao a
temperatura mnima para qual a reao espontnea, ou seja,
aproximadamente 1110,9K.


QUESTO 14
Logo, para qualquer valor de T maior que 111,90 K tem-se G <0 pode-se, portan-
to, operar o forno de calcinao temperatura de 1111K. Portanto, a temperatura
mais econmica deve ser superior a 1110,9K lembrando que no vivel uma tem-
peratura igual a 1110,9 K , pois o equilbrio seria estabelecido.
Questes escolhidas
Programa Nacional Olimpadas de Qumica <<
64
Em suma:
I) No. O departamento tcnico no pode aceitar a nova temperatura (800K). Como
a reao espontnea apenas para T>1110,9K, a 800K reao no ocorreria.
II) O caso foi negativo.
III) A temperatura mnima para se operar o forno de calcinao a temperatura
mnima para qual a reao espontnea, ou seja, aproximadamente 1110,9K.
QUESTO 14
Questo ANULADA. Contudo, teve a seguinte soluo dada pelo estudante Joo
Lucas Maehara Said dos Reis, IFES, Maratazes, ES.
I) Os elementos silcio, fsforo e enxofre possuem orbitais d em suas camadas
de valncia, que os permitem formar um nmero de orbitais hbridos maior, e
consequentemente um maior nmero de ligaes. E o carbono, nitrognio e o
oxignio possuem apenas orbitais s e p em sua camada de valncia, que a L. E
isso impede que ocorra uma expanso do octeto nesses elementos, limitando
assim o nmero de ligaes possveis.
II) As estruturas de Lewis so:

Questo ANULADA. Contudo, teve a seguinte soluo dada pelo estudante Joo
Lucas Maehara Said dos Reis, IFES, Maratazes, ES.


I) Os elementos silcio, fsforo e enxofre possuem orbitais d em suas camadas de
valncia, que os permitem formar um nmero de orbitais hbridos maior, e
consequentemente um maior nmero de ligaes. E o carbono, nitrognio e o
oxignio possuem apenas orbitais s e p em sua camada de valncia, que a L. E
isso impede que ocorra uma expanso do octeto nesses elementos, limitando
assim o nmero de ligaes possveis.

II) As estruturas de Lewis so:
S F S F
S S
F
F


III) Arranjo bipirmide de base trigonal:

ax
eq
ax
eq
e
O
O
O
O
O
O

ngulo entre equatoriais: 120 / ngulo entre equatorial e axial: 90

IV) A ligao com o O foi colocada na posio equatorial para oferecer mais
estabilidade molcula, devido conformao trigonal plana entre os tomos do
eixo equatorial.
III) Arranjo bipirmide de base trigonal:
Questo ANULADA. Contudo, teve a seguinte soluo dada pelo estudante Joo
Lucas Maehara Said dos Reis, IFES, Maratazes, ES.


I) Os elementos silcio, fsforo e enxofre possuem orbitais d em suas camadas de
valncia, que os permitem formar um nmero de orbitais hbridos maior, e
consequentemente um maior nmero de ligaes. E o carbono, nitrognio e o
oxignio possuem apenas orbitais s e p em sua camada de valncia, que a L. E
isso impede que ocorra uma expanso do octeto nesses elementos, limitando
assim o nmero de ligaes possveis.

II) As estruturas de Lewis so:
S F S F
S S
F
F


III) Arranjo bipirmide de base trigonal:

ax
eq
ax
eq
e
O
O
O
O
O
O

ngulo entre equatoriais: 120 / ngulo entre equatorial e axial: 90

IV) A ligao com o O foi colocada na posio equatorial para oferecer mais
estabilidade molcula, devido conformao trigonal plana entre os tomos do
eixo equatorial.
ngulo entre equatoriais: 120 / ngulo entre equatorial e axial: 90
OBQ-2012 Fase III
>> Olimpada Brasileira de Qumica - 2012
65
IV) A ligao com o O foi colocada na posio equatorial para oferecer mais esta-
bilidade molcula, devido conformao trigonal plana entre os tomos do
eixo equatorial.

S
F
F
F
F
O


V) Como a repulso de pares de eltrons no ligantes maior do que de eltrons
ligantes, o XeF
4
apresentar o menor ngulo entre ligaes, j que os pares
ligantes estaro mais prximos uns dos outros e ele possui 2 pares de eltrons
no ligantes, enquanto o SF
4
possui apenas 1.
S
F
F
Xe
F
F
F
F
F
F

1 par de e
-
no ligantes 2 pares de e
-
no ligantes


QUESTO 15 Modalidade A
Desenvolvida por Rafael Tedeschi Eugnio Pontes Barone, Colgio COC, Araatuba-SP.
I)

A amnia apresenta hibridizao sp
3
, possuindo um par de eltrons no-
ligantes. Logo, sua geometria piramidal.
II) NH
3(g)
+ H
2
O
(l)
NH
4
OH
(aq)

NH
4
OH
(aq)
NH
4(aq)
+
+ OH
(aq)
-

A soluo aquosa de hidrxido de amnio a usada em produtos
desinfetantes como Ajax ou Veja, sendo tambm conhecida como amonaco.
III) NH
3(g)
+ HCl
(g)
NH
4
Cl
(s)

V) Como a repulso de pares de eltrons no ligantes maior do que de eltrons
ligantes, o XeF
4
apresentar o menor ngulo entre ligaes, j que os pares li-
gantes estaro mais prximos uns dos outros e ele possui 2 pares de eltrons
no ligantes, enquanto o SF
4
possui apenas 1.

S
F
F
F
F
O


V) Como a repulso de pares de eltrons no ligantes maior do que de eltrons
ligantes, o XeF
4
apresentar o menor ngulo entre ligaes, j que os pares
ligantes estaro mais prximos uns dos outros e ele possui 2 pares de eltrons
no ligantes, enquanto o SF
4
possui apenas 1.
S
F
F
Xe
F
F
F
F
F
F

1 par de e
-
no ligantes 2 pares de e
-
no ligantes


QUESTO 15 Modalidade A
Desenvolvida por Rafael Tedeschi Eugnio Pontes Barone, Colgio COC, Araatuba-SP.
I)

A amnia apresenta hibridizao sp
3
, possuindo um par de eltrons no-
ligantes. Logo, sua geometria piramidal.
II) NH
3(g)
+ H
2
O
(l)
NH
4
OH
(aq)

NH
4
OH
(aq)
NH
4(aq)
+
+ OH
(aq)
-

A soluo aquosa de hidrxido de amnio a usada em produtos
desinfetantes como Ajax ou Veja, sendo tambm conhecida como amonaco.
III) NH
3(g)
+ HCl
(g)
NH
4
Cl
(s)

par de e- no ligantes 2 pares de e- no ligantes
QUESTO 15 - Modalidade A
Desenvolvida por Rafael Tedeschi Eugnio Pontes Barone, Colgio COC, Araatuba-SP.
I)
S
F
F
F
F
O


V) Como a repulso de pares de eltrons no ligantes maior do que de eltrons
ligantes, o XeF
4
apresentar o menor ngulo entre ligaes, j que os pares
ligantes estaro mais prximos uns dos outros e ele possui 2 pares de eltrons
no ligantes, enquanto o SF
4
possui apenas 1.
S
F
F
Xe
F
F
F
F
F
F

1 par de e
-
no ligantes 2 pares de e
-
no ligantes


QUESTO 15 Modalidade A
Desenvolvida por Rafael Tedeschi Eugnio Pontes Barone, Colgio COC, Araatuba-SP.
I)

A amnia apresenta hibridizao sp
3
, possuindo um par de eltrons no-
ligantes. Logo, sua geometria piramidal.
II) NH
3(g)
+ H
2
O
(l)
NH
4
OH
(aq)

NH
4
OH
(aq)
NH
4(aq)
+
+ OH
(aq)
-

A soluo aquosa de hidrxido de amnio a usada em produtos
desinfetantes como Ajax ou Veja, sendo tambm conhecida como amonaco.
III) NH
3(g)
+ HCl
(g)
NH
4
Cl
(s)


A amnia apresenta hibridizao sp
3
, possuindo um par de eltrons no-ligantes.
Logo, sua geometria piramidal.
II) NH
3(g)
+ H
2
O
(l)
NH
4
OH
(aq)
NH
4
OH
(aq)
NH
4(aq)
+
+ OH
(aq)-
Questes escolhidas
Programa Nacional Olimpadas de Qumica <<
66
A soluo aquosa de hidrxido de amnio a usada em produtos desinfetantes
como Ajax ou Veja, sendo tambm conhecida como amonaco.
III) NH
3(g)
+ HCl
(g)
NH
4
Cl
(s)
A soluo aquosa de NH
4
Cl, na presena de fenolftalena, ser incolor uma vez que
esse indicador cido-base no manifesta cor em meio cido.
Com efeito, NH
4
Cl
(aq)
NH
4
+
(aq)
+ H
+
(aq)
O Cl
-(aq)
no sofre hidrlise (uma vez que o HCl um cido forte).
Contudo, o NH
4
+
(aq)
sofre hidrlise, conforme a reao:
O aumento da concentrao de H
+
devida a hidrlise salina do NH
4
Cl
(s)
deixar a
soluo cida, de sorte a deix-la incolor na presena de fenolftalena.
IV) N
2(g)
+ 3 H
2(g)
2NH
3(g)
: Processo Haber- Bosch
(em cima da seta da reao tem escrito: alta temperatura e em baixo da seta tem:
alta presso)
NH
3(g)
+ HNO
3
NH
4
NO
3
: nitrao da amnia
QUESTO 15 - Modalidade B
Desenvolvida por Gabriel Mathews Viana Pinheiro, Colgio Master Fortaleza,CE.
Item I:
Supondo um arranjo com o carbono em uma estrutura plaa, no centro de um qua-
drado, teramos:
Para o CH
2
RR: Dois tipos de ismeros. A constatao experimental,
contudo, que CHRR e um composto sem ismeros.

ISMERO TRANS ISMERO CIS
A soluo aquosa de NH
4
Cl, na presena de fenolftalena, ser incolor uma vez que
esse indicador cido-base no manifesta cor em meio cido.
Com efeito, NH
4
Cl
(aq)
NH
4
+

(aq)
+ H
+
(aq)

O Cl
-
(aq)
no sofre hidrlise (uma vez que o HCl um cido forte).
Contudo, o NH
4
+
(aq)
sofre hidrlise, conforme a reao:
O aumento da concentrao de H
+
devida a hidrlise salina do NH
4
Cl
(s)
deixar a
soluo cida, de sorte a deix-la incolor na presena de fenolftalena.
IV) N
2(g)
+ 3 H
2(g)
2NH
3(g)
: Processo Haber- Bosch
(em cima da seta da reao tem escrito: alta temperatura e em baixo da seta tem:
alta presso)
NH
3(g)
+ HNO
3
NH
4
NO
3
: nitrao da amnia



QUESTO 15 Modalidade B
Desenvolvida por Gabriel Mathews Viana Pinheiro, Colgio Master Fortaleza,CE.
Item I:
Supondo um arranjo com o carbono em uma estrutura plaa, no centro de um
quadrado, teramos:
Para o CH
2
RR: Dois tipos de ismeros. A constatao experimental,
contudo, que CHRR e um composto sem ismeros.

ISMERO TRANS ISMERO CIS





H R


H R

R H

H R

C
C

OBQ-2012 Fase III
>> Olimpada Brasileira de Qumica - 2012
67
Para o CHRRR
Para o CHRRR





Trs tipos de ismeros. A constatao experimental, contudo, que CHRRR um
composto que possui somente 2 ismeros.
Item II: Os 2 ismeros de CHRRR so enantimeros (guardam um relao de
objeto-imagem, mas no so superponveis).








QUESTO 16 Modalidade A
Desenvolvida por Luis Fernando Machado Poletti Valle, Mater Amabilis, So Pailo, SP.
I) Processos:
- Agregao;
- Floculao;
- Decantao;
H R
C
R
R
C

H

R

R

R
H R
C
R
R
H R
C
R
R
H

R

R

R
C

Trs tipos de ismeros. A constatao experimental, contudo, que CHRRR
um composto que possui somente 2 ismeros.
Item II: Os 2 ismeros de CHRRR so enantimeros (guardam um relao de
objeto-imagem, mas no so superponveis).
Para o CHRRR





Trs tipos de ismeros. A constatao experimental, contudo, que CHRRR um
composto que possui somente 2 ismeros.
Item II: Os 2 ismeros de CHRRR so enantimeros (guardam um relao de
objeto-imagem, mas no so superponveis).








QUESTO 16 Modalidade A
Desenvolvida por Luis Fernando Machado Poletti Valle, Mater Amabilis, So Pailo, SP.
I) Processos:
- Agregao;
- Floculao;
- Decantao;
H R
C
R
R
C

H

R

R

R
H R
C
R
R
H R
C
R
R
H

R

R

R
C

QUESTO 16 - Modalidade A
Desenvolvida por Luis Fernando Machado Poletti Valle, Mater Amabilis, So Pai-
lo, SP.
I) Processos:
- Agregao;
- Floculao;
- Decantao;
- Filtrao;
- Purifcao;
- Fluorao/fuoretao;
- Neutralizao
II) Frmula do cido:
A frmula do cido em questo H
2
SiF
6
III) Clculo da quantidade de cido:
Note que para cada m
3
de gua, pode-se ter no mximo 1,5 mg x 1000 L = 1,5 F
-
.
Questes escolhidas
Programa Nacional Olimpadas de Qumica <<
68
Com a massa molar, vemos que isso corresponde a cerca de 78,9 mmol de F
-
(0,0789
mol F
-
). Mas pela frmula do cido vemos que 1 mol de cido gera 6 mols de F
-
,
logo o valor de F
-
corresponde aproximadamente 1,9 g de cido.
Pelo enunciado, temos que h 23% de 1,19 g (0,27 g) de cido por mL de soluo,
logo temos que se deve adicionar no mximo 6,93 mL da soluo de cido fuorsi-
lcico para cada m3 de gua para consumo humano.
QUESTO 16 - Modalidade B
Desenvolvida por Tiago Follmann Perin, Fundao Liberato Salzano, Novo Ham-
burgo, RS.
- Filtrao;
- Purificao;
- Fluorao/fluoretao;
- Neutralizao
II) Frmula do cido:
A frmula do cido em questo H
2
SiF
6
III) Clculo da quantidade de cido:
Note que para cada m
3
de gua, pode-se ter no mximo 1,5 mg x 1000 L = 1,5
F
-
. Com a massa molar, vemos que isso corresponde a cerca de 78,9 mmol de F
-

(0,0789 mol F
-
). Mas pela frmula do cido vemos que 1 mol de cido gera 6 mols
de F
-
, logo o valor de F
-
corresponde aproximadamente 1,9 g de cido.
Pelo enunciado, temos que h 23% de 1,19 g (0,27 g) de cido por mL de
soluo, logo temos que se deve adicionar no mximo 6,93 mL da soluo de cido
fluorsilcico para cada m
3
de gua para consumo humano.


QUESTO 16 Modalidade B
Desenvolvida por Tiago Follmann Perin, Fundao Liberato Salzano, Novo Hamburgo, RS.

85,71g C 100g A 14,28g H 100g A
X 70g X 70g
X = 60g C X = 10g
1 mol C 12g 1 mol H 1g
X 60g X 10g
X = 5 mol X= 10 mol
C
5
H
10


CH
3
CH C
CH
3
C H
3 +
O
3
C H
3
C
O
H
+
+
H
2
O
2 +
O H
2
C H
3
C CH
3
O

A B
C H
3
C
O
H
+
[O]
KMnO
4
C H
3
C
O
OH
C
Ismero de funo de acido =ster
+
K OH
+
C H
3
OH C H
O
O CH
3
C H
O
OK

D E F
Compostos nomenclatura IUPAC
A 2-metil-but-2-eno

B
H
3
C C
O
H
etanal

C
H
3
C C
O
OH
cido etanoico

D
H C
O
O CH
3
metanoato de metila
E
H C
O
O K
metanoato de potssio

F
H
3
C OH
metanol
Ismero de funo de acido =ster

CH
3
CH C
CH
3
C H
3 +
O
3
C H
3
C
O
H
+
+
H
2
O
2 +
O H
2
C H
3
C CH
3
O

A B
C H
3
C
O
H
+
[O]
KMnO
4
C H
3
C
O
OH
C
Ismero de funo de acido =ster
+
K OH
+
C H
3
OH C H
O
O CH
3
C H
O
OK

D E F
Compostos nomenclatura IUPAC
A 2-metil-but-2-eno

B
H
3
C C
O
H
etanal

C
H
3
C C
O
OH
cido etanoico

D
H C
O
O CH
3
metanoato de metila
E
H C
O
O K
metanoato de potssio

F
H
3
C OH
metanol
OBQ-2012 Fase III
>> Olimpada Brasileira de Qumica - 2012
69
Compostos nomenclatura IUPAC

CH
3
CH C
CH
3
C H
3 +
O
3
C H
3
C
O
H
+
+
H
2
O
2 +
O H
2
C H
3
C CH
3
O

A B
C H
3
C
O
H
+
[O]
KMnO
4
C H
3
C
O
OH
C
Ismero de funo de acido =ster
+
K OH
+
C H
3
OH C H
O
O CH
3
C H
O
OK

D E F
Compostos nomenclatura IUPAC
A 2-metil-but-2-eno

B
H
3
C C
O
H
etanal

C
H
3
C C
O
OH
cido etanoico

D
H C
O
O CH
3
metanoato de metila
E
H C
O
O K
metanoato de potssio

F
H
3
C OH
metanol
Questes escolhidas
Programa Nacional Olimpadas de Qumica <<
70
MODALIDADE A
O U R O
Nome Cidade UF
Nicholas de Souza Costa Lima Ari de S Fortaleza CE
Nathrcia Castro Mota Master Fortaleza CE
Lvia Rodrigues de Arajo Farias Brito Fortaleza CE
Joaquim Ivo Vasques Dantas Landim Farias Brito Fortaleza CE
Gabriel Sena Galvo Militar de Braslia Braslia DF
Nathrcia Castro Mota Fortaleza CE CE
Vitria Nunes Medeiros Fortaleza CE CE
P R A T A
Nome Escola Cidade UF
Diego Henrique de Oliveira Sales Sagrado Corao Recife PE
Felipe Brando Forte Farias Brito Fortaleza CE
Vittria Nobre Jacinto Farias Brito Fortaleza CE
Drvylla de Sousa Lima Farias Brito Fortaleza CE
Douglas Igor Santos de Oliveira IFRJ Rio de Janeiro RJ
Pedro Henrique Fonseca Duque IFRJ Rio de Janeiro RJ
Joo Guilherme N. Carvalho Col. Dom Bosco Teresina PI
Arthur Duarte de Marins Costa Pedro II Niteri RJ
Nathan de Souza Mateus IFRJ Rio de Janeiro RJ
Paulo Rogrio Moritz Postigo Pedro II UEEN II Rio de Janeiro RJ
lcio Koodiro Yoshida Colgio Etapa So Paulo SP
Joo Lucas Fernandes dos Santos GGE Recife PE
B R O N Z E
Nome Escola Cidade UF
Thiago Silva Viana IFRJ Rio de Janeiro RJ
Lus Guilherme Gomes Aguiar Ponto de Ensino Rio de Janeiro RJ
Gesa Costa Oliveira de M. Santana Farias Brito Fortaleza CE
Bheatriz Pereira Valverde CEPRON Dom Pedro MA
Giancarlo Ferrigno Poli Ide Alves Bandeirantes So Paulo SP
OBQ-2012 Fase III
>> Olimpada Brasileira de Qumica - 2012
71
Idlia Maria Barbosa do Nascimento CEPRON Dom Pedro MA
Thiago Matheus Santos Rios IFBA Salvador BA
Lucas Felipe Albuquerque Lins Ari de S Fortaleza CE
Mauroclio Rocha Pontes Filho Farias Brito Fortaleza CE
Leyna Misachi Suzuki Etapa So Paulo SP
Lia de Oliveira Domingues Farias Brito Fortaleza CE
Carmem Castieira Bandeirantes So Paulo SP
Francisco Markan Nobre de Souza Filho Farias Brito Fortaleza CE
Pablo Jango Lima Marques Militar de Manaus Manaus AM
Mateus Caracas Veras Farias Brito Fortaleza CE
Pedro talo Oliveira Gomes Ari de S Fortaleza CE
Mateus Vasconcelos Albuquerque Santa Ceclia Fortaleza CE
Luis Fernando Machado Poletti Valle Mater Amabilis Gusrulhos SP
Emanuel Pereira de Queiroz Farias Brito Fortaleza CE
Alexandre Amatruda Marum Albert Sabin So Paulo SP
Murilo Piotrowskisantos Santos Bertoni Foz do Iguau PR
Rafael Tedeschi Eugnio Pontes Barone COC Araatuba SP
Matheus Souza Matsukawa Colgio Pedro II Niteri RJ
Lucas Rebelo Vieira da Silva GGE Recife PE

MENO HONROSA
Nome Escola Cidade UF
Kaynan Bezerra de Lima 7 de setembro Fortaleza CE
Alexandre Marchesini dos Santos Lemos IFRJ Niteri RJ
Pedro Henrique Rocha de Freitas Militar de Braslia Braslia DF
Luciano Pinheiro Batista Farias Brito Fortaleza CE
Yohanna Ferreira Forte Ari de S Fortaleza CE
Ianne Pessoa Holanda Santa Maria Recife PE
George Henrique Nunes da M. Jnior Farias Brito Fortaleza CE
Nilo Parente Pessoa Dias Farias Brito Fortaleza CE
Artur Souto Martins Ari de S Fortaleza CE
Resultados
Programa Nacional Olimpadas de Qumica <<
72
Thiago Tarraf Varella COOPEN Dr. Zerbini S. Jos do R. Preto SP
Andr Andrade Longaray Filho Militar de Braslia Porto Alegre RS
Daniel Imbassahy de S B. C. E Silva Colgio Ofcina Salvador BA
Alexsander Felipe Alencar Justa Ari de S Fortaleza CE
Mayara Melo dos Santos Ari de S Fortaleza CE
Isabelle de Sousa Pereira Farias Brito Fortaleza CE
Gabriel Fernandes de Andrade IFRJ Rio de Janeiro RJ
Thiago Henrique Brasileiro Pedrosa IFRJ Rio de Janeiro RJ
Nicolas Chiu Ogassavara So Paulo Etapa SP
Kaue Felipe Morcelles Santo Antonio Joinville SC
Joo Pedro Pazian Montes Soldera So Paulo Etapa SP
Ariane Cristina Boechie Carvalho Militar Braslia DF
Liara Guinsberg Objetivo So Paulo SP
Leonardo Enrico Marchioro Mendes Militar de Braslia Braslia DF
Jos Rodolfo de Farias Neto Contato Macei AL
Marina Moura Costa Spnola Anchieta Salvador BA
Francisco Maxsuwel Nunes de Oliveira Fortaleza Farias Brito CE
Yasmin Rase Lisboa da Veiga IFPA Belm PA
Pedro Henrique Gonalves Carvalho Farias Brito Fortaleza CE
Narjara Smya Rodrigues Pereira CEPRON Dom Pedro MA
Yago de Lima Barrozo Farias Brito Fortaleza CE
Joo Victor de Moraes Silva IFRJ Rio de Janeiro RJ
Caio Vidal Bezerra Farias Brito Fortaleza CE
Victor Sousa Silva Ari de S Fortaleza CE
Joel Ferreira Madureira Farias Brito Fortaleza CE
Jonadab Dos Santos Silva IFAL Macei AL
Thaisa Gabriele da Silva Belan 13 de Maio Tres Rios RJ
Ana Letcia de Lima Santos Positivo Ang. Sampaio Curitiba PR
Giovani Florencio Scarpelli Junior IFRJ Rio de Janeiro RJ
Fbio Kenji Arai Etapa So Paulo SP
OBQ-2012 Fase III
>> Olimpada Brasileira de Qumica - 2012
73
Isadora Santos Bittar Integrado Ja Goinia GO
Gabriel Castro Tavares Militar de Manaus Manaus AM
Orestes Fontana Romeiro Militar de Curitiba Curitiba PR
Gabriela Costa Caetano Santo Agostinho Goinia GO
Joo Bosco de Carvalho Jnior COLTEC Belo Horizonte MG
Leonardo Gonalves Chiquita Marista Paranaense Curitiba PR
Everton de Arajo Franco IFRJ Rio de Janeiro RJ
Ivana Rodrigues Silva IFPE Recife PE
Nikolas Henrique Oliveira Liberato Salzano Novo Hamburgo RS
Bruno Variani Carpeggiani CETEC - Centro Tec. da UCS Caxias do Sul RS
Andr Pozzobon Capeletti Politcnico da UFSM Santa Maria RS
Fabrcio Rossi Marques Matias Albert Sabin So Paulo SP
Camila de Moura Fortes Anglo Lavoisier Teresina PI
Daniel Meira Nobrega de Lima Geo -Tambau Joao Pessoa PB
caro Sampaio Viana 7 de setembro Fortaleza CE
Clisostenes Arruda Barbosa IFPB Campina Grande PB
Pedro Henrique Caldeira Porto So Francisco Xavier Ipatinga MG
D E M A I S C L A S S I F I C A D O S
Ver lista em www.obquimica.org
Resultados
Programa Nacional Olimpadas de Qumica <<
74
MODALIDADE B
O U R O
Nome Escola Cidade UF
Vitria Nunes Medeiros Farias Brito Fortaleza
CE
Daniel Arjona de Andrade Hara Objetivo Cotia
SP
Gabriel Mathews Viana Pinheiro MASTER Fortaleza
CE
Arthur Eduardo Pastore de Lima UTFPR Medianeira
PR
Jlio Csar Prado Soares Militar de Brasilia Brasilia
DF
P R A T A
Nome Escola Cidade UF
Bruno Fernando Abreu de Melo Ponto de Ensino Rio de Janeiro RJ
Ramon Santos Gonalves da Silva Ideal Belm PA
Bruno Matissek Worm Tiradentes Porto Alegre RS
Henrique Magri Maron Etapa Valinhos SP
merson Holanda Marinho Farias Brito Fortaleza CE
Giuvanni Mutton Etapa Itatiba SP
Henrique Gasparini Fiuza do Nascimento Militar de Brasilia Brasilia DF
Ivan Monteiro Padalko Etapa So Paulo SP
Joo Lucas Maehara Said dos Reis IFES Maratazes ES
Tiago Follmann Perin Liberato Salzano N. Hamburgo RS
Francisco Davi Barbosa dos Santos Farias Brito Fortaleza CE
Marco Aurlio B. Cardoso Bandeirantes So Paulo SP
Daniel de Oliveira Sampaio Vasconcelos e S GGE Recife PE
Vtor Juc Policarpo 7 de setembro Fortaleza CE
Brandon Wahib Bogossian Khalil IFJR Rio de Janeiro RJ
Matheus Salmito Rodrigues Ponte Farias Brito Fortaleza CE
Flvio Luis Schneider Junior Militar de Manaus Manaus AM
OBQ-2012 Fase III
>> Olimpada Brasileira de Qumica - 2012
75
B R O N Z E
Nome Escola Cidade UF
Andr Santos Fernandes Albert Sabin Osasco SP
Tssio Perotti Arruda MASTER Fortaleza CE
Rogrio Jos Menezes Alves Leonardo da Vinci Vitria ES
Igor Lucas Rodrigues Dias COLTEC Contagem MG
Lucas Niemeyer Carneiro Messina de Assis IFJR Rio de Janeiro RJ
Mateus Compart Hemerly Leonardo da Vinci Vitria ES
Gabriel Prescinotti Vivan PGD Londrina PR
Pablo talo do Nascimento Santos Santa Rita Picos PI
Pedro Henrique Almeida Fraiman Cincias Aplicadas Natal RN
Rafael Ribeiro Alves Motiva C. Grande PB
Rogrio Yuuki Motisuki Etapa So Paulo SP
Leonardo Zeine Mendes de Souza Militar de Brasilia Brasilia DF
Gustavo Xavier de Jesus IFJR Rio de Janeiro RJ
Vinicius Lima Ferreira IFJR Rio de Janeiro RJ
Srgio Pereira de Oliveira Jnior Farias Brito Fortaleza CE
Juliana Schenhel Zotti E. E. E. M. Gustavo Biazus Tupanci do Sul RS
MENO HONROSA
Nome Escola Cidade UF
Eduardo Martins Neto IFJR Rio de Janeiro RJ
Ana Raquel Ferraz Rameiro Col. Aplicao da UFPE Recife PE
Yan Fonseca dos Santos IFJR Rio de Janeiro RJ
Amadeo Zimermann IFSC So Pedro de Alcntara SC
Victor Hugo V. S. Oliveira IFJR Rio de Janeiro RJ
Rayssa Oliveira do Nascimento IFJR Duque de caxias RJ
Rafael Lucas Gouveia COLTEC Belo Horizonte MG
Juliana Barbosa Echenique IFJR Rio de Janeiro RJ
Luiz Felipeh Aguiar de Lima Alves IFJR Rio de Janeiro RJ
Hugo de Albuquerque Meira Santa Maria Recife PE
Resultados
Programa Nacional Olimpadas de Qumica <<
76
Lucas Costa Machado CEFET Ipatinga MG
Ian Elmr Lang Santa Cruz So Paulo SP
Lucas Gabriel de Barros Silva Militar Porto Alegre Porto Alegre RS
Brenda Breda Passon Unio de Professores Vila Velha ES
Ricardo Kazu Nakanishi Podicon Brasilia DF
Carolina Larissa Machado COLTEC Belo Horizonte MG
Talize Fac de Paula Pessoa Queiroz Farias Brito Fortaleza CE
Leonardo de Andrade Mesquita Militar Porto Alegre Porto Alegre RS
talo Pena de Oliveira Coluni Viosa MG
Jeferson de Queiroz Gomes Farias Brito Fortaleza CE
Pedro Henrique Rodrigues da Mota CEI Natal RN
Gustavo Henrique P. C. de Albuquerque GGE Recife PE
Renato Xavier de Jesus IFJR Rio de Janeiro RJ
Mariana Camyla Duarte Pontes FBA Fortaleza CE
Renan Picoli de Souza COC Vitria ES
Rafael Eller Cruz Olimpo Brasilia DF
Gabriel Ilharco Magalhes IFJR Rio de Janeiro RJ
Mateus Leite Pucciarelli Etapa So Paulo SP
Diego Rodrigues Carvalho Antonio Vieira Salvador BA
Fernanda Moura Borges Santa Rita Picos PI
Mariana Lcia Branco Zeitune IFJR Rio de Janeiro RJ
Juliana Silva Brasil MASTER Fortaleza CE
Victor Venturi Etapa Campinas SP
Eduardo Poleze Bertoni Foz do Iguau PR
Athos Cotta Couto Charles Darwin Vitria ES
Igor Chiasso Marques Santa Maria Jaboto dos Guararapes PE
Dbora Barreto Ornellas Anchieta Salvador BA
Wendrey Lustosa Cardoso Militar de Manaus Manaus AM
Renato Frankiley da Silva Lima GGE Recife PE
Pedro Silveira Quintana Politcnico da UFSM Santa Maria RS
OBQ-2012 Fase III
>> Olimpada Brasileira de Qumica - 2012
77
Thiago Oliveira Rodrigues Santa Rita Picos PI
Bruna Favoretto Regular Mau SP
Mariane Greice Pereira Ventura IFBA Salvador BA
Carolina Konrdrfer Rangel Liberato Salzano Novo Hamburgo RS
Daniel Costa Xavier de Oliveira Olimpo Goiania GO
Jlia Carvalinho de Souza So Domingos Vitria ES
Ana Paula Carvalho Simes Ifba - Salvador Salvador BA
Ncolas Mouro Lataro Hoehne COLTEC Belo Horizonte MG
Niander Neves de Assis CEFET-MG - Campus 2 Belo Horizonte MG
D E M A I S C L A S S I F I C A D O S
Ver lista em www.obquimica.org
Resultados
Programa Nacional Olimpadas de Qumica <<
78
Processo Seletivo para as Olimpadas Internacionais
Seletiva para a 44
th
International Chemistry Olympiad, Estados Unidos
Filme exibido em 14.01.2012, 14:00 horas (Horrio de Braslia)
Caro estudante,
Este exame de cunho experimental tem por fnalidade selecionar os 15 (quinze)
estudantes que participaro do XI Curso de Aprofundamento e Excelncia que
ser ministrado na UFMG (Belo Horizonte) no perodo de 06 a 17 de fevereiro
prximo. Um novo exame ser aplicado no dia 17 de maro prximo para defnir
a equipe (4 estudantes) que representar o Brasil na 44
th
IChO que acontecer em
julho prximo, em Washington DC - USA.
Voc dispe de 3 (trs) horas para ver o vdeo e responder s questes deste exame.
I N S T R U E S
1. Veja atentamente, no televisor ou tela de projeo, as imagens do flme que
contm os fundamentos deste exame. Tempo de exibio: 13 (treze) minutos.
2. Seu coordenador, inicialmente, exibir a gravao completa do exame e, a
seguir, apresentar cada parte separadamente. Se necessrio, repassar as ima-
gens vrias vezes at esclarecer suas dvidas.
3. Leia as perguntas relativas a cada experimento e escreva as respostas nos es-
paos reservados para tal, nas folhas ofciais de respostas.
4. A prova contm 7 (sete) questes, que abrangem os 6 (seis) experimentos con-
tidos no vdeo.
5. Os resultados sero encaminhados at 07/04 para seu coordenador (e tambm
diretamente para voc, caso tenha e-mail). Veja o resultado, tambm, na inter-
net em www.obquimica.org (clique em novidades).
Processo Seletivo
>> Olimpada Brasileira de Qumica - 2012
79
QUE S T I ON R I O
QUESTO 1 (EXPERIMENTO 1)
Explique o surgimento da colorao brilhante observada em um dos erlenmeyers.
Processo Seletivo para as Olimpadas Internacionais
Seletiva para a 44th International Chemistry Olympiad, Estados Unidos
Filme exibido em 14.01.2011, 14:00 horas (Horrio de Braslia)

Caro estudante,

Este exame de cunho experimental tem por finalidade selecionar os 15
(quinze) estudantes que participaro do XI Curso de Aprofundamento e Excelncia
que ser ministrado na UFMG (Belo Horizonte) no perodo de 06 a 17 de fevereiro
prximo. Um novo exame ser aplicado no dia 17 de maro prximo para definir a
equipe (4 estudantes) que representar o Brasil na 44
th
IChO que acontecer em julho
prximo, em Washington DC - USA.
Voc dispe de 3 (trs) horas para ver o vdeo e responder s questes
deste exame.


I N S T R U E S

1. Veja atentamente, no televisor, as imagens do filme que contm os fundamentos
deste exame. Tempo de exibio: 13 (treze) minutos.
2. Seu coordenador, inicialmente, exibir a gravao completa do exame e, a seguir,
apresentar cada parte separadamente. Se necessrio, repassar as imagens
vrias vezes at esclarecer suas dvidas.
3. Leia as perguntas relativas a cada experimento e escreva as respostas nos
espaos reservados para tal, nas folhas oficiais de respostas.
4. A prova contm 7 (sete) questes, que abrangem os 6 (seis) experimentos contidos
no vdeo.
5. Os resultados sero encaminhados at 07/04 para seu coordenador (e tambm
diretamente para voc, caso tenha e-mail). Veja o resultado, tambm, na internet
em www.obquimica.org (clique em novidades).

QUESTIONRIO

QUESTO 1 (EXPERIMENTO 1)
Explique o surgimento da colorao brilhante
observada em um dos erlenmeyers.




Quinina
QUESTO 2 (EXPERIMENTO 2)
Utilize um modelo terico para explicar as diferentes cores das chamas observadas
nas cpsulas de porcelana.

QUESTO 3 (EXPERIMENTO 3)
Utilizando as equaes das reaes qumicas observadas em cada procedimento,
identifique o tubo que contm o lcool primrio.

QUESTO 4 (EXPERIMENTO 4)
Escreva as 5 equaes balanceadas para esse ciclo de reaes do cobre.

QUESTO 5 (EXPERIMENTO 5)
Considere os seguintes resultados
Condies
Teste
1 2 3
Volume de H
2
C
2
O
4
(mL) 5,0 10,0 5,0
Volume de KMnO
4
(mL) 1,0 1,0 2,0
Volume de H
2
O (mL) 6,0 1,0 5,0
Tempo de reao (minutos) 14 6 11
Determine a lei de velocidade para a reao entre o cido oxlico e o permanganato.

QUESTO 6 (EXPERIMENTO 6)
Calcule a constante de estabilidade do on complexo formado.

QUESTO 7 (TODOS OS EXPERIMENTOS)
Indique procedimentos laboratoriais incorretos realizados em algumas dessas
atividades experimentais.
QUESTO 2 (EXPERIMENTO 2)
Utilize um modelo terico para explicar as diferentes cores das chamas observa-
das nas cpsulas de porcelana.
QUESTO 3 (EXPERIMENTO 3)
Utilizando as equaes das reaes qumicas observadas em cada procedimento,
identifque o tubo que contm o lcool primrio.
QUESTO 4 (EXPERIMENTO 4)
Escreva as 5 equaes balanceadas para esse ciclo de reaes do cobre.
QUESTO 5 (EXPERIMENTO 5)
Considere os seguintes resultados
Condies
Teste
1 2 3
Volume de H
2
C
2
O
4
(mL) 5,0 10,0 5,0
Volume de KMnO
4
(mL) 1,0 1,0 2,0
Volume de H
2
O (mL) 6,0 1,0 5,0
Tempo de reao (minutos) 14 6 11
OBQ-2012 Fase IV
Programa Nacional Olimpadas de Qumica <<
80
Determine a lei de velocidade para a reao entre o cido oxlico e o permanga-
nato.
QUESTO 6 (EXPERIMENTO 6)
Calcule a constante de estabilidade do on complexo formado.
QUESTO 7 (TODOS OS EXPERIMENTOS)
Indique procedimentos laboratoriais incorretos realizados em algumas dessas
atividades experimentais.
Processo Seletivo
>> Olimpada Brasileira de Qumica - 2012
81
Olimpada Brasileira de Qumica - 2011 Fase IV
Resultado da prova experimental - 2012
Os estudantes que participaram da Fase IV da Olimpada Brasileira de Qumica -
2012 (etapa inicial do processo seletivo para a formao das delegaes represen-
taram o Brasil nas olimpadas internacionais) obtiveram os seguintes resultados no
exame de conhecimento de laboratrio.
Nome Escola Cidade Resultado
Gabriel Matheus Viana Pinheiro Mster Fortaleza 96.28
Joaquim Ivo Vasques Dantas Landim Paraiso Juazeiro do Norte 95.71
Vitria Nunes Medeiros Farias Brito Fortaleza 92.25
Bruno Matissek Worm Tiradentes Porto Alegre 91.49
Francisco Davi Barbosa dos Santos Farias Brito Fortaleza 91.00
Daniel Arjona de Andrade Hara Objetivo Cotia 90.54
Nicholas de Souza Costa Lima Ari de S Fortaleza 89.93
Vinicius Lopes Braga Dom Barreto Teresina 89.76
Sergio Pereira de Oliveira Jnior Farias Brito Fortaleza 88.24
Ramon Santos Gonalves da Silva Ideal Militar Belm 88.16
Lvia Rodrigues de Arajo Farias Brito Fortaleza 87.99
Kelvin Azevedo dos Santos Uniclass Objetivo Goinia 87.25
Nathrcia Castro Mota Master Fortaleza 84.67
Emerson Holanda Marinho Farias Brito Fortaleza 84.55
Henrique Magri Maron Etapa Valinhos 84.49
Leonardo Kazunori Tsuji Etapa So Paulo 84.33
OBQ Fase IV
Programa Nacional Olimpadas de Qumica <<
82
Olimpada Brasileira de Qumica - 2011 Fase V
Curso de Aprofundamento e Excelncia em Qumica
O Curso de Aprofundamento e Excelncia em Qumica foi ministrado pelo Depar-
tamento de Qumica da Universidade Federal de Minas Gerais no perodo de 6 a
17 de fevereiro de 2012. Participaram os quinze estudantes mais bem classifcados
aps a aplicao do exame de conhecimentos de laboratrio (mdia ponderada
das notas na Fase III e Fase IV). Aps a concluso deste curso os professores en-
vilvidos elaboraram e avaliaram os exames da etapa fnal da olimpada (Fase VI).

F
o
t
o
:

I
n

c
i
o

H
a
r
a
Da esquerda para a direita: Nathercia Mota, Bruno Worm, Emerson Marinho, Daniel
Hara, Matheus Salmito, Francisco Barbosa, Vitria Meedeiros, Leonardo Tsuji, Kelvin
Santos, Ramon Silva, Joaquim Ivo, Gabriel Pinheiro, Nicholas Lima, Henrique Magri e
Vincius Braga.
Processo Seletivo
>> Olimpada Brasileira de Qumica - 2012
83
Na UFMG os estudantes participaram de atividades culturais e discutiram com
os professores do Departamento de Qumica temas variados de qumica e as-
suntos relevantes sugeridos pelos organizadores da 44
th
International Chemistry
Olympiad. Alm da oportunidade de se aprofundar nos estudos de qumica os
estudantes ampliam o crculo de amizades com os demais participantes, discutem
com professores dos cursos de ps-graduao em qumica das universidades par-
ceiras, participam das atividades e conhecem o potencial da universidade anftri .
Na imerso que fazem nesta Quinzena Olmpica, os estudantes aprofundam temas
pouco abordados no ensino mdio e, por conseguinte, retornam s suas cidades
enriquecidos de ideias e mais motivados para o estudo dessa cincia.
Estudantes em sala de aula.
OBQ Fase V
Programa Nacional Olimpadas de Qumica <<
84
Olimpada Brasileira de Qumica
OBQ - 2011 FASE VI
Seletiva para a 44
th
International Chemistry Olympiad
Washington - USA 17.03.2012
CADERNO DE QUESTES
QUESTO 1

I) Em 1998, Lugez e co-autores (J. Chem. Phys., 108, 9639 (1998)), investigaram
um mistura de Ne-SF
6
atravs de clculos tericos e de experimentos com foto-
-ionizacao. Espcies inicas foram detectadas em armadilhas de nenio slido
(trapped in solid neon). Eles detectaram as seguintes espcies: SF
5
+
, SF
5
-
, SF
3
+
.
a) Escreva a estrutura de Lewis de todas as espcies.
b) Baseando-se no mtodo VSEPR, proponha a geometria para as 3 espcies de-
tectadas neste estudo.
c) Sugira o conjunto de orbitais hbridos do tomo central para cada uma das
espcies.
d) Algumas destas espcies poderia ser sintetizada substituindo o enxofre pelo
oxignio? Justifque a sua resposta em termos de TLV.
II) Um dos ons conhecidos mais estvel o VO
2
+
.
a) Desenhe esquematicamente a estrutura de Lewis do VO
2
+
.
b) Utilize o modelo de VSEPR para propor a geometria deste on.
c) Sabe-se que o ngulo O-V-O do on VO
2
+
de 105,5
o
. Recentemente, Duarte
e colaboradores [ J. Inorg. Chem, 72 (1998), 71-77], demonstraram que o n-
gulo de 105,5
o
pode ser explicado a partir da TLV utilizando orbitais hbridos
adequados.
Proponha uma explicao a partir da TLV, porque o on VO
2
+
angular. Mostre es-
quematicamente os orbitais hbridos formados.
Processo Seletivo
>> Olimpada Brasileira de Qumica - 2012
85
OBQ Fase VI
QUESTO 2

Considere as estruturas dos ligantes apresentadas abaixo.

Cisteamina Glicinato

Quando o on glicinato reage com ons de Pt
2+
, forma-se um complexo de geome-
tria quadrada. Por outro lado, quando ons cloreto e cisteamina reagem com ons
de Cr
3+
, forma-se um complexo octadrico monometlico, que contm o mesmo
nmero dos dois ligantes. Sabendo que os ligantes cisteamina e glicinato so bi-
dentados, indique o nmero total de ismeros para cada complexo e apresente as
estruturas de todos os ismeros.
QUESTO 3

Solues tampo so solues que atenuam a variao dos valores de pH, man-
tendo-o aproximadamente constante, mesmo com adio de pequenas quantida-
des de cidos ou de bases fortes.
As solues tampo podem ser formadas por um par-cido base conjugado e, so
usadas sempre que se necessita de um meio com pH aproximadamente constante.
Elas so preparadas dissolvendo-se os solutos em gua.
Em estudos ligados medicina e biologia muito importante o conceito de so-
luo tampo, pois os fuidos biolgicos (animais ou vegetais) so, em geral, meios
aquosos tamponados.
Um dos sistemas tampes mais importantes o do sangue, que permite a manu-
teno das trocas gasosas e das protenas. O pH do sangue de 7,4 e o principal
sistema tampo um equilbrio entre o cido carbnico e o on a ele associado, o
bicarbonato. Este sistema evita variaes de 0.3 unidades de pH as quais poderiam
trazer graves consequncias ao ser humano.
Considere as solues preparadas pela
I dissoluo de 8,00 mmol de acetato de sdio em 200 mL de soluo de cido actico
0,100 mol/L.
Programa Nacional Olimpadas de Qumica <<
86
II adio de 100 mL de soluo de hidrxido de sdio 0,0500 mol/L a 100 mL de
soluo de cido actico 0,175 mol/L.
III adio de 5,5104 g de acetato de sdio a 40,0 mL de soluo de cido clordrico
0,1200 mol/L e diluio at completar 200 mL.
a) Calcular o pH de cada uma dessas trs solues.
b) Qual ser o pH da soluo c se a mesma for diluda 100 vezes. Justifque a
sua resposta.
c) Qual ser a variao de pH (pH) se a 100 mL da soluo c forem adiciona-
dos:
c.1) 2,0 mL de soluo de cido clordrico 0,1 mol/L.
c.2) 2,0 mL de soluo de hidrxido de sdio 0,1 mol/L.
Dado: Ka(CH
3
COOH) = 1,8x10
-5
QUESTO 4

a) Calcule o G

do processo de solubilizao do cloreto de prata e cloreto de


sdio. Discuta o signifcado dos valores encontrados.
Dados termodinmicos a 25
o
C
Substncia fH

/ kJ mol
-1
fG

/ kJ mol
-1
Sm

/ J K
1
mol
-1
AgCl (s) -127,1 -110 +96,2
Ag
+
(aq) +105,6 +77,1 +72,7
Cl

(aq) -167,2 -131,2 +56,5


NaCl (s) -411,1 -384,1 +72,1
Na
+
(aq) -240,1 -261,9 +59
b) Utilizando somente as informaes da variao de entropia do processo de so-
lubilizao e da vizinhana, explique a diferena de solubilidade desses com-
postos.
Processo Seletivo
>> Olimpada Brasileira de Qumica - 2012
87
c) O ciclo de Born-Haber pode ser utilizado para o clculo da entalpia de formao
de um composto slido.

Quais etapas do ciclo de Born-Haber que so determinantes para a diferena ob-
servada na entalpia de formao do NaCl e AgCl?
d) Explique por que a energia do retculo do NaCl menor do que a do AgCl.
e) As diferenas dos valores calculados tericos e os experimentos das energias
dos retculos do NaCl so maiores do que as observadas para o AgCl. Explique a
razo desta diferena.
OBQ Fase VI
Programa Nacional Olimpadas de Qumica <<
88
Composto
Energia do retculo (kJ mol
-1
)
Experimental Born-Haber Terico
AgCl 905 770
NaCl 771 766
QUESTO 5

Um experimento calorimtrico foi realizado para a determinao do calor de com-
busto completa do antraceno. Esta substncia um hidrocarboneto aromtico
slido, de massa molar 178,23 g mol
_1
, incolor mas que apresenta uma fuorescn-
cia azul quando irradiado por uma fonte de radiao ultravioleta.
O calormetro utilizado, uma bomba calorimtrica adiabtica, tem como com-
ponentes principais um vaso de reao construdo com ao inoxidvel com suas
paredes espessas e rgidas para suportar uma presso de oxignio at 20 atm e um
sistema eltrico de ignio das amostras a serem investigadas. Todo experimento
calorimtrico realizado no interior deste calormetro feito sob o regime de volu-
me constante. A alta presso de gs oxignio no vaso de reao necessria para
que seja assegurada a combusto completa das amostras estudadas.
A reao de combusto neste equipamento iniciada ao passar uma corrente el-
trica atravs de um pequeno fo fno de ferro que mantido em contado com a
amostra. O vaso de reao imerso em um banho de gua que mantido isolado
das vizinhanas. O calormetro utilizado do tipo adiabtico, isto , todo o calor
liberado pela reao permanece no calormetro; nada perdido para as vizinhan-
as. O calor liberado na reao de combusto absorvido pela massa do banho de
gua e outras partes do colormetro. Este processo causa um aumento na tempe-
ratura interna no calormetro. O aumento da temperatura no interior deste calor-
metro medida por um termmetro sensvel previamente adaptado ao sistema.
Utilizando um calormetro adiabtico como descrito acima, uma amostra de 1,228
g de antraceno foi queimada em um calormetro com um banho de gua de 2 L.
A presso de O
2
no incio da reao marcava 20 atm. Um fo de ferro de 10,00 cm
foi utilizado inicialmente na a ignio do sistema e, aps o experimento, 0,65 cm
intacto restante deste fo foi coletado. Neste experimento a temperatura medida
no termmetro imerso no banho da gua utilizada registrou uma variao do ser
valor inicial de 23,74 C at o valor fnal constante de 30,28 C.
Processo Seletivo
>> Olimpada Brasileira de Qumica - 2012
89
Considerando os valores aproximados para a capacidade trmica especfca e den-
sidade da gua como c(H
2
O,l)=1,0 cal g
_1
grau
_1
e r(H
2
O,l)=1,0 g ml
_1
. A capacidade
trmica do calormetro adiabtico utilizado vale C
cal
=881 J grau
_1
e o calor de com-
busto linear por comprimento de fo de ferro vale
1
cm J 1 , 0 ) Fe (

=
linear
comb
Q .
Sabe-se tambm que 1 cal = 4,184 J e que a constante dos gases ideias tem o valor
de R=8,314 J K
-1
mol
_1
. Utilizando os dados acima, determine a entalpia de combus-
to completa DH
comb
do antraceno.
QUESTO 6
A) Explique as observaes abaixo. Mostre o mecanismo e as conformaes em cadeira
envolvidas.
CH
3
Cl
NaOEt
CH
3
CH
3
+
CH
3
Cl
NaOEt
CH
3
250 vezes mais
lento
B) D o produto formado nas reaes abaixo. Inclua a estereoqumica.
F
1. O
3
2. (CH
3
)
2
S
KMnO
4
,
H
3
O
+
CHBr
3
1. Hg(OAc)
2
, MeOH
2. NaBH
4
1
.
B
H
3 ,
T
H
F
2
.
H
2 O
2 ,
N
a
O
H
HBr
KOH
OBQ Fase VI
Programa Nacional Olimpadas de Qumica <<
90
QUESTO 7

A) Na sntese da halenaquinona, realizada por Rodrigo et AL (J. Or. Chem. 66, 3639-
3641, 2001), esquema abaixo, as etapas que levam formao dos compostos X
e Y, envolvem reaes de Diels-Alder.

Desenhe as estruturas dos compostos X e Y, incluindo a correta estereoqumica.
B) Entre as classes de reaes pericclicas tem-se os rearranjos sigmatrpicos. A
seguir est representada uma reao, realizada sob aquecimento, que envolve
dois rearranjos sigmatrpicos e conduz tambm a um aldedo cclico de cinco
membros como produto principal. Represente a estrutura deste aldedo.

O 382
0
C
10 segundos
A
Aldedo B
Processo Seletivo
>> Olimpada Brasileira de Qumica - 2012
91
Olimpada Brasileira de Qumica - 2011 Fase VI
Resultado da prova Final - 2012
(Relao dos estudantes convocados para compor a delegao brasileira na
Olimpada Internacional de Qumica - Washington)
Nome Escola Estado Resultado
Anterior
Resultado
Final
Gabriel Matheus Viana Pinheiro Mster CE 96.37
96.34
Ramon Santos Gonalves da Silva Ari de S CE 100.00
96.05
Daniel Arjona de Andrade Hara Objetivo SP 96.16
94.28
Vitria Nunes Medeiros Farias Brito CE 94.45
93.71
Kelvin Azevedo dos Santos Uniclass Objetivo GO 90.39
89.34
Emerson Holanda Marinho Farias Brito CE 89.53
87.87
Francisco Davi Barbosa dos Santos Farias Brito CE 85.79
87.53
Leonardo Kazunori Tsuji Etapa SP 88.55
87.14
Joaquim Ivo Vasques Dantas Landim Farias Brito CE 80.66
85.68
Henrique Magri Maron Etapa SP 82.91
83.44
Nicholas de Souza Costa Lima Ari de S CE 76.28
80.83
Matheus Salmito Rodrigues Ponte Farias Brito CE 78.85
80.59
Lvia Rodrigues de Arajo Farias Brito CE 61.33
70.21
Bruno Matissek Worm Tiradentes RS 51.07
64.54
Alunos Marcados em NEGRITO integraram a delegao que representou o Brasil
nas olimpadas realizadas em Washington (44
th
IChO) e em Santa F (17 OIAQ)
OBQ Fase VI
Programa Nacional Olimpadas de Qumica <<
92
44
th
IChO
44
th
International
Chemistry Olympiad
Exame Terico
21-30 julho de 2012 Washington, DC
Estados Unidos da Amrica
PROBLEMA 1
ai aii a-iii b c Problema 1
7,5%
4 2 2 2 10 20

a. Hidretos de Boro e outros compostos de Boro
A qumica dos hidretos de Boro foi inicialmente desenvolvida por Alfred Stock
(1876-1946). Foram caracterizadas mais de 20 formas moleculares neutras de hi-
dretos de boro, de frmula geral B
x
H
y
. A mais simples de todas a do diborano,
B
2
H
6
.
i. Utilizando os dados fornecidos na tabela, deduza a frmula molecular para
dois outros membros, A e B, desta srie de hidretos de boro.
Substncia Estado fsico
(25 C, 1 bar)
Percentagem de
Boro (em massa)
Massa Molar
(g/mol)
A Lquido 83,1 65,1
B Slido 88,5 122,2
>> Olimpada Brasileira de Qumica - 2012
93
Exame Terico
ii. William Lipscomb recebeu o Prmio Nobel da Qumica em 1976 pelos seus es-
tudos sobre as estruturas de hidretos de boro e o esclarecimento de problemas
relacionados com problemas relacionados com a ligao qumica. Lipscomb
reconheceu que em todos os hidretos de boro, cada tomo B estabelece uma liga-
o simples com 2 eltrons com pelo menos um tomo de H (B-H). Contudo, como
possvel estabelecer outros tipos de ligaes adicionais, ele desenvolveu um
mtodo para descrever a estrutura de um borano fornecendo o nmero styx,
onde:
s = o nmero de pontes BHB na molcula
t = o nmero de centros com 3-ligaes BBB na molcula

B
B
B
y = o nmero de centros com 2-ligaes BB na molcula
x = o nmero de grupos BH2 na molcula
O nmero styx para o B
2
H
6
2002. Proponha a estrutura para o tetraborano, B
4
H
10
,
sabendo que o seu nmero styx o 4012.
iii. Considere um composto contendo boro, carbono, cloro e oxignio (B
4
CCl
6
O). As
medidas espectroscpicas indicam que a molcula tem dois tipos de tomos de B,
com geometria tetradrica e triangular planar, na razo 1:3 respectivamente. Estes
dados espectroscpicos so tambm consistentes com uma ligao tripla CO.
Considerando que a frmula molecular do composto B
4
CCl
6
O, sugira a estrutura
desta molcula.
b. Termoqumica dos compostos de Boro
Estime a entalpia de dissociao da ligao simples B-B no B
2
Cl
4
(g) usando as infor-
maes disponveis:
Ligao Entalpia de dissociao da ligao (kJ/mol)
BCl 443
ClCl 242
Composto D
f
H (kJ/mol)
BCl
3
(g) 403
B2Cl
4
(g) 489
Programa Nacional Olimpadas de Qumica <<
94
c. Qumica dos Diboranos
Escreva a estrutura de cada um dos compostos (1-5) do esquema abaixo. Cada n-
mero corresponde a um composto contendo boro.
DADOS:
a. O ponto de ebulio do composto 5 55 C.
b. Em todas as reaes utlizado um excesso de reagente.
c. O abaixamento do ponto de congelamento de 0,312 g do composto 2 em 25.0
g de benzeno 0,205 C. A constante crioscpica do benzeno 5,12 C/molal
PROBLEMA 2 7,8% do total
ai aii b-i b-ii c Problema 2 7,8%
4 4 6 1 5 20

Compostos de Platina(II), Ismeros e efeito Trans.
A Platina e outros metais do grupo 10 formam complexos quadrados planares e
os mecanismos das suas reaes foram extensivamente estudados. Por exemplo,
sabe-se que as reaes de substituio nestes complexos ocorrem com a reteno
da estereoqumica.
T L
Pt
L X
T L
Pt
L Y
+ Y + X

44
th
IChO
>> Olimpada Brasileira de Qumica - 2012
95
Exame Terico
Sabe-se tambm que a velocidade da substituio do ligante X pelo Y depende
da natureza do ligante em posio trans em relao ao X, ou seja o ligante T. Isto
denominado efeito trans. Quando T uma das molculas ou ions indicados na
lista abaixo, a velocidade de substituio na posio trans diminui da esquerda
para a direita.
CN

> H

> NO
2
, I

> Br

, Cl

> piridina, NH
3
, OH

, H
2
O
A preparao de cis- e trans-Pt(NH
3
)
2
Cl
2
depende do efeito trans. A preparao do
ismero cis, um agente anticancergeno utilizado em quimioterapia, comercial-
mente designado por cisplatin, envolve a reao de K
2
PtCl
4
com amnia.


DADOS:
a. O ponto de ebulio do composto 5 55 C.
b. Em todas as reaes utlizado um excesso de reagente.
c. O abaixamento do ponto de congelamento de 0,312 g do composto 2 em 25.0 g de
benzeno 0,205 C. A constante crioscpica do benzeno 5,12 C/molal


PROBLEMA 2 7,8% do total

ai aii b-i b-ii c Problema 2 7,8%
4 4 6 1 5 20


Compostos de Platina(II), Ismeros e efeito Trans.
A Platina e outros metais do grupo 10 formam complexos quadrados planares e os
mecanismos das suas reaes foram extensivamente estudados. Por exemplo, sabe-se
que as reaes de substituio nestes complexos ocorrem com a reteno da
estereoqumica.


Sabe-se tambm que a velocidade da substituio do ligante X pelo Y depende da
natureza do ligante em posio trans em relao ao X, ou seja o ligante T. Isto
denominado efeito trans. Quando T uma das molculas ou ions indicados na lista
abaixo, a velocidade de substituio na posio trans diminui da esquerda para a
direita.

CN

> H

> NO
2

, I

> Br

, Cl

> piridina, NH
3
, OH

, H
2
O

A preparao de cis- e trans-Pt(NH
3
)
2
Cl
2
depende do efeito trans. A preparao do
ismero cis, um agente anticancergeno utilizado em quimioterapia, comercialmente
designado por cisplatin, envolve a reao de K
2
PtCl
4
com amnia.



i. Represente as estruturas de todos os possveis estereoismeros para o compos-
to Pt(py)(NH
3
)BrCl (onde py = piridina, C
5
H
5
N) tendo em conta que a platina(II)
apresenta uma geometria quadrado planar.
ii. Escreva os esquemas das reaes qumicas, incluindo os intermedirios, que
representam a preparao em soluo aquosa de cada um dos estereoismeros
[Pt(NH
3
)(NO
2
)Cl
2
] usando como reagentes PtCl
4
2-
, NH
3
e NO
2

. Considere que
as reaes so controladas cineticamente pelo efeito trans.


i. Represente as estruturas de todos os possveis estereoismeros para o composto
Pt(py)(NH
3
)BrCl (onde py = piridina, C
5
H
5
N) tendo em conta que a platina(II)
apresenta uma geometria quadrado planar.


ii. Escreva os esquemas das reaes qumicas, incluindo os intermedirios, que
representam a preparao em soluo aquosa de cada um dos estereoismeros
[Pt(NH
3
)(NO
2
)Cl
2
]

usando como reagentes PtCl


4
2-
, NH
3
e NO
2

. Considere que as
reaes so controladas cineticamente pelo efeito trans.


Ismero cis:





Ismero trans:






b. Estudos Cinticos de Reaes de Substituio de Complexos Quadrados
Planares
Substitues de ligantes X por Y em complexos quadrados planares
ML
3
X + Y ML
3
Y + X
podem ocorrer por dois processos:

Substituio Direta: O ligante Y que entra liga-se ao metal central, formando um
complexo de nmero de coordenao cinco, que rapidamente elimina o ligante X para
formar o porduto ML
3
Y.
Pt
Cl
Cl
Cl
Cl
2
Pt
Cl
Cl
Cl
Cl
2


i. Represente as estruturas de todos os possveis estereoismeros para o composto
Pt(py)(NH
3
)BrCl (onde py = piridina, C
5
H
5
N) tendo em conta que a platina(II)
apresenta uma geometria quadrado planar.


ii. Escreva os esquemas das reaes qumicas, incluindo os intermedirios, que
representam a preparao em soluo aquosa de cada um dos estereoismeros
[Pt(NH
3
)(NO
2
)Cl
2
]

usando como reagentes PtCl


4
2-
, NH
3
e NO
2

. Considere que as
reaes so controladas cineticamente pelo efeito trans.


Ismero cis:





Ismero trans:






b. Estudos Cinticos de Reaes de Substituio de Complexos Quadrados
Planares
Substitues de ligantes X por Y em complexos quadrados planares
ML
3
X + Y ML
3
Y + X
podem ocorrer por dois processos:

Substituio Direta: O ligante Y que entra liga-se ao metal central, formando um
complexo de nmero de coordenao cinco, que rapidamente elimina o ligante X para
formar o porduto ML
3
Y.
Pt
Cl
Cl
Cl
Cl
2
Pt
Cl
Cl
Cl
Cl
2
Programa Nacional Olimpadas de Qumica <<
96
b. Estudos Cinticos de Reaes de Substituio de Complexos Quadrados
Planares
Substitues de ligantes X por Y em complexos quadrados planares
ML
3
X + Y ML
3
Y + X
podem ocorrer por dois processos:
Substituio Direta: O ligante Y que entra liga-se ao metal central, formando um
complexo de nmero de coordenao cinco, que rapidamente elimina o ligan-
te X para formar o porduto ML
3
Y.




** = etapa determinante da velocidade da reao, constante de velocidade = k
Y


Substituio assistida por solvente: Uma molcula do solvente S liga-se ao metal
central para dar ML
3
XS, que elimina X para dar ML
3
S. Rapidamente, Y substitui S para
dar ML
3
Y.


** = etapa determinante da velocidade da reao, constante de velocidade = k
S


A lei da velocidade total para estas substituies
constante de velocidade = k
s
[ML
3
X] + k
Y
[Y][ML
3
X]
Quando [Y] >> [ML
3
X], ento a constante de velocidade = k
obs
[ML
3
X].

Os valores de k
s
e k
Y
dependem dos reagentes e do solvente envolvidos. Um exemplo
a substituio do ligante Cl

num complexo quadrado planar de platina(II), ML


2
X
2
, pela
piridina (C
5
H
5
N). (O que foi citado acima para o ML
3
X aplica-se ao ML
2
X
2
.)



A tabela que se segue apresenta os dados da reao a 25 C em metanol, onde [piridina]
>> que a concentrao do complexo de platina.
ML
3
X [ML
3
XY]
+ Y
**
X
ML
3
Y
ML
3
X [ML
3
XS]
+ S
**
X
ML
3
Y [ML
3
S]
+ Y
S
N
N
Pt
Cl
Cl
N
+
N
N
Pt
Cl N
+ Cl
-
CH
3
OH
** = etapa determinante da velocidade da reao, constante de velocidade = kY
Substituio assistida por solvente: Uma molcula do solvente S liga-se ao me-
tal central para dar ML
3
XS, que elimina X para dar ML
3
S. Rapidamente, Y substi-
tui S para dar ML
3
Y.




** = etapa determinante da velocidade da reao, constante de velocidade = k
Y


Substituio assistida por solvente: Uma molcula do solvente S liga-se ao metal
central para dar ML
3
XS, que elimina X para dar ML
3
S. Rapidamente, Y substitui S para
dar ML
3
Y.


** = etapa determinante da velocidade da reao, constante de velocidade = k
S


A lei da velocidade total para estas substituies
constante de velocidade = k
s
[ML
3
X] + k
Y
[Y][ML
3
X]
Quando [Y] >> [ML
3
X], ento a constante de velocidade = k
obs
[ML
3
X].

Os valores de k
s
e k
Y
dependem dos reagentes e do solvente envolvidos. Um exemplo
a substituio do ligante Cl

num complexo quadrado planar de platina(II), ML


2
X
2
, pela
piridina (C
5
H
5
N). (O que foi citado acima para o ML
3
X aplica-se ao ML
2
X
2
.)



A tabela que se segue apresenta os dados da reao a 25 C em metanol, onde [piridina]
>> que a concentrao do complexo de platina.
ML
3
X [ML
3
XY]
+ Y
**
X
ML
3
Y
ML
3
X [ML
3
XS]
+ S
**
X
ML
3
Y [ML
3
S]
+ Y
S
N
N
Pt
Cl
Cl
N
+
N
N
Pt
Cl N
+ Cl
-
CH
3
OH
** = etapa determinante da velocidade da reao, constante de velocidade = kS
A lei da velocidade total para estas substituies
constante de velocidade = ks[ML
3
X] + kY[Y][ML
3
X]
Quando [Y] >> [ML
3
X], ento a constante de velocidade = k
obs
[ML
3
X].
Os valores de ks e kY dependem dos reagentes e do solvente envolvidos. Um exem-
plo a substituio do ligante Cl num complexo quadrado planar de platina(II),
ML
2
X
2
, pela piridina (C
5
H
5
N). (O que foi citado acima para o ML
3
X aplica-se ao ML
2
X
2
.)




** = etapa determinante da velocidade da reao, constante de velocidade = k
Y


Substituio assistida por solvente: Uma molcula do solvente S liga-se ao metal
central para dar ML
3
XS, que elimina X para dar ML
3
S. Rapidamente, Y substitui S para
dar ML
3
Y.


** = etapa determinante da velocidade da reao, constante de velocidade = k
S


A lei da velocidade total para estas substituies
constante de velocidade = k
s
[ML
3
X] + k
Y
[Y][ML
3
X]
Quando [Y] >> [ML
3
X], ento a constante de velocidade = k
obs
[ML
3
X].

Os valores de k
s
e k
Y
dependem dos reagentes e do solvente envolvidos. Um exemplo
a substituio do ligante Cl

num complexo quadrado planar de platina(II), ML


2
X
2
, pela
piridina (C
5
H
5
N). (O que foi citado acima para o ML
3
X aplica-se ao ML
2
X
2
.)



A tabela que se segue apresenta os dados da reao a 25 C em metanol, onde [piridina]
>> que a concentrao do complexo de platina.
ML
3
X [ML
3
XY]
+ Y
**
X
ML
3
Y
ML
3
X [ML
3
XS]
+ S
**
X
ML
3
Y [ML
3
S]
+ Y
S
N
N
Pt
Cl
Cl
N
+
N
N
Pt
Cl N
+ Cl
-
CH
3
OH
A tabela que se segue apresenta os dados da reao a 25 C em metanol, onde
[piridina] >> que a concentrao do complexo de platina.
44
th
IChO
>> Olimpada Brasileira de Qumica - 2012
97
Concentrao de piridina (mol/L) k
obs
(s
-1
)
0,122 7,20 x 10
-4
0,061 3,45 x 10
-4
0,030 1,75 x 10
-4
i. Calcule os valores de ks e kY. Indique as unidades corretas para cada constante.
Voc pode utilizar o quadriculado fornecido abaixo.

ii. Qual a frase que verdadeira quando a [piridina] = 0,10 mol/L?. (Assinale o qua-
drado correspondente resposta que voc considera verdadeira.)
A maior parte do produto contendo piridina formado pelo processo de
substituio assistido por solvente (ks)
A maior parte do produto contendo piridina formado pelo processo de
substituio direta (kY)
Quantidades semelhantes de produto contendo piridina so formadas pe-
los dois processos
No podem ser tiradas concluses relativamente quantidade de produto
formado por cada processo.

Exame Terico
Programa Nacional Olimpadas de Qumica <<
98
c. Agente quimioteraputico
Num esforo para melhorar a seletividade do cisplatin para as clulas tumorais, o
grupo do Professor Lippards do MIT ligou um complexo de platina(IV) a oligonu-
cleotdeos ligados a nanopartculas de ouro.
nanopartculas de ouro Oligonucleotdeo Complexo de Pt(IV) ligado
Os experimentos utilizam nanopartculas de ouro com dimetro de 13 nm. Ligado
a cada nanopartcula esto 90 oligonucleotdeos, sendo que 98% deles esto liga-
dos ao complexo de Pt(IV). Suponha que o recipiente usado para tratar as clulas
com a nanopartcula de Pt(IV) tem um volume de 1,0 mL e que a soluo tem uma
concentrao em Pt de 1,0 x 10
-6
M. Calcule a massa de ouro e de platina usada
nesta experincia. (A densidade do ouro 19,3 g/cm
3
)
PROBLEMA 3 7,5 % Total
a b c-i c-ii Problema 3
7,5%
4 12 6 12 34

Ions de tiomolibdato so obtidos a partir de ions molibdato, MoO
4
2-
, por substi-
tuio dos tomos de oxignio por enxofre. Na natureza, os ions tiomolibdato so
formados em locais como as guas profundas do mar negro, onde a reduo bio-
lgica de sulfato gera H
2
S. A transformao do molibdato em tiomolibdato origina
a perda rpida de Mo dissolvido na gua do mar que sedimenta sob a forma de
44
th
IChO
>> Olimpada Brasileira de Qumica - 2012
99
Exame Terico
tiomolibdato e, consequentemente, as quantidades deste elemento essencial para
a vida decresce no oceano.
Os seguintes equilbrios em soluo aquosa controlam as concentraes relativas
dos ies molibdato e tiomolibdato em soluo.


MoS
4
2-
+ H
2
O(l) MoOS
3
2-
+ H
2
S(aq) K
1
= 1,310
-5

MoOS
3
2-
+ H
2
O(l) MoO
2
S
2
2-
+ H
2
S(aq) K
2
= 1,010
-5

MoO
2
S
2
2-
+ H
2
O(l) MoO
3
S
2-
+ H
2
S(aq) K
3
= 1,610
-5

MoO
3
S
2-
+ H
2
O(l) MoO
4
2-
+ H
2
S(aq) K
4
= 6,510
-6


a. Se uma soluo em equilbrio contm MoO
4
2
110
-7
M
-
e H
2
S(aq) 110
-6
M, qual ser
a concentrao de MoS
4
2-
?

Solues contendo MoO
2
S
2
2-
, MoOS
3
2-
e MoS
4
2-
apresentam mximos de absorbncia na
regio do visvel, na faixa de comprimento de onda de 395 e 468 nm. A absorbncia dos
outros ions, bem como a do H
2
S, so desprezveis na regio do visvel. As absortividades
molares () nestes dois comprimentos de onda so dados na tabela seguinte:

a 468 nm
L mol
-1
cm
-1

a 395 nm
L mol
-1
cm
-1

MoS
4
2-
11870 120
MoOS
3
2-
0 9030
MoO
2
S
2
2-
0 3230
b. Uma soluo, que no est em equilbrio, contm a mistura de MoS
4
2-
, MoOS
3
2-
e
MoO
2
S
2
2-
e no contm nenhuma outra espcie de Mo. A concentrao total de todas as
espcies de Mo 6,010
-6
M. Em uma cela de 10,0 cm, a absorbncia de uma soluo em
468 nm 0,365 e em 395 nm 0,213,. Calcule as concentraes das 3 espcies de Mo
existentes na mistura.

c. A soluo inicial contendo MoS
4
2-
2,010
-7
M hidrolisada num sistema fechado. O
H
2
S produzido acumula at o equilbrio ser atingido. Calcule as concentraes finais, no
equilbrio, de H
2
S(aq) e dos 5 nions contendo Mo (isto , MoO
4
2-
, MoO
3
S
2-
, MoO
2
S
2
2-
,
MoOS
3
2-
e MoS
4
2-
). Ignore a possibilidade do H
2
S se ionizar em HS
-
em determinadas
condies de pH. (um tero da pontuao ser dada pela escrita das seis equaes
matemticas importantes para o sistema e 2/3 da pontuao sero dados para o clculo
correto das concentraes).

i. Escreva as seis equaes matemticas importantes para o sistema.
a. Se uma soluo em equilbrio contm MoO
4
2-
110
-7
M e H
2
S(aq) 110
-6
M, qual
ser a concentrao de MoS
4
2-
?
Solues contendo MoO
2
S
2
2-
, MoOS
3
2-
e MoS
4
2-
apresentam mximos de absorbncia
na regio do visvel, na faixa de comprimento de onda de 395 e 468 nm. A absorbncia
dos outros ions, bem como a do H
2
S, so desprezveis na regio do visvel. As absorti-
vidades molares () nestes dois comprimentos de onda so dados na tabela seguinte:


MoS
4
2-
+ H
2
O(l) MoOS
3
2-
+ H
2
S(aq) K
1
= 1,310
-5

MoOS
3
2-
+ H
2
O(l) MoO
2
S
2
2-
+ H
2
S(aq) K
2
= 1,010
-5

MoO
2
S
2
2-
+ H
2
O(l) MoO
3
S
2-
+ H
2
S(aq) K
3
= 1,610
-5

MoO
3
S
2-
+ H
2
O(l) MoO
4
2-
+ H
2
S(aq) K
4
= 6,510
-6


a. Se uma soluo em equilbrio contm MoO
4
2
110
-7
M
-
e H
2
S(aq) 110
-6
M, qual ser
a concentrao de MoS
4
2-
?

Solues contendo MoO
2
S
2
2-
, MoOS
3
2-
e MoS
4
2-
apresentam mximos de absorbncia na
regio do visvel, na faixa de comprimento de onda de 395 e 468 nm. A absorbncia dos
outros ions, bem como a do H
2
S, so desprezveis na regio do visvel. As absortividades
molares () nestes dois comprimentos de onda so dados na tabela seguinte:

a 468 nm
L mol
-1
cm
-1

a 395 nm
L mol
-1
cm
-1

MoS
4
2-
11870 120
MoOS
3
2-
0 9030
MoO
2
S
2
2-
0 3230
b. Uma soluo, que no est em equilbrio, contm a mistura de MoS
4
2-
, MoOS
3
2-
e
MoO
2
S
2
2-
e no contm nenhuma outra espcie de Mo. A concentrao total de todas as
espcies de Mo 6,010
-6
M. Em uma cela de 10,0 cm, a absorbncia de uma soluo em
468 nm 0,365 e em 395 nm 0,213,. Calcule as concentraes das 3 espcies de Mo
existentes na mistura.

c. A soluo inicial contendo MoS
4
2-
2,010
-7
M hidrolisada num sistema fechado. O
H
2
S produzido acumula at o equilbrio ser atingido. Calcule as concentraes finais, no
equilbrio, de H
2
S(aq) e dos 5 nions contendo Mo (isto , MoO
4
2-
, MoO
3
S
2-
, MoO
2
S
2
2-
,
MoOS
3
2-
e MoS
4
2-
). Ignore a possibilidade do H
2
S se ionizar em HS
-
em determinadas
condies de pH. (um tero da pontuao ser dada pela escrita das seis equaes
matemticas importantes para o sistema e 2/3 da pontuao sero dados para o clculo
correto das concentraes).

i. Escreva as seis equaes matemticas importantes para o sistema.
b. Uma soluo, que no est em equilbrio, contm a mistura de MoS
4
2-
, MoOS
3
2-
e
MoO
2
S
2
2-
e no contm nenhuma outra espcie de Mo. A concentrao total de
todas as espcies de Mo 6,010
-6
M. Em uma cela de 10,0 cm, a absorbncia de
uma soluo em 468 nm 0,365 e em 395 nm 0,213,. Calcule as concentraes
das 3 espcies de Mo existentes na mistura.
c. A soluo inicial contendo MoS
4
2-
2,010
-7
M hidrolisada num sistema fecha-
do. O H
2
S produzido acumula at o equilbrio ser atingido. Calcule as concentra-
es fnais, no equilbrio, de H
2
S(aq) e dos 5 nions contendo Mo (isto , MoO
4
2-
,
MoO
3
S
2-
, MoO
2
S
2
2-
, MoOS
3
2-
e MoS
4
2-
). Ignore a possibilidade do H
2
S se ionizar
em HS
-
em determinadas condies de pH. (um tero da pontuao ser dada
pela escrita das seis equaes matemticas importantes para o sistema e 2/3 da
pontuao sero dados para o clculo correto das concentraes).
Programa Nacional Olimpadas de Qumica <<
100
i. Escreva as seis equaes matemticas importantes para o sistema.
ii. Calcule as seis concentraes, efetuando aproximaes razoveis (vlidas) e
apresente a sua resposta com dois algarismos signifcativos.


ii. Calcule as seis concentraes, efetuando aproximaes razoveis (vlidas) e apresente
a sua resposta com dois algarismos significativos.


H
2
S __________ MoO
4
2-
_________ MoO
3
S
2-
__________



MoO
2
S
2
2-
_______ MoOS
3
2-
__________ MoS
4
2-
___________



PROBLEMA 4 7,8% do Total
a b c d-i d-ii d-iii d-iv e-i e-ii Problema 4
7,8%
12 14 10 4 2 2 4 4 8 60

Nos anos 1980 foi descoberta uma classe de materiais cermicos que exibe su-
percondutividade pouco habitual a temperaturas de 90 K. Um desses materiais
contm trio, brio, cobre e oxignio e designado por YBCO. Ele tem uma com-
posio molecular YBa
2
Cu
3
O
7
, mas a sua composio real varia de acordo com a
formula YBa
2
Cu
3
O
7-d
(0 < d < 0,5).
a. Uma clula unitria de um cristal ideal YBCO apresentada abaixo. Faa a cor-
respondncia entre cada crculo e o respectivo elemento existente na estrutura.


PROBLEMA 4 7,8% do Total

a b c d-i d-ii d-iii d-iv e-i e-ii Problema 4
7,8% 12 14 10 4 2 2 4 4 8 60


Nos anos 1980 foi descoberta uma classe de materiais cermicos que exibe
supercondutividade pouco habitual a temperaturas de 90 K. Um desses materiais contm
trio, brio, cobre e oxignio e designado por YBCO. Ele tem uma composio
molecular YBa2Cu3O7, mas a sua composio real varia de acordo com a formula
YBa2Cu3O7-
(0 < < 0,5).

a. Uma clula unitria de um cristal ideal YBCO apresentada abaixo. Faa a
correspondncia entre cada crculo e o respectivo elemento existente na estrutura.








=



=



=



=




a
b
c
44
th
IChO
>> Olimpada Brasileira de Qumica - 2012
101
Exame Terico
A estrutura verdadeira na realidade uma estrutura ortorrmbica (a b c), mas
aproximadamente tetragonal, com a b (c/3).
b. Uma amostra de YBCO com d = 0,25 foi submetida a difrao de Raios-X, usando
radiao Cu Kal (l = 154,2 pm). O menor ngulo do pico de difrao foi obser-
vado a 2q = 7,450. Assumindo que a = b = (c/3), calcule os valores de a e c.
c. Calcule a densidade desta amostra de YBCO (com d = 0,25) em g cm
-3
. Se no
tiver obtido valores para a e c no item (b), ento utilize a = 500 pm, c = 1500 pm.
d. Quando o YBCO dissolvido numa soluo aquosa de HCl 1,0 M libertam-se
bolhas de um gs (identifcado por cromatografa gasosa como sendo O
2
). Aps
o aquecimento ebulio, durante 10 min, para remover os gases dissolvidos,
adiciona-se um excesso de soluo de KI que torna a soluo anterior amarelo-
-acastanhada. Esta soluo pode ser titulada com uma soluo de tiossulfato,
usando amido para determinar o ponto fnal, Se o YBCO for adicionado dire-
tamente a uma soluo contendo KI e HCl, ambos na concentrao de 1,0 M,
numa atmosfera de Ar (argnio), a soluo fca amarelo-acastanhada, mas no
se observa liberao de gs.
i. Escreva a equao inica, devidamente balanceada, correspondente reao
que ocorre quando o slido YBa
2
Cu
3
O
7-d
dissolvido na soluo aquosa de HCl
e ocorre desprendimento de O
2
.
ii. Escreva a equao inica, devidamente balanceada, correspondente reao
que ocorre quando a soluo obtida em (i) reage com excesso de KI, em condi-
es cidas e depois do oxignio dissolvido ter sido eliminado.
iii. Escreva a equao inica, devidamente balanceada, correspondente reao
que ocorre quando a soluo obtida em (ii) titulada com tiossulfato (S
2
O
3
2-
).
iv. Escreva a equao inica, devidamente balanceada, correspondente reao
que ocorre quando o slido YBa
2
Cu
3
O
7-d
dissolvido numa soluo aquosa de
HCl contendo excesso de KI e sob atmosfera de Ar.
e. Foram preparadas duas amostras idnticas de YBCO com valores de d desco-
nhecidos. A primeira amostra foi dissolvida em 5 mL de uma soluo aquosa
de HCl 1,0 M e liberou O
2
. Aps o aquecimento ebulio para libertar os gases
dissolvidos, resfriamento e adio, sob atmosfera de Ar, de 10 mL de uma solu-
o aquosa de KI a 0,7 M, foi titulada com tiossulfato, utilizando amido como in-
dicador. At atingir o ponto fnal, foram gastos 1,542 x 10
-4
mols de tiossulfato.
A segunda amostra de YBCO foi adicionada diretamente a 7 mL de uma soluo
contendo KI a 1,0 M e HCl a 0,7 M sob atmosfera de Ar. Na sua titulao foram
gastas 1,696 x 10
-4
mols de tiossulfato at ser atingido o ponto fnal.
Programa Nacional Olimpadas de Qumica <<
102
i. Calcule o nmero de mols de Cu em cada uma das amostras de YBCO.
ii. Calcule o valor de d para estas amostras de YBCO.
PROBLEMA 5 7.0 % do Total
a-i a-ii b c d e f Problema 5
7.0%
2 4 4 2 12 6 4 34

O cido desoxirribonucleico (DNA) uma das molculas fundamentais para a vida.
Esta questo ir examinar formas pelas quais a estrutura molecular do DNA pode
ser modifcada, naturalmente ou por caminhos elaborados pela humanidade.
a. Considere as bases pirimidnicas, citosina (C) e timina (T). O tomo de N-3 (indi-
cado por *) de uma destas bases um stio nucleoflico comum para a alquila-
o do DNA, enquanto que o da outra base no o .
i. Selecione (circule) qual base, C ou T, tem o tomo N-3 mais nucleoflico.



ii. Calcule o valor de para estas amostras de YBCO.


PROBLEMA 5 7.0 % do Total






O cido desoxirribonucleico (DNA) uma das molculas fundamentais para a vida. Esta
questo ir examinar formas pelas quais a estrutura molecular do DNA pode ser
modificada, naturalmente ou por caminhos elaborados pela humanidade.

a. Considere as bases pirimidnicas, citosina (C) e timina (T). O tomo de N-3 (indicado
por *) de uma destas bases um stio nucleoflico comum para a alquilao do DNA,
enquanto que o da outra base no o .
i. Selecione (circule) qual base, C ou T, tem o tomo N-3 mais nucleoflico



ii. Desenhe duas estruturas de ressonncia complementares da molcula que voc
selecionou, para justificar sua resposta. Indique todas as cargas formais, diferentes de
zero, sobre os tomos das estruturas de ressonncia que voc desenhou

b.Uma modificao comum do DNA na natureza a metilao na posio marcada (*) na
estrutura da guanina (G) pela S-adenosil-metionina (SAM). Desenhe as estruturas de
ambos os produtos da reao entre guanina e SAM.



a-i a-ii b c d e f Problema 5
7.0% 2 4 4 2 12 6 4 34

ii. Desenhe duas estruturas de ressonncia complementares da molcula que
voc selecionou, para justifcar sua resposta. Indique todas as cargas formais,
diferentes de zero, sobre os tomos das estruturas de ressonncia que voc de-
senhou.
b. Uma modifcao comum do DNA na natureza a metilao na posio marca-
da (*) na estrutura da guanina (G) pela S-adenosil-metionina (SAM). Desenhe as
estruturas de ambos os produtos da reao entre guanina e SAM.



ii. Calcule o valor de para estas amostras de YBCO.


PROBLEMA 5 7.0 % do Total






O cido desoxirribonucleico (DNA) uma das molculas fundamentais para a vida. Esta
questo ir examinar formas pelas quais a estrutura molecular do DNA pode ser
modificada, naturalmente ou por caminhos elaborados pela humanidade.

a. Considere as bases pirimidnicas, citosina (C) e timina (T). O tomo de N-3 (indicado
por *) de uma destas bases um stio nucleoflico comum para a alquilao do DNA,
enquanto que o da outra base no o .
i. Selecione (circule) qual base, C ou T, tem o tomo N-3 mais nucleoflico



ii. Desenhe duas estruturas de ressonncia complementares da molcula que voc
selecionou, para justificar sua resposta. Indique todas as cargas formais, diferentes de
zero, sobre os tomos das estruturas de ressonncia que voc desenhou

b.Uma modificao comum do DNA na natureza a metilao na posio marcada (*) na
estrutura da guanina (G) pela S-adenosil-metionina (SAM). Desenhe as estruturas de
ambos os produtos da reao entre guanina e SAM.



a-i a-ii b c d e f Problema 5
7.0% 2 4 4 2 12 6 4 34


44
th
IChO
>> Olimpada Brasileira de Qumica - 2012
103
Exame Terico
c. Um dos primeiros agentes alquilantes do DNA feitos pelo homem foi o gs
mostarda.



c. Um dos primeiros agentes alquilantes do DNA feitos pelo homem foi o gs mostarda.


Atravs de uma reao intramolecular, o gs mostarda forma primeiramente o
intermedirio A, o qual, alquila diretamente o DNA, produzindo o cido nucleico
mostrado na equao acima. Desenhe a estrutura do intermedirio A.

d. A mostarda de nitrognio reage via um caminho anlogo ao da mostarda de enxofre,
do item parte c. A reatividade do composto pode ser modificada, dependendo do terceiro
substituinte no tomo de nitrognio. A reatividade da mostarda de nitrognio aumenta
com o aumento da nucleofilicidade do tomo de nitrognio central. Selecione o reagente
mais reativo e menos reativo dos seguintes grupos de mostardas de nitrognio.

i.


I II III



MAIS REATIVO:

MENOS REATIVO:


ii.


I II III

MAIS REATIVO:
N
Cl Cl
N
Cl Cl
N
Cl Cl
NO
2
NO
2
N
Cl Cl
N
Cl Cl
N
Cl Cl
OCH
3
NO
2
Atravs de uma reao intramolecular, o gs mostarda forma primeiramente o
intermedirio A, o qual, alquila diretamente o DNA, produzindo o cido nucleico
mostrado na equao acima. Desenhe a estrutura do intermedirio A.
d. A mostarda de nitrognio reage via um caminho anlogo ao da mostarda de
enxofre, do item parte c. A reatividade do composto pode ser modifcada, de-
pendendo do terceiro substituinte no tomo de nitrognio. A reatividade da
mostarda de nitrognio aumenta com o aumento da nucleoflicidade do tomo
de nitrognio central. Selecione o reagente mais reativo e menos reativo dos
seguintes grupos de mostardas de nitrognio.
i.



c. Um dos primeiros agentes alquilantes do DNA feitos pelo homem foi o gs mostarda.


Atravs de uma reao intramolecular, o gs mostarda forma primeiramente o
intermedirio A, o qual, alquila diretamente o DNA, produzindo o cido nucleico
mostrado na equao acima. Desenhe a estrutura do intermedirio A.

d. A mostarda de nitrognio reage via um caminho anlogo ao da mostarda de enxofre,
do item parte c. A reatividade do composto pode ser modificada, dependendo do terceiro
substituinte no tomo de nitrognio. A reatividade da mostarda de nitrognio aumenta
com o aumento da nucleofilicidade do tomo de nitrognio central. Selecione o reagente
mais reativo e menos reativo dos seguintes grupos de mostardas de nitrognio.

i.


I II III



MAIS REATIVO:

MENOS REATIVO:


ii.


I II III

MAIS REATIVO:
N
Cl Cl
N
Cl Cl
N
Cl Cl
NO
2
NO
2
N
Cl Cl
N
Cl Cl
N
Cl Cl
OCH
3
NO
2

MAIS REATIVO:
MENOS REATIVO:
ii.



c. Um dos primeiros agentes alquilantes do DNA feitos pelo homem foi o gs mostarda.


Atravs de uma reao intramolecular, o gs mostarda forma primeiramente o
intermedirio A, o qual, alquila diretamente o DNA, produzindo o cido nucleico
mostrado na equao acima. Desenhe a estrutura do intermedirio A.

d. A mostarda de nitrognio reage via um caminho anlogo ao da mostarda de enxofre,
do item parte c. A reatividade do composto pode ser modificada, dependendo do terceiro
substituinte no tomo de nitrognio. A reatividade da mostarda de nitrognio aumenta
com o aumento da nucleofilicidade do tomo de nitrognio central. Selecione o reagente
mais reativo e menos reativo dos seguintes grupos de mostardas de nitrognio.

i.


I II III



MAIS REATIVO:

MENOS REATIVO:


ii.


I II III

MAIS REATIVO:
N
Cl Cl
N
Cl Cl
N
Cl Cl
NO
2
NO
2
N
Cl Cl
N
Cl Cl
N
Cl Cl
OCH
3
NO
2

Programa Nacional Olimpadas de Qumica <<
104
MAIS REATIVO:
MENOS REATIVO:
iii.



MENOS REATIVO:



iii.

I II III

MAIS REATIVO:

MENOS REATIVO:


e. Algumas classes de produtos naturais atuam como alquilantes do DNA e, desta forma,
tm potencial para servir como terapias para o cncer, devido sua atividade antitumoral.
As duocarmicinas constituem uma dessas classes. Abaixo so mostradas as etapas de uma
sntese assimtrica total desse produto natural. Desenhe as estruturas de compostos
isolveis J e K.



f.Pequenas molculas relacionadas foram sintetizadas para estudar a maneira como as
duocarmicinas atuam. Um exemplo o tioster mostrado abaixo. Desenhe a estrutura do
intermedirio Z.
N
Cl Cl
Me
N
Cl Cl
N
Cl Cl
CH
3
O OCH
3
O
MAIS REATIVO:
MENOS REATIVO:
e. Algumas classes de produtos naturais atuam como alquilantes do DNA e, desta
forma, tm potencial para servir como terapias para o cncer, devido sua ativi-
dade antitumoral. As duocarmicinas constituem uma dessas classes. Abaixo so
mostradas as etapas de uma sntese assimtrica total desse produto natural.
Desenhe as estruturas de compostos isolveis J e K.



MENOS REATIVO:



iii.

I II III

MAIS REATIVO:

MENOS REATIVO:


e. Algumas classes de produtos naturais atuam como alquilantes do DNA e, desta forma,
tm potencial para servir como terapias para o cncer, devido sua atividade antitumoral.
As duocarmicinas constituem uma dessas classes. Abaixo so mostradas as etapas de uma
sntese assimtrica total desse produto natural. Desenhe as estruturas de compostos
isolveis J e K.



f.Pequenas molculas relacionadas foram sintetizadas para estudar a maneira como as
duocarmicinas atuam. Um exemplo o tioster mostrado abaixo. Desenhe a estrutura do
intermedirio Z.
N
Cl Cl
Me
N
Cl Cl
N
Cl Cl
CH
3
O OCH
3
O
f. Pequenas molculas relacionadas foram sintetizadas para estudar a maneira
como as duocarmicinas atuam. Um exemplo o tioster mostrado abaixo. De-
senhe a estrutura do intermedirio Z.
44
th
IChO
>> Olimpada Brasileira de Qumica - 2012
105
Exame Terico






PROBLEMA 6 6.6 % do Total





Vareniclina foi desenvolvida para o tratamento oral contra o vcio de fumar e pode ser
sintetizada pela via mostrada abaixo. Todos os compostos indicados por uma letra (A -
H) so espcies isolveis, sem cargas.

a b c d Problema 6
6.6% 2 4 6 8 20

PROBLEMA 6 6.6 % do Total
a b c d Problema 6
2 4 6 8 20 6.6%

Vareniclina foi desenvolvida para o tratamento oral contra o vcio de fumar e pode
ser sintetizada pela via mostrada abaixo. Todos os compostos indicados por uma
letra (A - H) so espcies isolveis, sem cargas.




Programa Nacional Olimpadas de Qumica <<
106
a. Sugira uma estrutura para o composto A.
b. Sugira uma estrutura para o composto B consistente com os seguintes dados
de 1H-RMN data:
7.75 (singleto, 1H), 7.74 (dupleto, 1H, J = 7.9 Hz), 7.50 (dupleto, 1H, J = 7.1 Hz),
7.22 (multipleto, 2 H no equivlentes), 4.97 (tripleto, 2H, J = 7.8 Hz), 4.85 (tripleto,
2H, J = 7.8 Hz)



a. Sugira uma estrutura para o composto A.

b. Sugira uma estrutura para o composto B consistente com os seguintes dados de
1
H-
RMN data:
7.75 (singleto, 1H), 7.74 (dupleto, 1H, J = 7.9 Hz), 7.50 (dupleto, 1H, J = 7.1 Hz), 7.22
(multipleto, 2 H no equivlentes), 4.97 (tripleto, 2H, J = 7.8 Hz), 4.85 (tripleto, 2H, J =
7.8 Hz)


1
H RMN - Faixas de deslocamentos qumicos

12.0 11.0 10.0 9.0 8.0 7.0 6.0 5.0 4.0 3.0 2.0 1.0 0.0
RCH=O
Aromatics
R
2
C=CH
2
Alkyl-H
RHC=CHR
RCCH
PhO-CH ArCH R
2
C=CR-CH
F-CH Cl-CH I-CH
Br-CH
RC(=O)-CH
RCO
2
-CH NC-CH
O
2
N-CH
R
2
N-CH
ROH
R
2
NH
PhOH
RCONH
RCOOH
(ppm)
c. Sugira uma estrutura para cada composto, C, D, e F.
d. Sugira os reagentes X e Y indicados para converter o composto G em vareni-
clina, e fornea o intermedirio isolvel H.
44
th
IChO
>> Olimpada Brasileira de Qumica - 2012
107
Exame Terico
PROBLEMA 7 7.5 % do Total
a b c d e f Problema 7
7.5%
9 15 8 6 8 6 52

Uma enzima artifcial foi concebida para se ligar s duas molculas de substrato-
mostradas abaixo (dieno e dienflo) e catalisar uma reao de Diels-Alder entre
elas.
a. H oito produtos que potencialmente se formam a partir de uma reao de
Diels-Alder envolvendo estas duas molculas na reao sem qualquer enzima.
i. Desenhe as estruturas de quaisquer dois dos
potenciais produtos que so regioismeros um
do outro, nas caixas que so dadas abaixo. Use
cunhas ( ) e traos ( ) para mostrar a es-
tereoqumica de cada produto em seus desenhos.
Use R e R mostrados abaixo, para representar os
substituintes nas molculas que no esto direta-
mente envolvidas na reao.
ii. Nos espaos reservados abaixo, desenhe as estru-
turas de quaisquer dois dos potenciais produtos
que so enantimeros um do outro. Use cunhas (
) e traos ( ) para mostrar a estereoqumi-
ca de cada produto em seus desenhos. Use R e R
como no item (i).
iii. Desenhe as estruturas de quaisquer dois dos potenciais produtos que so
diastermeros um do outro. Use cunhas ( ) e traos ( ) para mostrar a
estereoqumica de cada produto em seus desenhos. Use R e R como no item (i).
b. A velocidade e a regiosseletividade de uma reao de Diels-Alder dependem do
grau de complementaridade eletrnica entre os dois reagentes. As estruturas
do dieno e do dienflo da parte a so dadas abaixo.
i. Circule o tomo de carbono no dieno que tem a densidade de eltrons aumen-
tada e, portanto, pode atuar como um doador de eltrons durante a reao. De-
senhe, no espao abaixo, uma estrutura de ressonncia do dieno que justifca
sua resposta. Indique todas as cargas as cargas formais diferentes de zero sobre
os tomos na estrutura de ressonncia que voc desenhou.


c. Sugira uma estrutura para cada composto, C, D, e F.


d. Sugira os reagentes X e Y indicados para converter o composto G em vareniclina, e
fornea o intermedirio isolvel H.

PROBLEMA 7 7.5 % do Total

a b c d e f Problema 7
7.5%
9 15 8 6 8 6 52


Uma enzima artificial foi concebida para se ligar s duas molculas de substrato
mostradas abaixo (dieno e dienfilo) e catalisar uma reao de Diels-Alder entre elas.

a. H oito produtos que potencialmente se formam a partir de uma reao de Diels-Alder
envolvendo estas duas molculas na reao sem qualquer enzima.


i. Desenhe as estruturas de quaisquer dois dos potenciais
produtos que so regioismeros um do outro, nas caixas
que so dadas abaixo. Use cunhas ( ) e traos ( )
para mostrar a estereoqumica de cada produto em seus
desenhos. Use R e R' mostrados abaixo, para representar
os substituintes nas molculas que no esto diretamente
envolvidas na reao.








ii. Nos espaos reservados abaixo, desenhe as estruturas de quaisquer dois dos potenciais
produtos que so enantimeros um do outro. Use cunhas ( ) e traos ( ) para mostrar
a estereoqumica de cada produto em seus desenhos. Use R e R' como no item (i).
C O 2
-
O
O N
H
R
O
N
M e M e
R '
CO
2
-
O
O N
H
diene
O
N
Me Me
dienophile
Programa Nacional Olimpadas de Qumica <<
108


iii. Desenhe as estruturas de quaisquer dois dos potenciais produtos que so
diastermeros um do outro. Use cunhas ( ) e traos ( ) para mostrar a
estereoqumica de cada produto em seus desenhos. Use R e R' como no item (i).

b. A velocidade e a regiosseletividade de uma reao de Diels-Alder dependem do grau
de complementaridade eletrnica entre os dois reagentes. As estruturas do dieno e do
dienfilo da parte a so dadas abaixo.


i. Circule o tomo de carbono no dieno que tem a densidade de eltrons aumentada e,
portanto, pode atuar como um doador de eltrons durante a reao. Desenhe, no espao
abaixo, uma estrutura de ressonncia do dieno que justifica sua resposta. Indique todas as
cargas as cargas formais diferentes de zero sobre os tomos na estrutura de ressonncia
que voc desenhou.






ii. Circule o tomo de carbono no dienfilo que tem a densidade de eltrons diminuda e,
portanto, pode atuar como um receptor de eltrons durante a reao. Desenhe no espao
abaixo uma estrutura de ressonncia do dienfilo que justifique sua resposta. Indique
todas as cargas as cargas formais diferentes de zero sobre os tomos na estrutura de
ressonncia que voc desenhou.


ii. Circule o tomo de carbono no dienflo que tem a densidade de eltrons dimi-
nuda e, portanto, pode atuar como um receptor de eltrons durante a reao.
Desenhe no espao abaixo uma estrutura de ressonncia do dienflo que justi-
fque sua resposta. Indique todas as cargas as cargas formais diferentes de zero
sobre os tomos na estrutura de ressonncia que voc desenhou.






iii. Com base nas suas atribuies feitas nos itens (i) e (ii), prediga a regioqumica da
reao
no
catalisad
a de
Diels-
Alder do
dieno e
dienfil
o,
desenha
ndo o
produto
principal
. Voc
no
precisa
mostrar
a
estereoq
umica
do produto em seu desenho.

c. A figura abaixo mostra os reagentes de Diels-Alder como eles esto ligados, antes de
entrar no estado de transio para a formao de um produto no stio ativo da enzima
artificial. A rea cinzenta representa um corte transversal atravs da enzima. O dienfilo
est abaixo do plano da seo transversal, ao passo que o dieno est acima do plano de
seo transversal, quando as duas molculas so ligados no stio ativo que est mostrado.
Desenhar a estrutura do produto da reao catalisada por enzima na caixa abaixo. Mostre
em seu desenho a estereoqumica do produto e use R e R' conforme voc fez para uma
parte a.

d. Considere as seguintes afirmaes sobre enzimas (artificial ou natural). Para cada
situao, indique se essa afirmao Verdadeira ou Falsa (desenhe um crculo em torno
de "Verdadeiro" ou "Falso").

O
N
Me Me
iii. Com base nas suas atribui-
es feitas nos itens (i) e
(ii), prediga a regioqumica
da reao no catalisada
de Diels-Alder do dieno e
dienflo, desenhando o
produto principal. Voc no
precisa mostrar a estereo-
qumica do produto em seu
desenho.







iii. Com base nas suas atribuies feitas nos itens (i) e (ii), prediga a regioqumica da
reao
no
catalisad
a de
Diels-
Alder do
dieno e
dienfil
o,
desenha
ndo o
produto
principal
. Voc
no
precisa
mostrar
a
estereoq
umica
do produto em seu desenho.

c. A figura abaixo mostra os reagentes de Diels-Alder como eles esto ligados, antes de
entrar no estado de transio para a formao de um produto no stio ativo da enzima
artificial. A rea cinzenta representa um corte transversal atravs da enzima. O dienfilo
est abaixo do plano da seo transversal, ao passo que o dieno est acima do plano de
seo transversal, quando as duas molculas so ligados no stio ativo que est mostrado.
Desenhar a estrutura do produto da reao catalisada por enzima na caixa abaixo. Mostre
em seu desenho a estereoqumica do produto e use R e R' conforme voc fez para uma
parte a.

d. Considere as seguintes afirmaes sobre enzimas (artificial ou natural). Para cada
situao, indique se essa afirmao Verdadeira ou Falsa (desenhe um crculo em torno
de "Verdadeiro" ou "Falso").

O
N
Me Me
44
th
IChO
>> Olimpada Brasileira de Qumica - 2012
109
Exame Terico
c. A fgura abaixo mostra os reagentes de Diels-Alder como eles esto ligados, an-
tes de entrar no estado de transio para a formao de um produto no stio ati-
vo da enzima artifcial. A rea cinzenta representa um corte transversal atravs
da enzima. O dienflo est abaixo do plano da seo transversal, ao passo que
o dieno est acima do plano de seo transversal, quando as duas molculas
so ligados no stio ativo que est mostrado.
Desenhar a estrutura do produto da reao catalisada por enzima na caixa abai-
xo. Mostre em seu desenho a estereoqumica do produto e use R e R conforme
voc fez para uma parte a.
d. Considere as seguintes afrmaes sobre enzimas (artifcial ou natural). Para
cada situao, indique se essa afrmao Verdadeira ou Falsa (desenhe um
crculo em torno de Verdadeiro ou Falso).
i. Enzimas ligam-se mais fortemente ao estado de transio do que aos reagentes
ou produtos da reao.
Verdadeiro Falso
ii. Enzimas alteraram a constante de equilbrio da reao para favorecer o produto.
Verdadeiro Falso
iii. Catlise enzimtica sempre aumenta a entropia de ativao da reao em com-
parao com a reao no catalisada.
Verdadeiro Falso
e. Verses modifcadas das enzimas artifciais com diferentes atividades catalticas
foram preparadas (enzimas I, II, III e IV, mostradas na fgura abaixo). Dois res-
duos de aminocidos diferentes so mostrados nas diferentes enzimas abaixo.
Assuma que os grupos funcionais das enzimas mostradas esto localizados em
estreita proximidade com os fragmentos correspondentes dos reagentes quan-
do se forma o estado de transio no stio de enzima ativa.
Destas quatro enzimas, qual delas poderia causar maior aumento na velocida-
de da reao de Diels-Alder em comparao com a reao no catalisada?
Enzima #

Programa Nacional Olimpadas de Qumica <<
110



Leu
Phe
O
O
N H
C
O
O
N
O
NH
2
O
Gl n
H
O
Tyr
O
O
N H
C
O
O
N
O
Enzyme I Enzyme II
Leu
O
O
N H
C
O
O
N
O
H
O
Tyr
NH
2
O
Gl n
Phe
O
O
N H
C
O
O
N
O
Enzyme III Enzyme IV
f. A especifcidade do substrato das enzimas artifciais V e VI (ver abaixo) foi testa-
da usando os reagentes dienflos 1 - 6, mostrados abaixo.


f. A especificidade do substrato das enzimas artificiais V e VI (ver abaixo) foi testada
usando os reagentes dienfilos 1 - 6, mostrados abaixo.



O dienfilo # 1 reagiu mais rapidamente na reao catalisada pela enzima artificial V
(ver abaixo). No entanto, a enzima artificial VI catalisou a reao mais rapidamente com
outro dienfilo. Dos seis dienfilos mostrados acima, qual deles poderia reagir mais
rapidamente na reao de Diels-Alder catalisada pela enzima VI?




N
CH
3
H
3
C
O
N
CH
3
O
CH
3
N
O
CH
3
CH
3
N
O
CH
3
N
O
CH
3
N
O
1 2 3 4 5 6
CH
3
O OH
O dienflo # 1 reagiu mais rapidamente na reao catalisada pela enzima artif-
cial V (ver abaixo). No entanto, a enzima artifcial VI catalisou a reao mais rapi-
damente com outro dienflo. Dos seis dienflos mostrados acima, qual deles po-
deria reagir mais rapidamente na reao de Diels-Alder catalisada pela enzima VI?
44
th
IChO
>> Olimpada Brasileira de Qumica - 2012
111
Exame Terico


f. A especificidade do substrato das enzimas artificiais V e VI (ver abaixo) foi testada
usando os reagentes dienfilos 1 - 6, mostrados abaixo.



O dienfilo # 1 reagiu mais rapidamente na reao catalisada pela enzima artificial V
(ver abaixo). No entanto, a enzima artificial VI catalisou a reao mais rapidamente com
outro dienfilo. Dos seis dienfilos mostrados acima, qual deles poderia reagir mais
rapidamente na reao de Diels-Alder catalisada pela enzima VI?




N
CH
3
H
3
C
O
N
CH
3
O
CH
3
N
O
CH
3
CH
3
N
O
CH
3
N
O
CH
3
N
O
1 2 3 4 5 6
CH
3
O OH
PROBLEMA 8 8.3% do Total
a b-i b-ii b-iii b-iv b-v c-i c-ii c-iii Problema 8
8.3%
2 3 4 6 4 2 5 8 2 36

Hidrocarbonetos aromticos policclicos (PAHs) so poluentes atmosfricos, com-
ponentes de diodos orgnicos emissores de luz e componentes do meio interes-
telar. Este problema trata dos chamados PAHs lineares, ou seja, aqueles que tm
apenas um anel benznico de altura, variando no comprimento. Exemplos espec-
fcos so: benzeno, antraceno e pentaceno, cujas estruturas so mostradas abaixo.
Suas propriedades fsicas e qumicas dependem da extenso com que a nuvem de
eltrons est deslocalizada ao longo da molcula.

Programa Nacional Olimpadas de Qumica <<
112


PROBLEMA 8 8.3% do Total

a b-i b-ii b-iii b-iv b-v c-i c-ii c-iii Problema 8
8.3% 2 3 4 6 4 2 5 8 2 36


Hidrocarbonetos aromticos policclicos (PAHs) so poluentes atmosfricos, componentes de
diodos orgnicos emissores de luz e componentes do meio interestelar. Este problema trata
dos chamados PAHs lineares, ou seja, aqueles que tm apenas um anel benznico de altura,
variando no comprimento. Exemplos especficos so: benzeno, antraceno e pentaceno, cujas
estruturas so mostradas abaixo. Suas propriedades fsicas e qumicas dependem da extenso
com que a nuvem de eltrons est deslocalizada ao longo da molcula

x
y
d
d
a
d
p
benzene anthracene pentacene


a. A largura do anel benznico d = 240 pm. Use esta informao para estimar o
comprimento ao longo de eixo horizontal (x) do antraceno e do pentaceno, d
a
e d
p
,
respectivamente.



Para antraceno, d
a
=


Para pentaceno, d
p
=






b. Para simplificar, suponha, que os eltrons do benzeno podem ser modelados como se
estivessem confinados em um quadrado. Neste modelo, os eltrons conjugados dos PAHs
podem ser considerados como partculas livres em uma caixa bidimensional, retangular, no
plano x-y.

Para eltrons em uma caixa bidimensional ao longo dos eixos x e y, os estados de
energia quantizados desses eltrons so dados por:


a. A largura do anel benznico d = 240 pm. Use esta informao para estimar o
comprimento ao longo de eixo horizontal (x) do antraceno e do pentaceno, d
a
e
d
p
, respectivamente.
Para antraceno, d
a
=
Para pentaceno, d
p
=
b. Para simplifcar, suponha, que os eltrons do benzeno podem ser modelados
como se estivessem confnados em um quadrado. Neste modelo, os eltrons
conjugados dos PAHs podem ser considerados como partculas livres em uma
caixa bidimensional, retangular, no plano x-y.
Para eltrons em uma caixa bidimensional ao longo dos eixos x e y, os estados
de energia quantizados desses eltrons so dados por:


e
y
y
x
x
m
h
L
n
L
n
E
8
2
2
2
2
2
|
|
.
|

\
|
+ =
Nesta equao, n
x
e n
y
so os nmeros qunticos para o estado de energia e so
nmeros inteiros entre 1 e , h a constante de Planck, m
e
a massa do eltron e
L
x
e L
y
so as dimenses da caixa.

Para este problema, trate os eltrons t dos PAHs como partculas em uma caixa
bidimensional. Neste caso, os nmeros qunticos n
x
e n
y
so independentes.

i. Para este problema, suponha que a unidade de benzeno tem dimenses x e y, ambas de
comprimento d. Derive uma frmula geral para as energias quantizados dos PAHs
lineares como uma funo dos nmeros qunticos n
x
e n
y
, comprimento d, nmero de
anis fundidos w, e as constantes fundamentais h e m
e
.


ii. O diagrama de nvel de energia abaixo para o pentaceno mostra qualitativamente as
energias e os nmeros qunticos n
x
, n
y
, para todos os nveis ocupados por eltrons e para
o nvel de energia mais baixo desocupado, com os eltrons de spins opostos representados
como setas apontando para cima ou para baixo. Os nveis esto identificados com os
nmeros qunticos (n
x
; n
y
).

Pentaceno:
__ (3; 2)
(9; 1)
(2; 2)
(1; 2)
(8; 1)
(7; 1)
(6; 1)
(5; 1)
(4; 1)
(3; 1)
(2; 1)
(1; 1)
Nesta equao, n
x
e n
y
so os nmeros qunticos para o estado de energia e so
nmeros inteiros entre 1 e , h a constante de Planck, m
e
a massa do eltron
e L
x
e L
y
so as dimenses da caixa.
Para este problema, trate os eltrons p dos PAHs como partculas em uma caixa
bidimensional. Neste caso, os nmeros qunticos n
x
e n
y
so independentes.
i. Para este problema, suponha que a unidade de benzeno tem dimenses x e y,
ambas de comprimento d. Derive uma frmula geral para as energias quantiza-
dos dos PAHs lineares como uma funo dos nmeros qunticos n
x
e n
y
, compri-
mento d, nmero de anis fundidos w, e as constantes fundamentais h e m
e
.
ii. O diagrama de nvel de energia abaixo para o pentaceno mostra qualitativa-
mente as energias e os nmeros qunticos n
x
e n
y
para todos os nveis ocupados
por eltrons e para o nvel de energia mais baixo desocupado, com os el-
trons de spins opostos representados como setas apontando para cima ou para
baixo. Os nveis esto identifcados com os nmeros qunticos (n
x
; n
y
).
44
th
IChO
>> Olimpada Brasileira de Qumica - 2012
113
Exame Terico
Pentaceno:


e
y
y
x
x
m
h
L
n
L
n
E
8
2
2
2
2
2
|
|
.
|

\
|
+ =
Nesta equao, n
x
e n
y
so os nmeros qunticos para o estado de energia e so
nmeros inteiros entre 1 e , h a constante de Planck, m
e
a massa do eltron e
L
x
e L
y
so as dimenses da caixa.

Para este problema, trate os eltrons t dos PAHs como partculas em uma caixa
bidimensional. Neste caso, os nmeros qunticos n
x
e n
y
so independentes.

i. Para este problema, suponha que a unidade de benzeno tem dimenses x e y, ambas de
comprimento d. Derive uma frmula geral para as energias quantizados dos PAHs
lineares como uma funo dos nmeros qunticos n
x
e n
y
, comprimento d, nmero de
anis fundidos w, e as constantes fundamentais h e m
e
.


ii. O diagrama de nvel de energia abaixo para o pentaceno mostra qualitativamente as
energias e os nmeros qunticos n
x
, n
y
, para todos os nveis ocupados por eltrons e para
o nvel de energia mais baixo desocupado, com os eltrons de spins opostos representados
como setas apontando para cima ou para baixo. Os nveis esto identificados com os
nmeros qunticos (n
x
; n
y
).

Pentaceno:
__ (3; 2)
(9; 1)
(2; 2)
(1; 2)
(8; 1)
(7; 1)
(6; 1)
(5; 1)
(4; 1)
(3; 1)
(2; 1)
(1; 1)
O diagrama de nvel de energia para antraceno mostrado abaixo. Note que al-
guns nveis podem ter a mesma energia. Preencha o diagrama de nvel de energia
com o nmero correto de setas para cima e para baixo, para representar os eltrons
no antraceno. Alm disso, os espaos em branco entre parnteses dentro deste
diagrama so os nmeros qunticos nx, ny, que voc precisa determinar. Preencha
estes espaos em branco com os valores pertinentes de nx, ny para cada nvel de
energia ocupado e para o mais baixo desocupado.
Antraceno:
__ (__; __)
__(__; __) __ (__; __)
__ (__; __)
__ (__; __)
__ (__; __)
__ (__; __)
__ (__; __)
__ (__; __)
__ (__; __)
Programa Nacional Olimpadas de Qumica <<
114
iii. Utilize este modelo para criar um diagrama de nvel de energia para o benze-
no e preencha com eltrons, os nveis de energia pertinentes. Inclua todos os
nveis de energia at chegar ao nvel de energia mais baixo desocupado. Iden-
tifque cada nvel de energia no seu diagrama com os correspondentes n
x
, n
y
.
No suponha que o modelo de partculas-na-caixa-quadrada usado aqui d os
mesmos nveis de energia que os outros modelos.
iv. Muitas vezes, a reatividade de PAHs correlaciona inversamente com a diferena
de energia DE entre o nvel mais alto de energia ocupado por -eltrons e o
menor nvel de energia desocupado. Calcule a diferena de energia DE (em
Joules) entre os nveis ocupados mais altos e os nveis desocupados de mais
baixa energia para o benzeno, antraceno e pentaceno. Use os seus resultados
obtidos nos itens ii) e iii) para antraceno ou benzeno, respectivamente, ou use
(2, 2) para o mais elevado nvel de energia ocupado e (3, 2) para o mais baixo
nvel de energia desocupado para estas duas molculas (estes podem no ser
os verdadeiros valores).
DE para benzeno =
DE para pentaceno =
Classifque benzeno (B), antraceno (A) e pentaceno (P), em ordem de aumento
de reatividade, colocando as letras correspondentes da esquerda para a direita
na caixa abaixo.
Menos reativo -----------------------------------> Mais reativo
v. Os espectros de absoro eletrnica (absortividade molar x comprimento de
onda) para o benzeno (B), antraceno (A) e pentaceno (P) so mostrados abaixo.
Baseado em um entendimento qualitativo do modelo da partcula de caixa, in-
dique que molcula corresponde a cada espectro, escrevendo a letra apropria-
da no quadrado direita do respectivo espectro.
44
th
IChO
>> Olimpada Brasileira de Qumica - 2012
115



Classifique benzeno (B), antraceno (A) e pentaceno (P), em ordem de aumento de
reatividade, colocando as letras correspondentes da esquerda para a direita na
caixa abaixo.


Menos reativo -----------------------------------> Mais reactivo

v. Os espectros de absoro eletrnica (absortividade molar x comprimento de onda) para o
benzeno (B), antraceno (A) e pentaceno (P) so mostrados abaixo. Baseado em um
entendimento qualitativo do modelo da partcula de caixa, indique que molcula corresponde
a cada espectro, escrevendo a letra apropriada no quadrado direita do respectivo espectro.












c. Grafeno uma folha de tomos de carbono dispostos num padro bidimensional tal
como a estrutura do favo de mel. Ele pode ser considerado como um caso extremo de um
hidrocarboneto poliaromtico com comprimento essencialmente infinito nas duas


c. Grafeno uma folha de tomos de carbono dispostos num padro bidimen-
sional tal como a estrutura do favo de mel. Ele pode ser considerado como um
caso extremo de um hidrocarboneto poliaromtico com comprimento essen-
cialmente infnito nas duas dimenses. O Prmio Nobel de Fsica foi concedido
em 2010 para Andrei Geim e Konstantin Novoselov por seus experimentos ino-
vadores com o grafeno.
Considere uma folha de grafeno com dimenses planas de L
x
= 25 nm por Ly =
25 nm. Uma seo desta folha mostrada abaixo.


dimenses. O Prmio Nobel de Fsica foi concedido em 2010 para Andrei Geim e
Konstantin Novoselov por seus experimentos inovadores com o grafeno.
Considere uma folha de grafeno com dimenses planas de L
x
= 25 nm por L
y
= 25 nm.
Uma seo desta folha mostrada abaixo.



i. A rea de uma unidade hexagonal com 6 carbonos ~ 52.400 pm
2
. Calcule o nmero
de eltrons em uma folha (25 nm x 25 nm) de grafeno. Para este problema, voc pode
ignorar os eltrons que esto fora (ou seja, aqueles que esto fora dos hexgonos
completos no desenho).

ii. Podemos imaginar que os eltrons no grafeno so os eltrons livres, em uma caixa
bi-dimensional.

Em sistemas que contm um grande nmero de eltrons, no existe um s nvel de mais
alta energia simplesmente ocupado. Em vez disso, existem muitos estados de energias
bem prximas, acima dos quais, os demais nveis esto vazios. Estes estados de mais alta
energia ocupados determinam o chamado nvel de Fermi. O nvel de Fermi no grafeno
consiste em mltiplas combinaes de nmeros qunticos n
x
e n
y
. Determine a energia do
nvel de Fermi para um quadrado de grafeno de 25 nm 25 nm, relativo ao nvel
ocupado, de mais baixa energia. Este nvel ocupado de mais baixa energia no tem
energia zero, no entanto, ela insignificante, e pode ser assumida como sendo zero. Para
resolver este problema pode ser til representar os estados qunticos (nx, ny) como
pontos em uma grfico de 2-D (conforme mostrado abaixo) e considerar a ocupao dos
nveis de energia com os pares de eltrons. Para o nmero de eltrons utilize o resultado
do item (i) ou um valor de 1000 (este pode no ser o verdadeiro valor).
i. A rea de uma unidade hexagonal com 6 carbonos ~ 52.400 pm
2
. Calcule o
nmero de eltrons p em uma folha (25 nm x 25 nm) de grafeno. Para este pro-
blema, voc pode ignorar os eltrons que esto fora (ou seja, aqueles que esto
fora dos hexgonos completos no desenho).
Exame Terico
Programa Nacional Olimpadas de Qumica <<
116
ii. Podemos imaginar que os eltrons p no grafeno so os eltrons livres, em uma
caixa bi-dimensional.
Em sistemas que contm um grande nmero de eltrons, no existe um s nvel de
mais alta energia simplesmente ocupado. Em vez disso, existem muitos estados de
energias bem prximas, acima dos quais, os demais nveis esto vazios. Estes esta-
dos de mais alta energia ocupados determinam o chamado nvel de Fermi. O nvel
de Fermi no grafeno consiste em mltiplas combinaes de nmeros qunticos n
x
e n
y
. Determine a energia do nvel de Fermi para um quadrado de grafeno de 25
nm 25 nm, relativo ao nvel ocupado, de mais baixa energia. Este nvel ocupado
de mais baixa energia no tem energia zero, no entanto, ela insignifcante, e pode
ser assumida como sendo zero. Para resolver este problema pode ser til represen-
tar os estados qunticos (nx, ny) como pontos em uma grfco de 2-D (conforme
mostrado abaixo) e considerar a ocupao dos nveis de energia com os pares de
eltrons. Para o nmero de eltrons utilize o resultado do item (i) ou um valor de
1000 (este pode no ser o verdadeiro valor).




iii. A condutividade de material tipo grafeno se correlaciona inversamente com a
diferena de energia entre os mais baixos nveis desocupados e mais altos nveis de
energia ocupados. Use sua anlise e compreenso de eletrons em PHAs e grafeno para
prever se a condutividade de um quadrado de grafeno 25 nm 25 nm, a uma dada
temperatura, menor, igual ou maior do que a condutividade de um quadrado de grafeno
de 1m 1m (que o maior obtido at esta data). Circule a resposta correta:

iii. A condutividade de material tipo grafeno se correlaciona inversamente com
a diferena de energia entre os mais baixos nveis desocupados e mais altos
nveis de energia ocupados. Use sua anlise e compreenso de eletrons p em
PHAs e grafeno para prever se a condutividade de um quadrado de grafeno 25
nm 25 nm, a uma dada temperatura, menor, igual ou maior do que a con-
dutividade de um quadrado de grafeno de 1m 1m (que o maior obtido at
esta data). Circule a resposta correta:
44
th
IChO
>> Olimpada Brasileira de Qumica - 2012
117

44
th
International
Chemistry Olympiad
Exame prtico
21-30 julho de 2012 Washington, DC
Estados Unidos da Amrica
EXPERIMENTO 1
Voc tem 2 horas e 15 minutos para completar o experimento 1.
Produtos qumicos ou material de laboratrio sero fornecidos sem qualquer penalizao apenas
uma vez exceto no caso da acetona-d
6
. Cada pedido adicional resultar numa perda de 1 ponto dos
40 pontos destinados prova prtica.
REAGENTES E EQUIPAMENTO (EXPERIMENTO 1)
Reagentes (o rtulo real o que est indicado em negrito)
~2 M HCl,* soluo aquosa, frasco com 50 mL
~0.01 M KI
3
,* soluo aquosa, frasco com 10 mL, etiquetado I
2
.
Acetona, (CH
3
)
2
CO, M = 58,08 g mol
-1
, densidade = 0,791 g mL
-1
, frasco com 10,0 mL
Acetona-d
6
, (CD)
2
CO, M = 64,12 g mol
-1
, densidade = 0,872 g mL
-1
, ampola de 3,0 mL
* A concentrao exata a indicada na etiqueta do frasco.
Material no Kit #1
Um frasco de vidro com gua destilada
Quinze frascos porta-amostras de 20 mL com
rolha de rosca verde em Tefon
Dez pipetas de plstico de 1 mL graduadas em
divises de 0,25 mL (ver desenho direita).
Dez pipetas de plstico de 3 mL graduadas em
divises de 0.50 mL (ver desenho direita).
Um cronmetro digital (stopwatch)

1








1.00 mL
0.50 mL
0.75 mL
0.25 mL
3.0 mL
1.0 mL
2.0 mL
Exame Prtico
Programa Nacional Olimpadas de Qumica <<
118
Experimento 1 18% do total
a b c d e f g Experimento 1 18%
10 2 10 12 16 12 8 70

CINTICA, EFEITO ISOTPICO E MECANISMO DE IODAO DA ACETONA
O conhecimento dos mecanismos das reaes qumicas motivam avanos quer na
sntese quer em catlise. Uma poderosa ferramenta para investigar os mecanismos
da reao o estudo da sua cintica porque o modo como a velocidade da reao
varia em funo das condies da reao, depende diretamente do mecanismo
da reao. Uma segunda ferramenta importante o estudo da substituio isot-
pica em molculas. Apesar dos istopos apresentarem reatividade semelhante a
diferena na massa nuclear provoca pequenas alteraes na velocidade da reao.
Este experimento vai usar estes efeitos cinticos e isotpicos para conseguir ter
informao sobre o mecanismo de iodao da acetona em meio cido:


3

R
3
C CR
3
O
+ I
3
-
R
3
C C
R
2
O
I
+ R
+
+ 2I
-
R = H or D
A reao ocorre segundo a seguinte lei da velocidade
Velocidade = k[acetona]
m
[I
3
]
n
[H
+
]
p
onde a constante da velocidade k e os valores da ordem da reao m, n, e p devem
ser determinados. Voc tambm ir comparar a reatividade da acetona com a da
acetona-d
6
(deuterada), onde os seis tomos de hidrognio (
1
H) foram substitudos
por deutrio (
2
H, D), com o objetivo de determinar o efeito isotpico (k
H
/k
D
) da rea-
o. Atravs destes dados tire concluses sobre o mecanismo desta reao.
Leia atentamente todo o procedimento desta tarefa e planeje
o seu trabalho antes de comear.
44
th
IChO
>> Olimpada Brasileira de Qumica - 2012
119
Exame Prtico
PROCEDIMENTO
A velocidade de uma reao depende da temperatura. Registre aqui a tempe-
ratura do seu laboratrio (solicite o valor ao assistente de laboratorio):

oC
Instrues para o uso do cronmetro digital (stopwatch)
(1) Pressione a tecla [MODE] at aparecer COUNT UP no visor.
(2) Para iniciar a contagem do tempo, pressione a tecla [START/STOP].
(3) Para parar a contagem, pressione novamente a tecla [START/STOP].
(4) Para voltar ao zero no visor, pressione a tecla [CLEAR].
Procedimento Geral
Mea os volumes de cido clordrico, de gua destilada e da soluo de tri-iodeto
de potssio (frasco etiquetado como I
2
) que escolheu para um frasco reacional.
As concentraes iniciais dos reagentes na mistura reacional devem estar entre os
valores abaixo indicados (no necessita de explorar toda a gama de valores, mas os
seus valores no devem estar fora dos intervalos dados):
[H+]: Entre 0,2 e 1,0 M
[I3]: Entre 0,0005 e 0,002 M
[acetona]: Entre 0,5 e 1,5 M
Para iniciar a reao, adicione o volume escolhido de acetona ao frasco reacional que
contm os outros reagentes. Tampe rapidamente o frasco reacional, inicie a crono-
metragem, agite-o vigorosamente e coloque-o sobre uma superfcie branca. Registre
os volumes de reagentes que escolheu na tabela a. A partir do momento que iniciar
a reao no toque ou segure no frasco reacional no nivel do lquido. O progresso da
reao pode ser detectado visualmente pelo desaparecimento da cor amarela-acas-
tanhada do ion tri-iodeto. Registre o tempo necessrio para que a cor desaparea.
Quando a reao estiver completa coloque o frasco reacional de lado e deixe-o fecha-
do para que voc mesmo no fque exposto aos vapores de iodoacetona.
Repita as vezes que considerar necessrias usando diferentes concentraes de re-
agentes. Registre os valores das concentraes de reagentes que usou na tabela c.
Sugesto: altere uma concentrao de cada vez.
Programa Nacional Olimpadas de Qumica <<
120
Uma vez estudada a velocidade da reao da acetona, voc deve fazer o mesmo
para estudar a velocidade da reao da acetona-d
6
. Note que no caso da acetona-d
6

voc s dispe de 3,0 mL devido ao fato dos reagentes isotopicamente marcados
serem muito caros. Consequentemente, no caso de necessitar de mais acetona-d
6

voc ser penalizado com 1 ponto. Quando necessitar de usar este reagente, le-
vante a mo e um surpervisor do laboratrio ir abrir a sua ampola. As reaes
de compostos deuterados so geralmente mais lentas do que as dos compostos
substitudos apenas por hidrognio. Assim, quando trabalhar com (CD
3
)
2
CO deve
usar as condies reacionais que promovem a reao mais rpida.
Quando terminar o trabalho:
a) Esvazie a garrafa de gua e coloque-a juntamente com todo o material no
utilizado na caixa etiquetada Kit #1;
b) Coloque as pipetas e os frascos reacionais usados nos recipientes apropria-
dos que se encontram nas capelas;
c) Utilize o recipiente etiquetado Broken Glass Disposal para colocar os res-
tos de vidro da ampola.
Voc deve arrumar e limpar a sua rea de trabalho depois de ter sido dado a ordem
de STOP.
a. Registre na tabela seguinte os resultados da experincia com a acetona,
(CH
3
)
2
CO. No preciso efetuar o nmero de experincias necessrias para preen-
cher totalmente a tabela.
Exp. # Volume da
soluo
HCl, mL
Volume
H
2
O, mL
Volume da
soluo I
3

,
mL
Volume
(CH
3
)
2
CO,
mL
Tempo de de-
saparecimento
do I
3

, s
1
2
3
4
5
6
7
8
Exp- experincia
44
th
IChO
>> Olimpada Brasileira de Qumica - 2012
121
Exame Prtico
b. Registre na tabela seguinte os resultados da experincia com a acetona-d6,
(CD
3
)
2
CO. No preciso efetuar o nmero de experincias necessrias para pre-
encher totalmente a tabela.
Exp. # Volume da
soluo
HCl, mL
Volume
H
2
O,
mL
Volume da
soluo I
3

, mL
Volume
(CD
3
)
2
CO,
mL
Tempo de
desaparecimento
do I
3

, s
1d
2d
3d
4d

c. Utilize a tabela seguinte para registrar as concentraes e as velocidades cal-
culadas para as reaes que estudou. Assuma que o volume de cada mistura
reacional igual soma dos volumes dos seus constituintes. No necessrio
usar todas as experincias que realizou para calcular o valor de k (partes e
e f) mas voc deve indicar que experincia ou experincias usou nos seus
clculos. Para tal, assinale na coluna da direita e na caixa respectiva a(s)
experincia(s) que utilizou para efetuar os clculos.
(CH
3
)
2
CO:
Exp # Concentra-
o Inicial
[H
+
], M
Concentra-
o Inicial
[I
3

], M
Concentra-
o Inicial
[(CH
3
)
2
CO],
M
Velocidade
de desapare-
cimento do
I
3
, M s
-1

Experincia
utilizada nos
clculos de k
H
?
Sim No
1
2
3
4
5
6
7
8
(CD
3
)
2
CO:
Programa Nacional Olimpadas de Qumica <<
122
Exp # Concentra-
o Inicial
[H
+
], M
Concentra-
o Inicial
[I
3

], M
Concentra-
o Inicial
[(CD
3
)
2
CO],
M
Velocidade
de desapare-
cimento do
I
3
, M s
-1

Experincia
utilizada nos
clculos de k
D
?
Sim No
1d
2d
3d
4d
d. Indique os valores da ordem de reao para a acetona, tri-iodito e ion hidrognio.

8













m = n = p =
e. Calcule a constante de velocidade k
H
da reao da acetona, (CH
3
)
2
CO, e indique
as suas unidades.
kH =
f. Calcule a constante de velocidade k
D
da reao da acetona-d
6
, (CD
3
)
2
CO, e calcu-
le o valor da razo de k
H
/k
D
(o efeito isotpico da reao).
g. Atravs dos dados da cintica e do efeito isotpico podem ser tiradas conclu-
ses sobre o mecanismo desta reao. Abaixo proposto um mecanismo pos-
svel para a iodao da acetona. Uma das reaes o passo determinante da
velocidade da reao (R.D.S).
44
th
IChO
>> Olimpada Brasileira de Qumica - 2012
123
Exame Prtico
No quadro abaixo, na coluna R.D.S consistente com a lei da velocidade direi-
ta de cada etapa do mecanismo, marque com ( ) se a lei de velocidade obtida
experimentalmente (parte d) consistente com essa etapa e marque com um
(X) se a lei de velocidade obtida experimentalmente no consistente com a
respectiva etapa . Na coluna R.D.S consistente com o efeito isotpico marque
com ( ) se as medidas do efeito isotpico obtidas experimentalmente (parte
f) so consistentes com essa etapa e marque com um (X) se o efeito isotpico
obtido experimentalmente no consistente com a respectiva etapa

10


R.D.S. consistente
com a lei da
velocidade?
R.D.S consistente
com o efeito
isotpico?

10



10



10




Programa Nacional Olimpadas de Qumica <<
124

Experimento 2
INSTRUES (EXPERIMENTO 2)
Voc tem 2 horas e 45 minutos para completar o Experimento 2. Quando planejar seu trabalho,
por favor, considere que uma das etapas requer 30 minutos.
Os seguintes itens devero ser deixados em sua bancada:
O exame / folhas de resposta (este caderno)
Uma placa de TLC em um saco plstico com zipper e com seu cdigo de estudante
O vidrinho (vial) rotulado Product
Voc deve seguir as regras de segurana dadas no regulamento da IChO. Enquanto voc estiver
no laboratrio use os culos de segurana ou seus prprios culos, se tiver sido aprovado. Use a pra
fornecida. Voc deve usar luvas quando manusear os reagentes.
Reagentes e materiais de laboratrio somente sero repostos, sem penalidade, a primeira vez.
Cada incidente posterior ser punido com a perda 1 ponto dos 40 pontos desse exame
REAGENTES E EQUIPAMENTOS
Reagentes e materiais (o rtulo real o que est em negrito)
(salen)H
2
,
a
~1.0 g
b
em um frasquinho (vial)
Mn(OOCCH
3
)
2
4H
2
O, ~1.9 g
b
em um frasquinho
Lithium chloride solution, LiCl, soluo 1M em etanol, 12 mL em um frasco
Ethanol, 70 mL em um frasco
Acetona, (CH
3
)
2
CO, 100 mL em um frasco
(salen*)MnCl
x
,
c
~32 mL de uma soluo ~3.5 mg/mL
b
em um frasco
KI
3
, soluo aquosa ~0.010 M,
b
50 mL em um frasco, rotulado I
2
.
Ascorbic Acid, soluo aquosa ~0.030 M,
b
20 mL em um frasco
1% Starch, soluo aquosa, 2 mL em um frasco
TLC plate uma placa de 5 cm 10 cm de slica gel em um saco plastico com zipper
a
(salen)H
2
:

12

OH
N N
OH
O
N N
O
Cl
x
Mn
R R
44
th
IChO
>> Olimpada Brasileira de Qumica - 2012
125
Exame Prtico
b
O valor exato est indicado no rtulo.
c
(salen*)MnClx (os grupos R so iguais e podem ser ou H, ou COOH ou SO
3
H):

12

OH
N N
OH
O
N N
O
Cl
x
Mn
R R
Equipmentos
Uso Comum: Balana
Dois suportes com garras localizados na capela, marcados com seu cdigo
Um placa de aquecimento com agitao
Uma rgua de 300 mm
Um lpis
Kit #2:
Dois Erlenmeyers de 250 mL (um para sntese e outro cristalizao)
Uma proveta, 50 mL
Uma barra magntica de 20 mm
Um funil de Hirsch
Papel fltro circular para o funil de Hirsch e para a cuba de TLC
Um Kitasato de 125 mL para fltrao vcuo
Um adapatador de borracha para o kitasato
Um recipiente de plstico para banho de gelo, de 0,5 L
Um anel de vidro
Duas pipetas de plstico de 1 mL (veja desenho direita)
Uma esptula de plstico
Um frasquinho vazio de 4 mL rotulado Product para o produto da reao
Kit #3:
Trs frasquinhos pequenos vazios (para as solues de TLC)
Dez tubos capilares (100 mm) para aplicao na placa de TLC
Um vidro de relgio (para a cuba de TLC)
Um bquer de 250 mL para usar como cuba de TLC
Kit #4:

Uma bureta 25 mL montada para ser usada
Um pequeno funil de plstico

13

























Programa Nacional Olimpadas de Qumica <<
126
Quarto Erlenmeyers de 125 mL
Um pra para pipeta
Uma pipeta volumtrica de 10 mL
Uma pipeta volumtrica de 5 mL
Experimento 2 22% do Total
Sntese de um complexo de Mangans Salen e Determinao da Frmula do
Produto
A B-i B-ii C-i C-ii Experimento 2 22%
10 15 4 4 2 35

Os complexos de metais de transio do bloco 3d derivados do ligante
bis(salicilideno)etilenodiamina (salen) tm mostrado ser efcientes catalisadores
de vrias reaes redox em sntese orgnica.

15

R OH
N N
HO R

A habilidade do ligante salen em estabilizar altos estados de oxidao de elemen-
tos do bloco 3d importante em qumica. Em particular, compostos de mangans
com estados de oxidao de +2 a +5 podem ser gerados, dependendo das condi-
es de reao quando o complexo mangans e salen preparado. Neste experi-
mento voc ir preparar um complexo de mangans e salen fazendo a reao de
(salen)H
2
com acetato de Mn(II) em etanol, ao ar, na presena de cloreto de ltio.
Sob estas condies, voc obter um complexo de frmula (salen)MnCl
x
, onde x =
0, 1, 2, ou 3.
Voc necessitar: i) determinar a massa do produto, ii) caracterizar a pu-
reza do material preparado, usando cromatografa em camada delgada (TLC), e iii)
determinar o estado de oxidao do metal no complexo usando titulao redox
iodomtrica. Para a titulao redox, voc receber uma soluo de um anlogo
de seu composto, previamente preparado, (salen*)MnCl
x
, onde o mangans tem o
mesmo estados de oxidao que em seu produto o substituinte R do anel benzni-
co ou H, ou COOH, ou SO
3
H.
44
th
IChO
>> Olimpada Brasileira de Qumica - 2012
127
Exame Prtico
Por favor, leia a descrio total desse experimento e planeje o seu trabalho antes
de comear. Alguns procedimentos tm que ser realizados em paralelo para que
voc possa completar o experimento no tempo certo.

Procedimento:
A. Sintese do (salen)MnClx

17

OH
N N
HO
+ Mn(OOCCH
3
)
2
+ O
2
+ LiCl
O
N N
O
Cl
x
Mn
(salen)MnCl
x








1) Coloque 2-3 cristais do (salen)H
2
, de lado, em um do frasquinho, para ser usado
mais tarde, no experimento de TLC.
2) Transfra a amostra de ~1.0-g de (salen)H
2
pr-pesada para um Erlenmeyer de
250 mL contendo uma barrinha magntica. Junte o reagente com 35 mL de
etanol absoluto.
3) Coloque o frasco na placa de aquecimento com agitao. Aquea com agitao
constante at a dissoluo do slido (normalmente, a dissoluo se completa
quando o etanol entra em ebulio). Ento, diminua a temperatura para manter
a mistura em uma temperatura prxima, porm, abaixo do seu ponto de ebu-
lio. No ferva a mistura, para que o gargalo do Erlenmeyer no fque quente.
Se o recipiente estiver quente demais para segurar com as mos, use uma toa-
lha de papel dobrado.
4) Retire o frasco da placa de aquecimento e adicione ao seu contedo a amostra
de ~1.9-g de Mn(OAc)
2
4H
2
O, pr-pesada. Uma colorao marron-escura apare-
cer. Retorne, imediatamente, o frasco para a placa de aquecimento; continue
aquecendo e agitando por 15 min. No ferva a mistura, para que o gargalo do
Erlenmeyer no fque quente.
5) Retire o balo da placa de aquecimento e adicione ao seu contedo a soluo
fornecida de LiCl 1M em etanol (12 mL, excesso). Retorne o balo para a placa
de aquecimento; continue o aquecimento e a agitao durante 10 min. No
ferva a mistura para que o gargalo Erlenmeyer no fque quente.
6) Aps este tempo retire o balo da placa de aquecimento, e coloque-o num ba-
nho de gelo para a cristalizao durante 30 min. A cada 5 min risque suave-
mente as paredes internas abaixo do nvel de lquido, com o basto de vidro
Programa Nacional Olimpadas de Qumica <<
128
a fm de acelerar a cristalizao de (salen) MnClx. Os primeiros cristais podem
aparecer imediatamente aps o resfriamento ou aps um perodo de apenas
10-15 minutos.
7) Use a linha de vcuo localizada dentro da capela (a vlvula correspondente
identifcada como vcuo) e fltre, vcuo, o slido cristalino formado usando
o funil de Hirsch pequeno e o kitasato. Use uma pipeta de transferncia para
lavar o slido com algumas gotas de acetona sem desligar o kitasato da linha
de vcuo, e deixe-o no fltro (com suco) durante 10-15 min para secar ao ar.
8) Transfra o produto slido para o frasco pr-pesado, rotulado como Produto e
em seguida, determine e registre sua massa, m
p
, no espao apropriado a seguir.
Escreva tambm as massas dos seguintes reagentes utilizados na sntese: (sa-
len)H
2
, m
S
, e Mn(OOCCH
3
)
2
4H
2
O, m
Mn..

9) Coloque o frasco rotulado com o produto dentro de um saco plstico com zper
Massa do frasquinho do produto, vazio: _______________ g
Massa do frasquinho que contm o produto seco: _______________ g
Massa do produto, m
p
: _______________ g
Massa de (salen)H
2
contido no frasquinho (copie do rtulo), m
S
: _______________ g
Massa de Mn(OOCCH
3
)
2
4H
2
O contido no frasquinho (copie do rtulo), m
Mn
: _______________ g
B. Anlise volumtrica da amostra do (salen*) MnClx fornecido

19










Uso da pra
1) Coloque a pra na pipeta
2) Aperte frmemente a pra de borracha
3) Aperte o boto indicado (seta para cima) para sugar para dentro da pipeta.
4) Aperte o boto indicado (seta para baixo) para escoar alguma soluo da pipeta.
Nota: As pipetas e buretas esto prontas para uso e no necessitam ser condi-
conadas.
44
th
IChO
>> Olimpada Brasileira de Qumica - 2012
129
Exame Prtico
1) Pipetar 10,00 mL da soluo (salen*)MnClx fornecida em um Erlenmeyer de 125
mL utilizando a pipeta volumtrica.
2) Adicionar 5,00 ml de soluo de cido ascrbico a esta soluo e misturar bem.
Deixar a soluo em repouso durante 3-4 minutos.
3) Para evitar a oxidao do cido ascrbico com O
2
, no demore, titule a soluo
imediatamente com a soluo de KI3 usando 5 gotas de uma soluo de amido
1% como indicador. As cores azul ou azul-esverdeado devem se manter duran-
te, pelo menos, 30 seg.
4) Se o tempo permitir, repetir a titulao 1 ou 2 vezes para melhorar a preciso da
sua determinao.
5) Coloque os resultados da sua(s) titulao(es) na tabela
# Volume inicial lido na
bureta com soluo de
KI
3
, mL
Volume fnal lido na
bureta com soluo
de KI
3
, mL
Volume consumido
da soluo
de KI
3
, mL
1
2
3

i. Indique o volume (selecionado ou mdia), em mL, da soluo de KI3 con-
sumida que voc ir usar para os clculos da massa molar de (salen*)MnClx :
Volume da soluo de KI
3
usada nos clculos: _____________ mL
Concentrao do (salen*)MnCl
x
(lido no rtulo do frasco): __________ mg/mL
Concentrao do cido ascrbico (lido no rtulo do frasco):______________ M
ii. A partir de dados de sua titulao e, de acordo com a tabela abaixo, deduzir o
valor de x, estado de oxidao do mangans e a identidade do substituinte no
ligante salen (R = H, COOH, SO
3
H). Aponte-os no modelo abaixo:

21

O
N N
O
Clx
Mn
Programa Nacional Olimpadas de Qumica <<
130
R x (Massa terica molar)/x, g/mol
H 1 357
H 2 196
H 3 143
COOH 1 445
COOH 2 240
COOH 3 172
SO
3
H 1 517
SO
3
H 2 276
SO
3
H 3 196

C. Caracterizao de TLC do (salen)MnClx
1) Dissolva alguns cristais do (salen)MnCl
x
que voc preparou com algumas gotas
de etanol absoluto, use um pequeno frasco e uma pipeta de plstico para trans-
ferir o etanol.
2) Dissolva alguns cristais de (salen)H
2
em gotas de etanol absoluto, use outro
frasco pequeno.
3) Se necessrio, use tesouras (pedir ao assistente de laboratrio) para cortar a
placa de TLC em uma altura adequada para a cuba de TLC.
4) Dobre ou corte um crculo grande de papel fltro e coloque-o no bquer de
modo que fque quase na altura do bquer cheio. Isto necessrio para saturar
a cuba (bquer) com vapor de etanol. Adicionar etanol no bquer at molhar
o papel fltro, e cobrir o fundo com uma camada de 3-4 mm de espessura de
solvente. Feche o bquer com o vidro de relgio.
5) Marque a aplicao.
6) Com ambas as solues, use os tubos capilares para aplicar as manchas na placa
de TLC.
7) Durante 10-15 min, elua uma placa TLC no bquer coberto com um vidro de
relgio.
8) Marque com um lpis a frente do solvente, bem como os pontos coloridos na
placa de TLC.
9) Seque a placa de TLC ao ar e coloque de volta em um saco com zipper.
10) Calcular o R
f
para ambos (salen)H
2
e (salen)MnClx.
44
th
IChO
>> Olimpada Brasileira de Qumica - 2012
131
Exame Prtico
i. Desenhe a placa de TLC em sua folha de respostas

ii. Determine e anote os valores de R
f
para (salen)H
2
e (salen)H
2
e (salen)MnClx
Rf, (salen)H
2
: _____

Rf, (salen)MnClx: _____
Quando voc terminar o trabalho:
a) Colocar resduos lquidos em um recipiente marcado Liquid Waste.
b) Colocar os frascos utilizados em um depsito chamado Broken Glass Disposal.
c) Colocar a vidraria utilizada de volta para caixas apropriadas rotuladas como Kit
#2, Kit #3 and Kit #4.
Programa Nacional Olimpadas de Qumica <<
132
17
a
Olimpada Iberoamericana
de Qumica

22 a 30 de setembro,
em Santa F, Argentina
INSTRUES
Voc tem 5 horas para completar todas as tarefas e registrar os resultados nas caixas de resposta.
Voc deve parar o trabalho imediatamente aps receber o sinal de parada. Uma interrupo de atra-
so de 3 minutos produzir uma pena de 10 (dez) pontos sua pontuao total.
CONSTANTES FSICAS, FRMULAS E EQUAES
Nmero de Avogadro, N = 6,0221 x 10
23
mo
l1

Constante de Boltzmann, k
B
= 1,3807 x 10
23
JK
1

Constante de Universal dos gases, R = 8,3145 JK
1
mol
1
= 0,08205 atmLK
1
mol
1

Velocidade da luz, c = 2,9979 x 10
8
ms
1

Constante de Planck, h = 6,6261 x 10
34
Js
Carga do eltron, e = 1,602 x 10
-19
C
Permissividade no vcuo, e
0
= 8,8541 x 10
-12
C2J
-1
m
-1
Massa do eltron, m
e
= 9,10938215 x 10
31
kg
Presso Padro, P = 1 bar = 10
5
Pa
Presso atmosfrica, P
atm
= 1,01325 x 10
5
Pa = 760 mmHg = 760 Torr
Zero da escala Celsius, 273,15 K
1 nanmetro (nm) = 10
9
m
1 picmetro (pm) = 10
12
m
p = 3,141592
Programa Olimpada Argentina de Qumica
Facultad de Ciencias Exactas e Naturales, UBA
Facultad de Ingeniera Qumica, UNL
Ministerio de Educacin de la Nacin Argentina

Instrues

- Voc tem 5 horas para completar todas as tarefas e registrar os resultados nas caixas de resposta.
Voc deve parar o trabalho imediatamente aps receber o sinal de parada. Uma interrupo de atraso
de 3 minutos produzir uma pena de 10 (dez) pontos sua pontuao total.

Constantes Fsicas, Frmulas e Equaes

Nmero de Avogadro, N = 6,0221 10
23
mol
1

Constante de Boltzmann, k
B
= 1,3807 10
23
JK
1

Constante de Universal dos gases, R = 8,3145 JK
1
mol
1
= 0,08205 atmLK
1
mol
1

Velocidade da luz, c = 2,9979 10
8
ms
1

Constante de Planck, h = 6,6261 10
34
Js
Carga do eltron, e = 1,60210
-19
C
Permissividade no vcuo, c
0
= 8,854110
-12
C
2
J
-1
m
-1

Massa do eltron, m
e
= 9,10938215 10
31
kg
Presso Padro, P = 1 bar = 10
5
Pa
Presso atmosfrica, P
atm
= 1,01325 10
5
Pa = 760 mmHg = 760 Torr
Zero da escala Celsius, 273,15 K
1 nanmetro (nm) = 10
9
m
1 picmetro (pm) = 10
12
m
t = 3,141592

c l A . . c = T A log =
0
I
I
T =
T T . 100 % = S T H G A A = A W Q U + = A
) (PV U H A + A = A
K RT G ln
0
= A
nRT pV =
3
a V
cubo
= P y p
i i
= 1 ...
2 1
= + + +
n
y y y
17
a
OIAQ
>> Olimpada Brasileira de Qumica - 2012
133
Exame Terico
1
PROBLEMA 1 (9%)
A desidratao de lcoois realizada por catlise cida. Em 1939,
E. Price descreveu a desidratao do 2-metil-1-ciclo-hexanol, uma
experincia que vem sendo realizada nos laboratrios de ensino
de qumica orgnica ao longo de dcadas.
Os ciclohexanos existem em soluo em um equilbrio conformacional tempe-
ratura ambiente (25 C). Alm disso, os compostos 1,2-di-substitudos, tais como o
2-metil-1-ciclo-hexanol, existem como estereoismeros. Por exemplo, os estereoi-
smeros 1 e 2.

PROBLEMA 1 (9%)

A desidratao de lcoois realizada por catlise cida. Em 1939, E. Price descreveu a desidratao do 2-
metil-1-ciclo-hexanol, uma experincia que vem sendo realizada nos laboratrios de ensino de qumica
orgnica ao longo de dcadas.
Os ciclohexanos existem em soluo em um equilbrio conformacional temperatura ambiente (25 C). Alm
disso, os compostos 1,2-di-substitudos, tais como o 2-metil-1-ciclo-hexanol, existem como estereoismeros.
Por exemplo, os estereoismeros 1 e 2.
OH
CH
3
Estereoismero 1
OH
CH
3
Estereoismero 2

(a) Desenhe a estrutura dos outros estereoismeros de 1 e 2.


(b) Escreva os equilbrios conformacionais dos estereoismeros 1 e 2.


(c) Escreva os nomes, segundo regras da IUPAC, dos estereoismeros do 2-metil-1-ciclohexanol.


Agora racionalizemos o mecanismo da reao de desidratao do 2-metil-1-ciclo-hexanol e estudemos a
sua regioqumica.
O diagrama a seguir mostra a reao estudada por E. Price, em 1939, para quaisquer dos estereoismeros:

Estereoismero 1
D
I-1
Base
H
3
PO
4
A (C
7
H
12
) + B (C
7
H
12
)
I-2
Base
A + C (C
7
H
12
)
85 % 15%
3%
97%


(d) Desenhe as estruturas dos intermedirios I-1 e I-2.



(e) Desenhe as estruturas dos produtos A, B, C e D.

(f) Qual a relao de estereoisomeria que existe entre os produtos A e B? Marque com uma cruz (X) as
respostas que consideras corretas.

(a) Desenhe a estrutura dos outros estereoismeros de 1 e 2.
(b) Escreva os equilbrios conformacionais dos estereoismeros 1 e 2.
(c) Escreva os nomes, segundo regras da IUPAC, dos estereoismeros do 2-metil-1-
-ciclohexanol.
Agora racionalizemos o mecanismo da reao de desidratao do 2-metil-1-ciclo-
-hexanol e estudemos a sua regioqumica.
O diagrama a seguir mostra a reao estudada por E. Price, em 1939, para quais-
quer dos estereoismeros:

PROBLEMA 1 (9%)

A desidratao de lcoois realizada por catlise cida. Em 1939, E. Price descreveu a desidratao do 2-
metil-1-ciclo-hexanol, uma experincia que vem sendo realizada nos laboratrios de ensino de qumica
orgnica ao longo de dcadas.
Os ciclohexanos existem em soluo em um equilbrio conformacional temperatura ambiente (25 C). Alm
disso, os compostos 1,2-di-substitudos, tais como o 2-metil-1-ciclo-hexanol, existem como estereoismeros.
Por exemplo, os estereoismeros 1 e 2.
OH
CH
3
Estereoismero 1
OH
CH
3
Estereoismero 2

(a) Desenhe a estrutura dos outros estereoismeros de 1 e 2.


(b) Escreva os equilbrios conformacionais dos estereoismeros 1 e 2.


(c) Escreva os nomes, segundo regras da IUPAC, dos estereoismeros do 2-metil-1-ciclohexanol.


Agora racionalizemos o mecanismo da reao de desidratao do 2-metil-1-ciclo-hexanol e estudemos a
sua regioqumica.
O diagrama a seguir mostra a reao estudada por E. Price, em 1939, para quaisquer dos estereoismeros:

Estereoismero 1
D
I-1
Base
H
3
PO
4
A (C
7
H
12
) + B (C
7
H
12
)
I-2
Base
A + C (C
7
H
12
)
85 % 15%
3%
97%


(d) Desenhe as estruturas dos intermedirios I-1 e I-2.



(e) Desenhe as estruturas dos produtos A, B, C e D.

(f) Qual a relao de estereoisomeria que existe entre os produtos A e B? Marque com uma cruz (X) as
respostas que consideras corretas.

(d) Desenhe as estruturas dos intermedirios I-1 e I-2.
(e) Desenhe as estruturas dos produtos A, B, C e D.
(f) Qual a relao de estereoisomeria que existe entre os produtos A e B? Marque
com uma cruz (X) as respostas que consideras corretas.
(g) Qual o mecanismo da reao de desidratao do 2-metil-1-ciclohexanol? Mar-
que com uma cruz (X) a resposta que voc considera correta.
Programa Nacional Olimpadas de Qumica <<
134

(g) Qual o mecanismo da reao de desidratao do 2-metil-1-ciclohexanol? Marque com uma cruz (X)
a resposta que voc considera correta.

(1) Mecanismo E
2
(2) Mecanismo SN
2
(3) Mecanismo E
1
(4) Mecanismo SN
1


Para transformar um grupo lcool em um ster, reagimos o mesmo com um halogeneto de acila como se
mostra a seguir:

OH +
Cl
O
Piridina
0C
O
O
+ HCl

Um reagente utilizado para converter o grupo lcool em um bom grupo de sada o cloreto de
metansulfonila, que d como como produto um sulfonato no lugar do ster.
A reao ocorre conforme mostramos a seguir:
OH
H
3
C
S
O
O
Cl
Piridina
0C
NH
4
Cl s. s.
E



(h) Desenhe a estrutura do confrmero mais estvel do composto E.



A reao do composto E com uma base forte e calor d um nico produto F; o mecanismo concertado e
verifica experimentalmente que a velocidade proporcional concentrao do composto E e concentrao
da base.
E F
t-BuOK
t-BuOH
72C


(i) Desenhe a estrutura do composto F.




(g) Qual o mecanismo da reao de desidratao do 2-metil-1-ciclohexanol? Mar-
que com uma cruz (X) a resposta que voc considera correta.

(g) Qual o mecanismo da reao de desidratao do 2-metil-1-ciclohexanol? Marque com uma cruz (X)
a resposta que voc considera correta.

(1) Mecanismo E
2
(2) Mecanismo SN
2
(3) Mecanismo E
1
(4) Mecanismo SN
1


Para transformar um grupo lcool em um ster, reagimos o mesmo com um halogeneto de acila como se
mostra a seguir:

OH +
Cl
O
Piridina
0C
O
O
+ HCl

Um reagente utilizado para converter o grupo lcool em um bom grupo de sada o cloreto de
metansulfonila, que d como como produto um sulfonato no lugar do ster.
A reao ocorre conforme mostramos a seguir:
OH
H
3
C
S
O
O
Cl
Piridina
0C
NH
4
Cl s. s.
E



(h) Desenhe a estrutura do confrmero mais estvel do composto E.



A reao do composto E com uma base forte e calor d um nico produto F; o mecanismo concertado e
verifica experimentalmente que a velocidade proporcional concentrao do composto E e concentrao
da base.
E F
t-BuOK
t-BuOH
72C


(i) Desenhe a estrutura do composto F.




Para transformar um grupo lcool em um ster, reagimos o mesmo com um halo-
geneto de acila como se mostra a seguir:

(g) Qual o mecanismo da reao de desidratao do 2-metil-1-ciclohexanol? Marque com uma cruz (X)
a resposta que voc considera correta.

(1) Mecanismo E
2
(2) Mecanismo SN
2
(3) Mecanismo E
1
(4) Mecanismo SN
1


Para transformar um grupo lcool em um ster, reagimos o mesmo com um halogeneto de acila como se
mostra a seguir:

OH +
Cl
O
Piridina
0C
O
O
+ HCl

Um reagente utilizado para converter o grupo lcool em um bom grupo de sada o cloreto de
metansulfonila, que d como como produto um sulfonato no lugar do ster.
A reao ocorre conforme mostramos a seguir:
OH
H
3
C
S
O
O
Cl
Piridina
0C
NH
4
Cl s. s.
E



(h) Desenhe a estrutura do confrmero mais estvel do composto E.



A reao do composto E com uma base forte e calor d um nico produto F; o mecanismo concertado e
verifica experimentalmente que a velocidade proporcional concentrao do composto E e concentrao
da base.
E F
t-BuOK
t-BuOH
72C


(i) Desenhe a estrutura do composto F.




Um reagente utilizado para converter o grupo lcool em um bom grupo de sada
o cloreto de metansulfonila, que d como como produto um sulfonato no lugar
do ster.

(g) Qual o mecanismo da reao de desidratao do 2-metil-1-ciclohexanol? Marque com uma cruz (X)
a resposta que voc considera correta.

(1) Mecanismo E
2
(2) Mecanismo SN
2
(3) Mecanismo E
1
(4) Mecanismo SN
1


Para transformar um grupo lcool em um ster, reagimos o mesmo com um halogeneto de acila como se
mostra a seguir:

OH +
Cl
O
Piridina
0C
O
O
+ HCl

Um reagente utilizado para converter o grupo lcool em um bom grupo de sada o cloreto de
metansulfonila, que d como como produto um sulfonato no lugar do ster.
A reao ocorre conforme mostramos a seguir:
OH
H
3
C
S
O
O
Cl
Piridina
0C
NH
4
Cl s. s.
E



(h) Desenhe a estrutura do confrmero mais estvel do composto E.



A reao do composto E com uma base forte e calor d um nico produto F; o mecanismo concertado e
verifica experimentalmente que a velocidade proporcional concentrao do composto E e concentrao
da base.
E F
t-BuOK
t-BuOH
72C


(i) Desenhe a estrutura do composto F.




A reao ocorre conforme mostramos a seguir:
(h) Desenhe a estrutura do confrmero mais estvel do composto E.
A reao do composto E com uma base forte e calor d um nico produto F; o
mecanismo concertado e verifca experimentalmente que a velocidade propor-
cional concentrao do composto E e concentrao da base.

(g) Qual o mecanismo da reao de desidratao do 2-metil-1-ciclohexanol? Marque com uma cruz (X)
a resposta que voc considera correta.

(1) Mecanismo E
2
(2) Mecanismo SN
2
(3) Mecanismo E
1
(4) Mecanismo SN
1


Para transformar um grupo lcool em um ster, reagimos o mesmo com um halogeneto de acila como se
mostra a seguir:

OH +
Cl
O
Piridina
0C
O
O
+ HCl

Um reagente utilizado para converter o grupo lcool em um bom grupo de sada o cloreto de
metansulfonila, que d como como produto um sulfonato no lugar do ster.
A reao ocorre conforme mostramos a seguir:
OH
H
3
C
S
O
O
Cl
Piridina
0C
NH
4
Cl s. s.
E



(h) Desenhe a estrutura do confrmero mais estvel do composto E.



A reao do composto E com uma base forte e calor d um nico produto F; o mecanismo concertado e
verifica experimentalmente que a velocidade proporcional concentrao do composto E e concentrao
da base.
E F
t-BuOK
t-BuOH
72C


(i) Desenhe a estrutura do composto F.




(i) Desenhe a estrutura do composto F.
17
a
OIAQ
>> Olimpada Brasileira de Qumica - 2012
135
Exame Terico
2
PROBLEMA 2 (7%)
A atividade neurotxica de (+)-muscarina tornou-se assunto de
intensa pesquisa direcionada para a sntese de estereoismeros
que permitiram estudar suas reaes com os receptores do siste-
ma nervoso. A estrutura desta substncia mostrada abaixo:

PROBLEMA 2 (7%)

A atividade neurotxica de (+)-muscarina tornou-se assunto de intensa pesquisa direcionada para a sntese
de estereoismeros que permitiram estudar suas reaes com os receptores do sistema nervoso. A estrutura
desta substncia mostrada abaixo:

O
H
3
C CH
2
N(CH
3
)
3
HO
(+)-Muscarina


No esquema seguinte mostra-se a sequncia sinttica para a obteno do precursor da (+)-muscarina.


(a) Desenhe as estruturas do intermedirio B, a estrutura do produto D e escreva o reagente C, indicando
o solvente da reao, nos correspondentes quadros.


O composto Z, que um de dos estereoismeros de A, tratado com NaBH
4
como se mostra no esquema:
O
O
H
3
C COOCH
3
NaBH
4
H + I
MeOH, 25 C 85 % 15 %
Z

(b) Desenhe as estruturas de H e I nos correspondentes quadros, sabendo que ambos compostos
apresentam a mesma frmula molecular: C
7
H
12
O
4
.


(c) Qual relao estereoisomrica existe entre H e I. Marque com uma cruz (X) a resposta correta.

(d) Determinar a configurao absoluta dos centros estereognicos presentes no composto indicado
abaixo e escreva esta configurao nos quadros correspondentes.

O
O
H
3
C COOCH
3


(e) Desenhe a estrutura do enantimero do composto Z no quadro correspondente.


No esquema seguinte mostra-se a sequncia sinttica para a obteno do precur-
sor da (+)-muscarina.

PROBLEMA 2 (7%)

A atividade neurotxica de (+)-muscarina tornou-se assunto de intensa pesquisa direcionada para a sntese
de estereoismeros que permitiram estudar suas reaes com os receptores do sistema nervoso. A estrutura
desta substncia mostrada abaixo:

O
H
3
C CH
2
N(CH
3
)
3
HO
(+)-Muscarina


No esquema seguinte mostra-se a sequncia sinttica para a obteno do precursor da (+)-muscarina.


(a) Desenhe as estruturas do intermedirio B, a estrutura do produto D e escreva o reagente C, indicando
o solvente da reao, nos correspondentes quadros.


O composto Z, que um de dos estereoismeros de A, tratado com NaBH
4
como se mostra no esquema:
O
O
H
3
C COOCH
3
NaBH
4
H + I
MeOH, 25 C 85 % 15 %
Z

(b) Desenhe as estruturas de H e I nos correspondentes quadros, sabendo que ambos compostos
apresentam a mesma frmula molecular: C
7
H
12
O
4
.


(c) Qual relao estereoisomrica existe entre H e I. Marque com uma cruz (X) a resposta correta.

(d) Determinar a configurao absoluta dos centros estereognicos presentes no composto indicado
abaixo e escreva esta configurao nos quadros correspondentes.

O
O
H
3
C COOCH
3


(e) Desenhe a estrutura do enantimero do composto Z no quadro correspondente.


(a) Desenhe as estruturas do intermedirio B, a estrutura do produto D e escreva o
reagente C, indicando o solvente da reao, nos correspondentes quadros.
O composto Z, que um de dos estereoismeros de A, tratado com NaBH
4
como
se mostra no esquema:

PROBLEMA 2 (7%)

A atividade neurotxica de (+)-muscarina tornou-se assunto de intensa pesquisa direcionada para a sntese
de estereoismeros que permitiram estudar suas reaes com os receptores do sistema nervoso. A estrutura
desta substncia mostrada abaixo:

O
H
3
C CH
2
N(CH
3
)
3
HO
(+)-Muscarina


No esquema seguinte mostra-se a sequncia sinttica para a obteno do precursor da (+)-muscarina.


(a) Desenhe as estruturas do intermedirio B, a estrutura do produto D e escreva o reagente C, indicando
o solvente da reao, nos correspondentes quadros.


O composto Z, que um de dos estereoismeros de A, tratado com NaBH
4
como se mostra no esquema:
O
O
H
3
C COOCH
3
NaBH
4
H + I
MeOH, 25 C 85 % 15 %
Z

(b) Desenhe as estruturas de H e I nos correspondentes quadros, sabendo que ambos compostos
apresentam a mesma frmula molecular: C
7
H
12
O
4
.


(c) Qual relao estereoisomrica existe entre H e I. Marque com uma cruz (X) a resposta correta.

(d) Determinar a configurao absoluta dos centros estereognicos presentes no composto indicado
abaixo e escreva esta configurao nos quadros correspondentes.

O
O
H
3
C COOCH
3


(e) Desenhe a estrutura do enantimero do composto Z no quadro correspondente.


(b) Desenhe as estruturas de H e I nos correspondentes quadros, sabendo que am-
bos compostos apresentam a mesma frmula molecular: C
7
H
12
O
4
.
(c) Qual relao estereoisomrica existe entreH e I. Marque com uma cruz (X) a
resposta correta.

PROBLEMA 2 (7%)

A atividade neurotxica de (+)-muscarina tornou-se assunto de intensa pesquisa direcionada para a sntese
de estereoismeros que permitiram estudar suas reaes com os receptores do sistema nervoso. A estrutura
desta substncia mostrada abaixo:

O
H
3
C CH
2
N(CH
3
)
3
HO
(+)-Muscarina


No esquema seguinte mostra-se a sequncia sinttica para a obteno do precursor da (+)-muscarina.


(a) Desenhe as estruturas do intermedirio B, a estrutura do produto D e escreva o reagente C, indicando
o solvente da reao, nos correspondentes quadros.


O composto Z, que um de dos estereoismeros de A, tratado com NaBH
4
como se mostra no esquema:
O
O
H
3
C COOCH
3
NaBH
4
H + I
MeOH, 25 C 85 % 15 %
Z

(b) Desenhe as estruturas de H e I nos correspondentes quadros, sabendo que ambos compostos
apresentam a mesma frmula molecular: C
7
H
12
O
4
.


(c) Qual relao estereoisomrica existe entre H e I. Marque com uma cruz (X) a resposta correta.

(d) Determinar a configurao absoluta dos centros estereognicos presentes no composto indicado
abaixo e escreva esta configurao nos quadros correspondentes.

O
O
H
3
C COOCH
3


(e) Desenhe a estrutura do enantimero do composto Z no quadro correspondente.


Programa Nacional Olimpadas de Qumica <<
136
(d) Determinar a confgurao absoluta dos centros estereognicos presentes no
composto indicado abaixo e escreva esta confgurao nos quadros correspon-
dentes.

PROBLEMA 2 (7%)

A atividade neurotxica de (+)-muscarina tornou-se assunto de intensa pesquisa direcionada para a sntese
de estereoismeros que permitiram estudar suas reaes com os receptores do sistema nervoso. A estrutura
desta substncia mostrada abaixo:

O
H
3
C CH
2
N(CH
3
)
3
HO
(+)-Muscarina


No esquema seguinte mostra-se a sequncia sinttica para a obteno do precursor da (+)-muscarina.


(a) Desenhe as estruturas do intermedirio B, a estrutura do produto D e escreva o reagente C, indicando
o solvente da reao, nos correspondentes quadros.


O composto Z, que um de dos estereoismeros de A, tratado com NaBH
4
como se mostra no esquema:
O
O
H
3
C COOCH
3
NaBH
4
H + I
MeOH, 25 C 85 % 15 %
Z

(b) Desenhe as estruturas de H e I nos correspondentes quadros, sabendo que ambos compostos
apresentam a mesma frmula molecular: C
7
H
12
O
4
.


(c) Qual relao estereoisomrica existe entre H e I. Marque com uma cruz (X) a resposta correta.

(d) Determinar a configurao absoluta dos centros estereognicos presentes no composto indicado
abaixo e escreva esta configurao nos quadros correspondentes.

O
O
H
3
C COOCH
3


(e) Desenhe a estrutura do enantimero do composto Z no quadro correspondente.


(e) Desenhe a estrutura do enantimero do composto Z no quadro corresponden-
te.
(f) Por intermdio das duas reaes esquematizadas abaixo, obtemos os produtos
J e K, respectivamente. Desenhe suas estruturas.

(f) Por intermdio das duas reaes esquematizadas abaixo, obtemos os produtos J e K, respectivamente.
Desenhe suas estruturas.
O
HO
COOMe H
3
C
(i) KOH / H
2
O
calor
(ii) H
+
/ H
2
O
J K
O
HO
COOH H
3
C
O
O
COOMe H
3
C
CrO
3
.Piridina




17
a
OIAQ
>> Olimpada Brasileira de Qumica - 2012
137
Exame Terico
3
DADOS
PROBLEMA 3 (13%)
Com uso de tcnicas convencionais de anlise APHA (2005), foram realizadas anli-
ses em uma amostra de gua de poo extrada da zona norte da provncia de Santa
F, com a fnalidade de verifcar sua adequao para o consumo humano. Para que
a gua seja apropriada para o consumo humano, os parmetros selecionados de-
vem atender aos limites estabelecidos da Lei 11.220.
Neste problema voc deve considerar os dados experimentais das determinaes
realizadas. Na folha de rosto deste problema, voc encontrar a informao que
poder ser til.
(a) A determinao de cloretos foi feita pelo mtodo volumtrico de Mohr. Para
uma alquota de 100 mL de gua gastou 3,80 mL de uma soluo de nitrato de
prata 0,0500 mol/L at chegar no ponto fnal do indicador K
2
CrO
4
.
i. Escreva as equaes qumicas balanceadas que representam tanto a reao de
titulao como a reao do indicador.
ii. Determine a concentrao de cloreto presente na amostra, expressando a mes-
ma em mg(Cl) L-1.
Parmetro Unidade Valor obtido
Cloreto (Cl

) mg(Cl

)/L

(b) A denominada dureza da gua se deve a presena de sais de clcio e de mag-
nsio dissolvidos. A dureza total representa a soma das concentraes dos c-
tions clcio e magnsio, expressa em mg(CaCO
3
) L
-1
. Para determinar tanto a du-
reza total como as concentraes individuais dos referidos ctions, procedeu-se
da seguinte forma:
Uma alquota de 50,0 mL de gua foi ajustada ao pH = 10 e titulada com soluo
Programa Nacional Olimpadas de Qumica <<
138
de EDTA 0,0100 mol/L gastando 11,70 mL at a viragem do indicador NET (Ne-
gro de eriocromo T ). Neste pH ambos ctions esto complexados (Ca
2+
+ Mg
2+
).
Outra alquota de 100,0 mL foi ajustada para pH = 12 (neste pH um dos ctions
precipitado de uma forma seletiva e quantitativamente) e foi titulada com
a mesma soluo de EDTA, gastando-se 18,60 mL at a viragem do indicador
murexida.
i. Escreva as equaes qumicas balanceadas que representam as reaes
que aconteceram na titulao em pH = 10,0. (Pode ser uma ou mais re-
aes!). Considere que uma vez produzida a coordenao, o H
4
Y estar
sempre em sua forma completamente desprotonada (isto , coordena
como Y
4-
).
i. Escreva as equaes qumicas balanceadas que representam as reaes
que aconteceram na titulao em pH = 12,0. (Pode ser uma ou mais rea-
es!). Considere que uma vez produzida a coordenao, o H
4
EDTA esta-
r sempre em sua forma completamente desprotonada (isto , coordena
como EDTA
4-
).
ii. Determine a dureza total da soluo, expressa em mg(CaCO
3
) L
-1
.
iii. Determine a concentrao de clcio, expressa em mg(Ca) L
-1
, e a do mag-
nsio, expressa em mg(Mg) L.
-1
Parmetro Unidade Valor obtido
Dureza total mg(CaCO
3
) L
-1
Clcio mg(Ca) L
-1
Magnsio mg(Mg) L
-1
(c) A alcalinidade uma medida da capacidade da gua para neutralizar os cidos
e devida, essencialmente, presena de ons tais como carbonato, hidroge-
nocarbonato (HCO
3

) e hidrxido. A chamada alcalinidade total determinada


atravs da determinao da quantidade de cido (por exemplo HCl) necessria
para levar uma amostra de gua at pH = 4,2. O resultado ser expresso em
mg(CaCO
3
) L
-1
.
Para sua determinao, uma alquota de 100 mL da amostra de gua de pH = 7,6
foi titulada com uma soluo de HCl 0,0980 mol L-1, gastou-se 4,90 mL para atingir
o ponto de viragem do indicador verde bromocresol (intervalo de mudana: pH =
3,8 - 5,4).
17
a
OIAQ
>> Olimpada Brasileira de Qumica - 2012
139
Exame Terico
(Para os ons mencionados neste item, marque com uma cruz (X) a relao entre
suas concentraes na amostra que voc considera correta.
Parmetro Unidade Valor obtido
Dureza total mg(CaCO
3
) L
-1

Clcio mg(Ca) L
-1

Magnsio mg(Mg) L
-1


(c) A alcalinidade uma medida da capacidade da gua para neutralizar os cidos e devida,
essencialmente, presena de ons tais como carbonato, hidrogenocarbonato (HCO
3

) e hidrxido. A
chamada alcalinidade total determinada atravs da determinao da quantidade de cido (por exemplo
HCl) necessria para levar uma amostra de gua at pH = 4,2. O resultado ser expresso em
mg(CaCO
3
) L
-1
.
Para sua determinao, uma alquota de 100 mL da amostra de gua de pH = 7,6 foi titulada com uma
soluo de HCl 0,0980 mol L
-1
, gastou-se 4,90 mL para atingir o ponto de viragem do indicador verde
bromocresol (intervalo de mudana: pH = 3,8 - 5,4).
.
(Para os iones mencionados neste item, marque com uma cruz (X) a relao entre suas concentraes na
amostra que voc considera correta.

[CO
3
2
] ~ [OH

] [HCO
3

] [HCO
3

] [CO
3
2
] [OH

]

[CO
3
2
] ~ [HCO
3

] [OH

] [CO
3
2
] [HCO
3

] [OH

]


i. Escreva a equao qumica balanceada que representa a reao principal da titulao [conforme
sua resposta no subitem (i)].

ii. Complete o quadro relativo determinao da alcalinidade da amostra. Use a caixa abaixo
para mostrar os seus clculos.


Parmetro Unidade Valor obtido
Alcalinidade Total mg(CaCO
3
) L
-1

Carbonatos mg(CO
3
2
) L
-1

Hidrogeno carbonatos mg(HCO
3

) L
-1


(d) As espcies nitrogenadas presentes na amostra de gua foram determinadas, obtendo-se 5,60 mg
(NO
3

)/L e 0,011 mg(NO


2

) L
-1
.

Usou-se o mtodo colorimtrico para determinar a concentrao de
ctions da amnia. A partir de 50,0 mL de uma soluo padro de amnia de 0,500 mg de amnio
(N)/L e 50,0 ml de uma amostra de gua foi adicionada a cada deles um reagente cromgeno que
forma um complexo de cor amarela com a amnia e foram completados at 100 ml. As absorbncias
foram lidas num espectrofotmetro em um comprimento de onda de 425 nm, utilizando uma cela de
1,00 cm de caminho ptico, obteve-se um valor de 0,392 para o padro e 0,281 para a amostra.
Supondo que a Ley de Beer obedecida, determine a concentrao de amnio na amostra expressa
em mg(NH
4
+
) L
-1
.



Parmetro Unidade Valor obtido
Amnio mg(NH
4
+
)/L



i. Escreva a equao qumica balanceada que representa a reao princi-
pal da titulao [conforme sua resposta no subitem (i)].
ii. Complete o quadro relativo determinao da alcalinidade da amostra.
Use a caixa abaixo para mostrar os seus clculos.
Parmetro Unidade Valor obtido
Alcalinidade Total mg(CaCO
3
) L
-1
Carbonatos mg(CO
3
2
) L
-1
Hidrogeno carbonatos mg(HCO
3
) L
-1
(d) As espcies nitrogenadas presentes na amostra de gua foram determinadas,
obtendo-se 5,60 mg (NO
3

)/L e 0,011 mg(NO


2

) L
-1
. Usou-se o mtodo colorim-
trico para determinar a concentrao de ctions da amnia. A partir de 50,0 mL
de uma soluo padro de amnia de 0,500 mg de amnio (N)/L e 50,0 ml de
uma amostra de gua foi adicionada a cada deles um reagente cromgeno que
forma um complexo de cor amarela com a amnia e foram completados at
100 ml. As absorbncias foram lidas num espectrofotmetro em um compri-
mento de onda de 425 nm, utilizando uma cela de 1,00 cm de caminho ptico,
obteve-se um valor de 0,392 para o padro e 0,281 para a amostra.
Supondo que a Ley de Beer obedecida, determine a concentrao de amnio na
amostra expressa em mg(NH
4
+
) L
-1
.
Parmetro Unidade Valor obtido
Amnio mg(NH
4
+
)/L
(e) As guas contm principalmente os niones carbonato, hidrogeno carbonato,
sulfato e cloreto, e os ctions clcio, magnsio, sdio e potssio. Faa um balan-
o de cargas com o qual voc estimar a concentrao do sdio. No necess-
rio que determine esta concentrao numericamente!
Programa Nacional Olimpadas de Qumica <<
140
(f) Com os resultados obtidos em cada um dos pontos anteriores, siga o protocolo
que se segue para determinar se a gua analisada est apropriada para o con-
sumo humano, de acordo com os limites obrigatrios estabelecidos.
Determinao Unidade Valor
encontrado
Limite mximo
permitido *
pH Unidades de pH 7,6 No regulado
Turbidez U.N.T. 0,55 2
Condutividade mmhos cm
-1
710 No regulado
Resduos secos mg/L secado a 180C 472,5 1500
Alcalinidade mg(CaCO
3
)/L No regulado
Carbonatos mg(CO
3
2-
)/L No regulado
Bicarbonatos mg(HCO
3
-
)/L No regulado
Dureza Total mg(CaCO
3
)/L 100 500
Clcio mg(Ca
2
+
)/L 250
Magnsio mg(Mg
2
+
)/L 50
Sdio mg(N
a
+
)/L 77,6 200
Potssio mg(K
+
)/L 12,3 No regulado
Sulfato mg(SO4
2
-
)/L 80,1 400
Cloreto mg(Cl
-
/)/L 400
Nitratos mg(NO
3
-
)/L 5,6 45
Nitritos mg(NO
2
-
)/L 0,011 0,1
Amnio mg(NH
4
+
)/L 0,5
A gua analisada apropriada para consumo?
SIM NO
* Limites exigidos pela Lei 11.220, Servios de Sade Agncia Reguladora da Pro-
vncia de Santa Fe.
17
a
OIAQ
>> Olimpada Brasileira de Qumica - 2012
141
Exame Terico
4
DADOS
Propriedades do propano:
Temperatura de ebulio a 1 atm: T
eb
(1 atm) = -42,1 C.
Densidade no estado lquido: 0,5 kg/L
Propriedades da agua:
Presso de vapor a 25 C: p*(25C) = 0,0311 atm.
Para a reao em fase gasosa: CO
2
+ H
2
CO
2
H
2
,ala depen-
dncia da constante de equilbrio com a temperatura e: lin.(K) =
2,0352 4304,12/T(K)
Considerar que toda fase gasosa se comporta como gs ideal.
PROBLEMA 4 (10%)
Na Argentina existe uma rede de gs natural que abastece deste combustvel uma
proporo considervel da populao do pas. No obstante, para aqueles onde
no ha acesso a rede de gs natural em seus domiclios, a comercializao e feita
atravs do gs engarrafado em cilindros. Os maiores, de 90 litros, contem 45 quilo-
gramas de propano a 14 atm. e 25 C.
(a) Das opes listadas, marque com um (X) a que corresponde ao estado de
agregao do propano dentro do cilindro nestas condies e a correspondente
justifcativa da sua escolha.
Gs, na temperatura de 25 C e maior que a tempe-
ratura de ebulio (42,1 C a 1 atm.).
Lquido, o propano se liquefaz a p > 1 atm nessa tem-
peratura.
Lquido, em que o volume corresponde ao que ocupa
essa massa de propano nesse estado.
Programa Nacional Olimpadas de Qumica <<
142
Equilbrio lquido-vapor, o volume do cilindro e menor
do que o correspondente para o propano nessa fase
gasosa (1785 L).
(b) Para dispor de um dado combustvel em uma residncia se realiza uma insta-
lao de canos que distribui o propano gasoso para os locais de uso ( cozinha,
aquecedores). O cilindro se localiza em um local bem ventilado e no exterior
da residncia e esta conectado a uma tubulao atravs de um regulador de
presso que controla a expanso do propano, o qual ocorre quando se abre a
vlvula de gs no lugar de uso. Indique com um (X) o que se pode esperar que
acontea enquanto ocorre a troca de calor com o meio ambiente durante este
processo de descompresso isotrmica.
O propano cede calor ao meio ambiente (processo exotrmico).
O propano recebe calor do meio ambiente (processo endotrmico).
O propano no troca calor com o meio ambiente sem trabalho.
O propano no troca calor nem trabalho com o meio ambiente.
Nenhuma destas opes.
(c) Nas residncias se utiliza o propano para se obter energia em forma de calor,
para o qual se produz a combusto completa com o oxignio do ar. Escreva a
equao balanceada da reao de combusto.
(d) Supondo que o ar est formado somente por O
2
e N
2
, sendo a frao molar de
O
2
igual a 0,2 e que C
3
H
8
e O
2
, reagem em quantidades estequiomtricas, res-
ponda o seguinte:
i. Que massa de ar e necessria para consumir totalmente o propano con-
tido no cilindro?
ii. Supondo-se que o ar se comporta como um gs ideal, que volume (ex-
presso em m
3
) ocuparia a dita massa a 1 atm e 25 C?
(e) Se a combusto do item (d) se processa completamente e em quantidades es-
tequiomtricas de reagentes, a temperatura constante de 25 C e presso cons-
tante de 1 atm., responda:
i. Das opes seguintes, indique com um (X) que compostos estaro pre-
sentes no sistema fnal.
CO
2
, H
2
O
17
a
OIAQ
>> Olimpada Brasileira de Qumica - 2012
143
Exame Terico
CO
2
, H
2
O, N
2

CO
2
, H
2
O, N
2
, O
2

CO
2
, H
2
O, N
2
, O
2
, C
3
H
8

Nenhuma das opes anteriores.
ii. Levando em conta que parte da agua produzida se condensa, qual ser
a presso parcial da agua no sistema fnal? Marque com um (X) a res-
posta que considera correta.
(d) Supondo que o ar est formado somente por O
2
e N
2
, sendo a frao molar de O
2
igual a 0,2 e que
C
3
H
8
e O
2
, reagem em quantidades estequiomtricas, responda o seguinte:
i. Que massa de ar e necessria para consumir totalmente o propano contido no cilindro?


ii. Supondo-se que o ar se comporta como um gs ideal, que volume (expresso em m
3
) ocuparia a
dita massa a 1 atm e 25 C?


(e) Se a combusto do item (d) se processa completamente e em quantidades estequiomtricas de
reagentes, a temperatura constante de 25 C e presso constante de 1 atm., responda:
i. Das opes seguintes, indique com um (X) que compostos estaro presentes no sistema final.
CO
2
, H
2
O

CO
2
, H
2
O, N
2


CO
2
, H
2
O, N
2
, O
2


CO
2
, H
2
O, N
2
, O
2
, C
3
H
8


Nenhuma das opes
anteriores.


ii. Levando em conta que parte da agua produzida se condensa, qual ser a presso parcial da agua
no sistema final? Marque com um (X) a resposta que considera correta.

( ) atm p p
n n
n
O H p
CO O H
O H
571 , 0
7
4
2 2
2
2
= |
.
|

\
|
=
|
|
.
|

\
|
+
=

( ) atm p p
n n n
n
O H p
CO O H N
O H
148 , 0
27
4
2 2 2
2
2
= |
.
|

\
|
=
|
|
.
|

\
|
+ +
=

( ) atm C p
n n n
n
O H p
O H
CO O H N
O H
0046 , 0 ) 25 (
*
2
2 2 2
2
2
=
|
|
.
|

\
|
+ +
=

( ) atm C p O H p
O H
0311 , 0 ) 25 (
*
2 2
= =

Nenhuma das opes anteriores correta.
















(f) Quanta energia se transfere como calor na combusto de 1,00 mol de propano
a 25 C e 1 atm? Para este clculo podes desprezar a quantidade de gua em
fase gasosa e supor que 1 atm 1 bar.
Devido a problemtica do aquecimento global, se avalia a possibilidade de reciclar
o CO
2
atravs da previa separao dos gases que o acompanham..
Para isso se planeja transforma-lo novamente em combustvel (cido frmico) por
reduo com H
2
em fase gasosa e em quantidades estequiomtricas a 10 atm,
mediante a reao catalisada: CO
2
+ H
2
CO
2
H
2
, a qual alcana o equilbrio rapi-
damente.
(g) Calcule a temperatura desejada para que a frao molar de cido frmico seja
igual a 0,25.
Programa Nacional Olimpadas de Qumica <<
144
5
DADOS
Considerar DH
0
form

e DS
0
form
de reao independentes de T.
Considerar que a fase gasosa se comporta como ideal.
Composto D
f
H
0
m

(J.mol
-1
) D
f
S
0
m

(J.K
-1
mol
-1
)
Hugo (slido) -90374,0 -107,5
O2 (gs) 0 0
Hg (gs) 60798,0 132,7

PROBLEMA 5 (7%)
Em um sistema fechado rgido, previamente evacuado, cujo volume e de 980 litros
se colocam 7,5 quilogramas de xido de mercrio. A temperatura e elevada a 350
C e mantida constante por uma fonte externa. Nesta temperatura se processa a
decomposio do oxido de mercrio para dar oxigeno e mercrio.
2 HgO (s) O
2
(g) + 2 Hg (g)
(a) Considerando que se alcana o equilbrio qumico e que as nicas substncias pre-
sentes na fase gasosa so aquelas formados na decomposio do xido, calcule:
i. A composio da fase gasosa, em frao molar.
ii. A constante de equilbrio da reao na temperatura do sistema.
iii. A presso do sistema (em atmosferas).
iv. A presso parcial (em atmosferas) de cada um dos componentes
Defne-se o grau de avano ( ) de uma reao da seguinte maneira:

Aqui, n
i
e o nmero de mols da espcie i, n
0
i
e o nmero de mols iniciais da esp-
cie i, e v
i
e o coefciente estequiomtrico da espcie i na reao estudada (v
i
est
defnido como negativo para reagentes e positivo para produtos)
v. Calcule o nmero de mols de mercrio e a converso a converso (por-
centual) de xido de mercrio no aquilbrio.
(b) Caso se coloque inicialmente N
2
em um sistema sob as mesmas condies de
temperatura do ponto (a)),
17
a
OIAQ
>> Olimpada Brasileira de Qumica - 2012
145
Exame Terico
i. Indique com um (X) o que se espera que ocorra com nmero de mols de
Hg no equilbrio.
- Duplica
- Aumenta, j que aumenta o nmero de mols em fase gasosa
- Diminui, porque a reao com aumento do nmero de mols na fase gasosa
- No se modifca porque s depende da temperatura
ii. Indique com um (X) o que se espera que ocorra com a presso total do
sistema.
- Aumenta, j que aumenta o nmero de mols na fase gasosa
- No se modifca, porque o nitrognio e um gs inerte
- Diminui, porque diminui a converso
- No se modifca porque a presso do sistema fcara defnida pelo valor
da constante de equilbrio
Programa Nacional Olimpadas de Qumica <<
146
6
DADOS
Dados do NiO (xido de Nquel (II)):
r(Ni2+) = 0,69
r(O2-) = 1,40
A = 1,7476 (constante de Malungo)
n = 8 (expoente de Born)
DHf(NiO)= -239,7 kJ.mol-1
Dados de Ni (Nquel metlico):
r(Mi)=1,25
EI
1
(Ni)= 736,7 kJ.mol
-1
(primeira energia de ionizao)
EI
2
(Mi)= 1752,9 kJ.mol
-1
(segunda energia de ionizao)
DH
atoum
(Ni)= 430,0 kJ.mol
-1
(Entalpia de Atomizao,
associada ao processo de sublimao do metal)i
Dados do O (oxignio atmico):
AE
1
(O)= -141,0 kJ.mol
-1
(primeira afnidade eletrnica)
AE
2
(O)= 844,0 kJ.mol
-1
(segunda afnidade eletrnica)
Dados do O
2
(oxignio molecular):
D(O
2
)= 496,0 kJ.mol
-1
(Energia de Dissociao da molcula de O
2
)

PROBLEMA 6 (14%)
O esquema apresentado abaixo e um resume (de forma simplifcada) dos prin-
cipais processos envolvidos na obteno e purifcao de Nquel a partir de NiS,
mineral que se encontra na natureza impurifcado principalmente com espcies
de ferro.


PROBLEMA 6 (14%)

O esquema apresentado abaixo e um resume (de forma simplificada) dos principais processos envolvidos
na obteno e purificao de Nquel a partir de NiS, mineral que se encontra na natureza impurificado
principalmente com espcies de ferro.

Buscando analisar os diferentes passos desse processo, responda as seguintes perguntas:

ETAPA 1: Formao de xidos (Tostao):

(a) Descreva o processo de combusto dos sulfetos metlicos de ferro e nquel usando equaes
qumicas balanceadas.


O NiO cristaliza em uma estrutura cbica compacta de iones O
2-
. Empregando um modelo eletrosttico
simples e possvel determinar o tipo de buraco que ocupam os ctions na rede a partir do parmetro
=c
tion
/a
non
.

Razo de raios Buraco
0,225<<0,414
Tetradrico
0,414<<0,732
Octadrico
>0,732
Cbico



(b) Marque com um (X) a unidade da clula que representa de maneira razovel a estrutura NiO.




Para realizar uma analise energtica que permita elucidar sobre o tipo de interaes presentes na rede do
xido de Nquel, e muito importante poder estimar a Energia Reticular do mesmo (U
ret
), o que estas a
variao energtico da seguinte reao:

NiS(s)
FeS(s)
O
2
(g)
Etapa 1:
Tostacin
NiO(s)
Fe
2
O
3
(s)
Etapa 2:
Reduccin
Selectiva
C(s)
CO(g)
Ni(s)
Fe
2
O
3
(s)
CO(g)
60C
Ni(CO)
x
(g)
Fe
2
O
3
(s)
200C
Ni(s)
Etapa 3:
Separacin y purif icacin

Buscando analisar os diferentes passos desse processo, responda as seguintes per-
guntas:
17
a
OIAQ
>> Olimpada Brasileira de Qumica - 2012
147
Exame Terico
ETAPA 1: Formao de xidos (Tostao):
(a) Descreva o processo de combusto dos sulfetos metlicos de ferro e nquel
usando equaes qumicas balanceadas.
O NiO cristaliza em uma estrutura cbica compacta de iones O
2-
. Empregando um
modelo eletrosttico simples e possvel determinar o tipo de buraco que ocupam
os ctions na rede a partir do parmetro =c
tion
/a
non
.
Razo de raios Buraco
0,225<<0,414 Tetradrico
0,414<<0,732 Octadrico
>0,732 Cbico
(b) Marque com um (X) a unidade da clula que representa de maneira razovel a
estrutura NiO.
Para realizar uma analise energtica que permita elucidar sobre o tipo de intera-
es presentes na rede do xido de Nquel, e muito importante poder estimar a
Energia Reticular do mesmo (U
ret
), o que estas a variao energtico da seguinte
reao:
Ni
2+
(g) + O
2-
(g) NiO(s)
Em uma primeira aproximao, a energia reticula pode ser pensada como com-
posta dos termos:

Ni
2+
(g) + O
2-
(g) NiO(s)

Em uma primeira aproximao, a energia reticula pode ser pensada como composta dos termos:
cov
E E U
el ret
+ =

Aqui, E
le
e a contribuio puramente eletrosttica da energia reticular, e est originada pela interao de
cargas em torno da simetria da rede cristalina.
E
cov
,e a contribuio da estabilizao da rede dada por interaes covalentes presentes no solido, que so
responsveis de certa forma pela de localizao eletrnica.
Uma forma muito empregada para estimar a contribuio eletrosttica (E
le
) e conhecido como o modelo de
Born-Land. Nesse modelo, supe-se uma interao puramente eletrosttica entre os iones da rede, e
considera-se os mesmos esferas rgidas.
A equao que representa este modelo e a seguinte:
2
0 0
1
1
4
BL
ret
NAz z e
U
r n tc
+
| |
=
|
\ .

Aqui, N e o Nmero de Avogadro, A e a chamada Constante de Madelung, z
+
e z
-
so as cargas do ction e
do nion respectivamente, e e a carga do eltron, c
0
e a permissividade no vcuo, n e o chamado exponente
de Born, e r
0
e a soma dos raios do anon e do ction na rede.
(c) Determine a Energia Reticular do NiO (em kJ.mol
-1
) empregando o modelo de Born-Land.


alm do uso de modelos tericos, a energia reticular no pode ser determinada experimentalmente de
maneira direta Outra forma de determina-la e empregando dados experimentais determinados para espcies
relacionadas (energias de ionizao, afinidades eletrnicas, entalpias de formao, etc), usando os mesmos
para calcula-la indiretamente.
(d) Calcule a U
ret
(em kJ.mol
-1
) para o NiO indiretamente, empregando informaes experimentais
conhecidas das espcies que considera conveniente.


(e) A partir de dados obtidos em (c) e (d), marque com um (X) a(s) afirmao(es) que considere
correta(s). Se no puder calcular U
ret
em (c) e (d), empregue os valores U
ret
= -6800,0 kJ.mol
-1
e -
6870,0 kJ.mol
-1
respectivamente.

No NiO as interaes eletrostticas so predominantes e por tanto as
interaes covalentes podem ser desconsideradas..

No NiO as interaes covalentes so de grande magnitude e so mais
importantes. Que as interaes eletrostticas, e contribuem significativamente na
medida da U
ret
, por tanto a predio de B-L no combina com o valor real de
U
ret
.


No NiO as interaciones covalentes so da mesma ordem de magnitude que as
interaes eletrostticas.

No NiO as interaes eletrostticas so de ordens de magnitude menores que a
eventual covalncia presente no sistema, o que podemos concluir que o modelo
do solido pode ser considerado como o modelo de B-L.


Nenhuma das anteriores e correta.


ETAPA 2: Reduo seletiva do NiO






Aqui, E
le
e a contribuio puramente eletrosttica da energia reticular, e est ori-
ginada pela interao de cargas em torno da simetria da rede cristalina.
E
cov
,e a contribuio da estabilizao da rede dada por interaes covalentes pre-
Programa Nacional Olimpadas de Qumica <<
148
sentes no solido, que so responsveis de certa forma pela de localizao eletr-
nica.
Uma forma muito empregada para estimar a contribuio eletrosttica (E
le
) e co-
nhecido como o modelo de Born-Land. Nesse modelo, supe-se uma interao
puramente eletrosttica entre os iones da rede, e considera-se os mesmos esferas
rgidas.
A equao que representa este modelo e a seguinte:

Ni
2+
(g) + O
2-
(g) NiO(s)

Em uma primeira aproximao, a energia reticula pode ser pensada como composta dos termos:
cov
E E U
el ret
+ =

Aqui, E
le
e a contribuio puramente eletrosttica da energia reticular, e est originada pela interao de
cargas em torno da simetria da rede cristalina.
E
cov
,e a contribuio da estabilizao da rede dada por interaes covalentes presentes no solido, que so
responsveis de certa forma pela de localizao eletrnica.
Uma forma muito empregada para estimar a contribuio eletrosttica (E
le
) e conhecido como o modelo de
Born-Land. Nesse modelo, supe-se uma interao puramente eletrosttica entre os iones da rede, e
considera-se os mesmos esferas rgidas.
A equao que representa este modelo e a seguinte:
2
0 0
1
1
4
BL
ret
NAz z e
U
r n tc
+
| |
=
|
\ .

Aqui, N e o Nmero de Avogadro, A e a chamada Constante de Madelung, z
+
e z
-
so as cargas do ction e
do nion respectivamente, e e a carga do eltron, c
0
e a permissividade no vcuo, n e o chamado exponente
de Born, e r
0
e a soma dos raios do anon e do ction na rede.
(c) Determine a Energia Reticular do NiO (em kJ.mol
-1
) empregando o modelo de Born-Land.


alm do uso de modelos tericos, a energia reticular no pode ser determinada experimentalmente de
maneira direta Outra forma de determina-la e empregando dados experimentais determinados para espcies
relacionadas (energias de ionizao, afinidades eletrnicas, entalpias de formao, etc), usando os mesmos
para calcula-la indiretamente.
(d) Calcule a U
ret
(em kJ.mol
-1
) para o NiO indiretamente, empregando informaes experimentais
conhecidas das espcies que considera conveniente.


(e) A partir de dados obtidos em (c) e (d), marque com um (X) a(s) afirmao(es) que considere
correta(s). Se no puder calcular U
ret
em (c) e (d), empregue os valores U
ret
= -6800,0 kJ.mol
-1
e -
6870,0 kJ.mol
-1
respectivamente.

No NiO as interaes eletrostticas so predominantes e por tanto as
interaes covalentes podem ser desconsideradas..

No NiO as interaes covalentes so de grande magnitude e so mais
importantes. Que as interaes eletrostticas, e contribuem significativamente na
medida da U
ret
, por tanto a predio de B-L no combina com o valor real de
U
ret
.


No NiO as interaciones covalentes so da mesma ordem de magnitude que as
interaes eletrostticas.

No NiO as interaes eletrostticas so de ordens de magnitude menores que a
eventual covalncia presente no sistema, o que podemos concluir que o modelo
do solido pode ser considerado como o modelo de B-L.


Nenhuma das anteriores e correta.


ETAPA 2: Reduo seletiva do NiO






Aqui, N e o Nmero de Avogadro, A e a chamada Constante de Madelung, z
+
e
z
-
so as cargas do ction e do nion respectivamente, e e a carga do eltron, e
0
e
a permissividade no vcuo, n e o chamado exponente de Born, e r
0
e a soma dos
raios do anon e do ction na rede.
(c) Determine a Energia Reticular do NiO (em kJ.mol
-1
) empregando o modelo de
Born-Land.
alm do uso de modelos tericos, a energia reticular no pode ser determinada
experimentalmente de maneira direta Outra forma de determina-la e empregando
dados experimentais determinados para espcies relacionadas (energias de ioni-
zao, afnidades eletrnicas, entalpias de formao, etc), usando os mesmos para
calcula-la indiretamente.
(d) Calcule a U
ret
(em kJ.mol
-1
) para o NiO indiretamente, empregando informaes
experimentais conhecidas das espcies que considera conveniente.
(e) A partir de dados obtidos em (c) e (d), marque com um (X) a(s) afrmao(es)
que considere correta(s). Se no puder calcular Uret em (c) e (d), empregue os
valores U
ret
= -6800,0 kJ.mol-1 e -6870,0 kJ.mol
-1
respectivamente.
No NiO as interaes eletrostticas so predominantes e por tan-
to as interaes covalentes podem ser desconsideradas.
No NiO as interaes covalentes so de grande magnitude e so
mais importantes. Que as interaes eletrostticas, e contribuem
signifcativamente na medida da U
ret
, por tanto a predio de B-L
no combina com o valor real de U
ret
.
No NiO as interaciones covalentes so da mesma ordem de mag-
nitude que as interaes eletrostticas.
17
a
OIAQ
>> Olimpada Brasileira de Qumica - 2012
149
Exame Terico
No NiO as interaes eletrostticas so de ordens de magnitude
menores que a eventual covalncia presente no sistema, o que
podemos concluir que o modelo do solido pode ser considerado
como o modelo de B-L.
Nenhuma das anteriores e correta.
ETAPA 2: Reduo seletiva do NiO
No diagrama abaixo se mostra a dependncia de DG com a temperatura para as
reaes de formao de diferentes xidos (padronizados por mol de O
2
). A constru-
o do mesmo implica aproximar que DS
f
(T)=DS
f
0
e DH
f
(T)=DH
f
0
.
(f) Em que faixa de temperaturas possvel realizar a reduo seletiva de NiO a
partir da mistura de xidos? Marque com um (X) a resposta que voc considera
correta.
Em qualquer T>200K
Em qualquer T<820K
Para 700K<T<1100K
Para 700K<T<900K
Para qualquer temperatura
Programa Nacional Olimpadas de Qumica <<
150
(g) Por que, no diagrama anterior, a reta associada ao processo 2C(s)+O
2
(g) 2CO(g)
esta em declive em relao aos outros processos? Marque com um (X)a(s)
resposta(s) que considera correta(s):
Porque DS
f
(CO,g)<0 implica que para os xidos metlicos DS
f
>0.
Porque DS
f
(CO,g)>0 implica que para os xidos metlicos DS
f
<0.
Porque DH
f
(CO,g)>0 implica que para os xidos metlicos DH
f
<0.
Porque DH
f
(CO,g)<0 implica que para os xidos metlicos DH
f
>0.
Nenhuma esta correta.
ETAPA 3: Separao e Purifcao do Ni
Uma vez produzida a reduo seletiva do nquel, o mesmo se purifca e se se-
para do xido de ferro fazendo circular uma corrente de CO(g) a 60C, gerando
um composto de coordenao voltil, que por aquecimento posterior, dissocia os
ligantes recristalizando o metal puro. Realizou-se uma anlise elementar do com-
plexo Mi(CO)
x
, e o mesmo apresentou o seguinte resultado:
%C
28,1
(h) Calcule o valor de x.
A temperatura ambiente, o Nquel metlico cristaliza em uma estrutura cbica de
corpo centrado.

(i) Determine a densidade do Nquel metlico.
17
a
OIAQ
>> Olimpada Brasileira de Qumica - 2012
151
17
a
Olimpada Iberoamericana
de Qumica

22 a 30 de setembro,
em Santa F, Argentina
Experimento N.1 Pontuao: 20 pontos

1.1 - 1.12 1.a 1.b 1.c
46 Marcas 30 6 7 3
SNTESE DO [CO(EN)
2
(OX)]CL.H
2
O
Os complexos de cobalto foram, historicamente, os primeiros a ser estudados, e a
partir de ento uma srie destes compostos tm sido sintetizados, com o on met-
lico central em seus estados de oxidao +2 ou +3. Esta versatilidade do Co como
on central permite a possibilidade de um troca sistemtica de ligantes de forma a
obter sries nas quais os ligantes originais possam ser substitudos um a um por
outro que se interessa e assim poder estudar, tambm, de forma sistemtica, as
mudanas nas propriedades qumicas e fsicas destes compostos. A sistemtica
dessas mudanas permite a elaborao de modelos e teorias.
Neste experimento voc ir sintetizar um dos compostos pertencentes a uma srie
na qual possvel variar a estequiometria dos dois ligantes.
Procedimento:
1) Transfra a quantidade total do slido contido no frasco rotulado Acetato de
Co [contm 4,00g de Co(CH
3
COO)
2
.4H
2
O] a um Erlenmeyer de 100 mL. Adicio-
ne 20 mL de agua destilada.
2) Por outro lado, transfra o total do contedo do frasco rotulado Ac. Oxlico
[contm 3,00 g de cido oxlico dihidratado (H
2
C
2
O
4
.2H
2
O)] a um bquer de 100
mL e junte 25 mL de soluo de etilendiamina (C
2
H
8
N
2
) 1,7 M [frasco rotulado
Etilendiamina 1,7 M].
3) Aquea a soluo de acetato de cobalto at 60 C e a mistura de cido oxlico-
-etilendiamina at 70 C.
Exame Prtico
Programa Nacional Olimpadas de Qumica <<
152
4) Coloque a soluo de cido oxlico-en em um Erlenmeyer que contm a solu-
o de cobalto e aquea a mistura resultante at 80 C. Uma vez atingida a essa
temperatura, adicione, de uma s vez, 2,0 g de PbO
2
[contido em um tubo de
ensaio rotulado PbO
2
, 2g]. Agite rapidamente para homogeneizar o sistema.
5) Ferva suavemente a soluo resultante durante 30 min, agitando periodica-
mente. Aps os primeiros 10 minutos adicione os outros 0,4 g de PbO
2
e aps
20 min os outros 0,4 g [contidos nos tubos de ensaio rotulados PbO
2
, 0,4 g].
6) Deixe esfriar a soluo at atingir a temperatura ambiente (voc pode acelerar
o esfriamiento colocando o Erlenmeyer em um banho de gelo, uma vez que se
aqueceu um pouco, porm, cuidado para no esfriar abaixo da temperatura
ambiente!). Adicione 2,0 mL de soluo de H
2
SO
4
10 M e homogeneze bem.
Filtre a soluo utilizando um funil de Bchner e uma trompa de vcuo (Figura
1). Lavar o Erlenmeyer duas vezes com gua destilada (no mais de 1 ou 2 mL
no total). Descarte o papel de fltro com o slido retido nos recipientes postos
no laboratrio para essa fnalidade.
7) Em uma proveta mea 30,0 mL de gua destilada e transfra para um bquer de
100 mL. Marque cuidadosamente nvel do lquido com um marcador permanen-
te. Descarta gua. Transferir o fltrado do Kitasato para bquer com marcao.
Lavar o Kitasato com pequenssimas pores de gua destilada, adicionando
estas guas de lavagem para o mesmo recipiente, para evitar perdas de produto
(no mais de 1 ou 2 mL no total!). Adicione, com proveta, 5,0 mL de soluo de
HCl 10 M. Aquecer a soluo obtida evitando a ebulio (pode haver liberao
de bolhas ao longo do tempo), e evapore at atingir a marca de 30 mL.
8) Esfrie a soluo at a proximidade da temperatura ambiente e coloque-a em
banho de gelo. Agite periodicamente a soluo para induzir e facilitar a precipi-
tao do composto sintetizado.
9) Filtre os cristais obtidos a presso reduzida com um fltro de Bchner (Figura 1).
Mantenha a suco at que no caia mais gotas de lquido.
10) Transfra o total do composto obtido para um bquer limpo de 100 mL e pro-
ceda a recristalizao a partir de HCl 3M a 80 C. Tenha muito cuidado para no
adicionar excesso de solvente pois baixar o rendimento. Enfrie a soluo at
prximo a temperatura ambiente e, em seguida, em banho de gelo. Agite pe-
riodicamente a soluo para induzir e facilitar a precipitao do composto re-
cristalizado.
11) Filtre o produto obtido do mesmo modo que no item 9. Prepare 10 mL de uma
mistura de Metanol: H
2
O 80:20, e lave os cristais com essa mistura. Finalmente,
17
a
OIAQ
>> Olimpada Brasileira de Qumica - 2012
153
Exame Prtico
lava-los com 10 mL de Metanol puro. Mantenha a suco ate que no caia mais
gotas de liquido.
12) Transfra o produto obtido para uma placa de petri pequena (cristalizador), ro-
tulado com o seu cdigo de estudante. Este cristalizador foi previamente pe-
sado e o supervisor escrever esse dado na sua folha de respostas durante o
experimento. Entregue este cristalizador com o produto obtido ao supervisor
encarregado do laboratrio.
Ao fnalizar este experimento, entregar ao supervisor do laboratrio:
O cristalizador com o complexo sintetizado, rotulado com o seu nome
Dispositivo para fltracao a vcuo
Figura 1


Dispositivo para filtracao a vcuo

Figura 1


1- Para filtrar, corte um disco de papel de filtro que cubra totalmente os orifcios do funil de
Bchner e que fique perfeitamente plano sobre a placa filtrante. No deve apresentar
pregas; isso evitara que o solido escorra e voc tenha que filtrar a solucao. Molhe com
agua destilada o papel de filtro para aderi-lo sobre o fundo do funil. Arme o equipamento
de acordo com a Figura 1 utilizando um funil de Bchner (com papel de filtro), e kitasato,
preso em um suporte universal.
2- Ligue o dispositivo a uma das trompas de vcuo encontradas no laboratrio. Abra a
torneira e mantenha a suco durante o tempo solicitado pelo procedimento.
3- Deixe secar o produto por suco durante alguns minutos, ou de acordo com as instrues
do experimento.
4- Quando terminar, desconecte, primeiro, a mangueira da conexo com a trompa, e depois
feche a torneira.
5- Em caso de dvida, voc pode consultar um supervisor.



Ligao bomba de
gua
Folha de borracha que
impede a perda de vcuo
Disco de papel de filtro
plano, sem dobras
1 - Para fltrar, corte um disco de papel de fltro que cubra totalmente os orifcios
do funil de Bchner e que fque perfeitamente plano sobre a placa fltrante. No
deve apresentar pregas; isso evitara que o solido escorra e voc tenha que
fltrar a solucao. Molhe com agua destilada o papel de fltro para aderi-lo sobre
o fundo do funil. Arme o equipamento de acordo com a Figura 1 utilizando um
funil de Bchner (com papel de fltro), e kitasato, preso em um suporte univer-
sal.
2 - Ligue o dispositivo a uma das trompas de vcuo encontradas no laboratrio. Abra
a torneira e mantenha a suco durante o tempo solicitado pelo procedimento.
3 - Deixe secar o produto por suco durante alguns minutos, ou de acordo com as
instrues do experimento.
4 - Quando terminar, desconecte, primeiro, a mangueira da conexo com a trom-
pa, e depois feche a torneira.
5 - Em caso de dvida, voc pode consultar um supervisor.
Programa Nacional Olimpadas de Qumica <<
154
17
a
Olimpada Iberoamericana
de Qumica

22 a 30 de setembro,
em Santa F, Argentina
Experimento N. 2 Pontuao: 20 pontos

B.1 B.2 B.3 B.4 B.5 C.1 C.2 D.1 D.2 D.3
105 Marcas 2 62 12 5 3 3 4 4 6 4
PURIFICAO E QUANTIFICAO DO CIDO BENZICO
PRESENTE NUMA AMOSTRA
Um laboratrio adquiriu um frasco de cido benzico, que depois de ter sido usado
algunas vezes, observou-se que estava contaminado com uma substncia desco-
nhecida. Como no era possvel a sua substituio, decidiu-se purifcar o que exis-
tia. Aps uma srie de ensaios foi possvel saber que o composto que contaminava
o cido benzico era apolar. Consequentemente decidiu-se separar os compostos
usando a tcnica que em seguida se descreve e que voc ir usar para separar uma
amostra semelhante e possibilitar determinar a percentagem de pureza da amostra.
Parte A Separao dos compostos por extrao.
1) Prende argola metlica ao suporte universal e coloca o funil de decantao com
a torneira FECHADA (Figura 1). Retire a rolha do funil.
2) Transfra quantitativamente a amostra problema fornecida e existente no tubo
rotulado XX-X (o teu cdigo de estudante) /Muestra Bz / Masa: XXX g para
o funil de decantao. Anote, imediatamente, na folha de respostas a massa de
amostra indicada na etiqueta do tubo. Chame o supervisor para que ele assine
a folha junto do valor.
3) Adicione 20 mL de tolueno ao funil e agite circularmente para dissolver. O sli-
do pode no se dissolver completamente.
4) Adicione 10,00 mL de soluo de NaOH aprox. 0,4 mol L
1
(frasco rotuladoNaOH
17
a
OIAQ
>> Olimpada Brasileira de Qumica - 2012
155
Exame Prtico
0,4 M). Feche bem o funil com a rolha, retire-a do suporte e agite bem a mistu-
ra. Para tal deves manter o funil na horizontal, segurando bem a rolha com uma
mo e colocar a outra mo na torneira (Figura 2). Entre cada agitao permite
que se liberte o gs abrindo com cuidado a torneira. Quando efetuares esta
operao tem o cuidado de virar o funil para onde no estejam pessoas, para
evitar acidentes.




Experimento No.2 Pontuao: 20 pontos

B.1 B.2 B.3 B.4 B.5 C.1 C.2 D.1 D.2 D.3
105 Marcas 2 62 12 5 3 3 4 4 6 4
Purificao e quantificao do cido benzico presente numa
amostra
Um laboratrio adquiriu um frasco de cido benzico, que depois de ter sido usado algunas
vezes, observou-se que estava contaminado com uma substncia desconhecida. Como no era
possvel a sua substituio, decidiu-se purificar o que existia. Aps uma srie de ensaios foi
possvel saber que o composto que contaminava o cido benzico era apolar.
Consequentemente decidiu-se separar os compostos usando a tcnica que em seguida se
descreve e que voc ir usar para separar uma amostra semelhante e possibilitar determinar a
percentagem de pureza da amostra.

Parte A: Separao dos compostos por extrao.
1) Prende argola metlica ao suporte universal e coloca o funil de decantao com a
torneira FECHADA (Figura 1). Retire a rolha do funil.
2) Transfira quantitativamente a amostra problema fornecida e existente no tubo rotulado
XX-X (o teu cdigo de estudante) /Muestra Bz / Masa: XXX g para o funil de
decantao. Anote, imediatamente, na folha de respostas a massa de amostra indicada
na etiqueta do tubo. Chame o supervisor para que ele assine a folha junto do valor.
3) Adicione 20 mL de tolueno ao funil e agite circularmente para dissolver. O slido pode
no se dissolver completamente.
4) Adicione 10,00 mL de soluo de NaOH aprox. 0,4 mol L
1
(frasco rotuladoNaOH 0,4
M). Feche bem o funil com a rolha, retire-a do suporte e agite bem a mistura. Para tal
deves manter o funil na horizontal, segurando bem a rolha com uma mo e colocar a
outra mo na torneira (Figura 2). Entre cada agitao permite que se liberte o gs
abrindo com cuidado a torneira. Quando efetuares esta operao tem o cuidado de
virar o funil para onde no estejam pessoas, para evitar acidentes.


Figura 2
5) Coloque o funil na argola, retire a rolha e deixe repousar para que se separem as fases.
Abra a torneira e recolha a fase inferior (aquosa bsica) para um balo volumtrico de
100 mL. Deixe a fase superior (orgnica) no funil.
MO MO
5) Coloque o funil na argola, retire a rolha e deixe repousar para que se separem
as fases. Abra a torneira e recolha a fase inferior (aquosa bsica) para um balo
volumtrico de 100 mL. Deixe a fase superior (orgnica) no funil.
6) Repita a extrao da fase orgnica mais duas vezes, com pores de 10,00 mL
de NaOH 0,4 mol L
1
(frasco rotuladoNaOH 0,4 M). Reuna todas as fases aquo-
sas no mesmo balo volumtrico.
7) Finalmente, execute mais duas extraes, porm substituindo os 10 mL de so-
luo de NaOH por 10 mL de gua destilada medida com a proveta. Transfra
ambas fases aquosas para o mesmo balo volumtrico.
8) Feche bem o funil e reserva a fase orgnica para usar na parte C desta experin-
cia.
9) Acerte o volume fnal do balo volumtrico com gua destilada. No caso de
ter passado um pouco da fase orgnica para o balo volumtrico, deves ter o
cuidado de acertar considerando o mensco correspondente fase aquosa.
Parte B Determinao da pureza da amostra.
1) Transfra 25,00 mL da soluo existente no balo volumtrico para um Erlen-
meyer de 250 mL. Adicione-lhe 25 mL de gua destilada e 4 gotas de indicador
vermelho de cresol (frasco conta-gotas rotulado Rj Cresol).
2) Titule a soluo anterior com a soluo de HCl aprox. 0,1 mol L
1
(frasco
rotuladoHCl 0,1 M) at que ocorra a alterao de cor (Prpura Amarelo).
Anote o volume de titulante gasto na folha de respostas.
Programa Nacional Olimpadas de Qumica <<
156
3) Execute mais ensaios (duplicado ou triplicado) se achares melhor. Anote na fo-
lha de respostas o volume de titulante gasto.
Parte C Verifcao da efcincia da extrao.
1) Tens disponvel uma alquota da amostra analizada, que se encontra num tubo
rotulado Muestra Bz TLC. Ponha nesse tubo 10 gotas de metanol, com a aju-
da de uma pipeta de Pasteur e dissolva a alquota.
2) Aplique, numa placa de cromatografa e com a ajuda de tubos capilares (di-
ferentes), uma gota da soluo do item anterior e outra da fase orgnica pro-
veniente da extrao. Controla com a lmpada UV que a mancha aplicada
sufciente e s depois efetue a eluio da placa (Figura 3), utilizando o eluente
fornecido (hexano:acetato de etilo, 95:5), que est no frasco rotuladoSolvente
TLC. Segue as intrues existentes no Anexo.
3) Determine a Relao frontal (Rf ) de cada amostra e escreva na folha de respos-
tas o valor obtido.
Quando terminar esta experincia, entrega ao supervisor do laboratrio:
a placa de tlc, dentro da bolsa plstica dispolibilizada e rotulada com o seu c-
digo de estudante.
Penalizaes!
Voc pode solicitar materiais e/ou reagentes caso quebrem ou se acabem . A pena-
lidade ser de 0,5 ponto por cada substituio.
N Reagente / material Assinatura do estudante Assinatura do supervisor
Penalizaco total:_________ pontos
17
a
OIAQ
>> Olimpada Brasileira de Qumica - 2012
157
Exame Prtico
Anexo
Uso do funil de decantao
Figura 1



N
o
Reactivo / material Firma del
estudiante
Firma del
supervisor




Penalizacin total:_________ puntos.




Anexo

Uso do funil de decantao

Figura 1














Argola de metal
Rolha do funil
Torneira
Cromatografa em camada fna
Fornecemos a voc uma placa de slica gel 60 com indicador fuorescente e uma c-
mara de cromatografa que ser compartilhada com o seu vizinho de bancada. Deitar
na cmara de cromatografa o eluente adequado uns 10 minutos antes de efetuar a
cromatografa.
No toque na superfcie da placa com os dedos, nem raspe a placa. Segure-a sempre
pelas bordas e usando a pina.
Marque com um lpis a linha do ponto de aplicao (origem) e os pontos 1 e 2 tal
como se indica na fgura 2. Utilizando capilares, aplique sobre cada um desses pon-
tos, respectivamente, uma gota das solues a analisar (fase orgnica proveniente
da extrao e amostra original) (no misture os capilares). Deixe secar muito bem o
solvente de aplicao. Introduza a placa dentro da cmara de cromatografa, usando
a pina e com cuidado para que o nvel do solvente nao ultrapasse o ponto de apli-
cao das amostras. (Figura 2). ATENO: No mexa na cmara para no estragar o
desenvolvimento da cromatografa.
Deixe desenvolver a placa e retire-a da cmara quando o eluente tiver chegado apro-
ximadamente a 2mm abaixo da parte superior da placa. Marque com uma rgua a
frente do solvente e a distncia percorrida pelo mesmo. Deixe secar.
Observe a placa na lmpada UV. Marque as manchas correspondentes aos compostos,
usando um lpis. NUNCA olhe diretamente para a luz ultravioleta. Determine a dis-
tncia percorrida por cada uma das manchas, considerando a zona central da mancha.
Determine a Relao frontal (Rf ) dos compostos que observa na placa.
Cromatografa em camada delgada
Figura 2


Cromatografia em camada delgada

Figura 2













Cromatografia em camada fina
Fornecemos a voc uma placa de slica gel 60 com indicador fluorescente e uma cmara de
cromatografia que ser compartilhada com o seu vizinho de bancada. Deitar na cmara de
cromatografia o eluente adequado uns 10 minutos antes de efetuar a cromatografia.
No toque na superfcie da placa com os dedos, nem raspe a placa. Segure-a sempre pelas
bordas e usando a pina.
Marque com um lpis a linha do ponto de aplicao (origem) e os pontos 1 e 2 tal como se
indica na figura 2. Utilizando capilares, aplique sobre cada um desses pontos, respectivamente,
uma gota das solues a analisar (fase orgnica proveniente da extrao e amostra original)
(no misture os capilares). Deixe secar muito bem o solvente de aplicao. Introduza a placa
dentro da cmara de cromatografia, usando a pina e com cuidado para que o nvel do solvente
nao ultrapasse o ponto de aplicao das amostras. (Figura 2). ATENO: No mexa na cmara
para no estragar o desenvolvimento da cromatografia.
Deixe desenvolver a placa e retire-a da cmara quando o eluente tiver chegado
aproximadamente a 2mm abaixo da parte superior da placa. Marque com uma rgua a frente
do solvente e a distncia percorrida pelo mesmo. Deixe secar.
Observe a placa na lmpada UV. Marque as manchas correspondentes aos compostos, usando
um lpis. NUNCA olhe diretamente para a luz ultravioleta. Determine a distncia percorrida
por cada uma das manchas, considerando a zona central da mancha.
Determine a Relao frontal (R
f
) dos compostos que observa na placa.




origem
Distncia
perecorrida
por el
solvente
1 2
solvente
tapa
Programa Nacional Olimpadas de Qumica <<
158

Destaques olmpicos - 2012
Aps a concluso da Fase III da Olimpada Brasileira de Qumica 2011 Modalidade
A que aconteceu em agosto do ano anterior divulgou-se o resultado que indicou
estudantes premiados com medalhas. Todos esses prosseguiram para a etapa se-
guinte, em janeiro deste ano, com o incio do processo seletivo para a escolha dos
componentes da delegao brasileira nas olimpadas internacionais em 2012.
Defnida como OBQ-2011 Fase IV teve como objetivo a avaliao dos conhecimentos
de laboratrio. Foram classifcados 16 estudantes com base em suas mdias pondera-
das em composio com a nota da fase anterior, sendo eles: Cear Gabriel Matheus
Viana Pinheiro, Joaquim Ivo Vasques Dantas Landim, Vitria Nunes Medeiros, Francis-
co Davi Barbosa dos Santos, Nicholas de Souza Costa Lima, Sergio Pereira de Oliveira
Jnior, Lvia Rodrigues de Arajo, Nathrcia Castro Mota e Emerson Holanda Marinho;
So Paulo - Daniel Arjona de Andrade Hara, Henrique Magri Maron e Leonardo Kazu-
nori Tsuji; Rio Grande do Sul - Bruno Matissek Worm; Par - Ramon Santos Gonalves
da Silva; Gois - Kelvin Azevedo dos Santos; Piau - Vinicius Lopes Braga.
Quinze estudantes compareceram ao exame e, aps o resultado, foram convocados
para cumprir a Fase V da OBQ-2011, o Curso de Aprofundamento e Excelncia em Qu-
mica sob a responsabilidade do Programa de Ps-graduao em Qumica da UFMG.
Finalizado esse curso, que teve durao de quinze dias, os professores envolvidos apli-
caram o exame da fase fnal desse certame que apontou os quatro estudantes que
representaram o Brasil na 44th International Chemistry Olympiad (Washington) e na
17a Olimpada Iberoamericana de Qumica, em Santa F de La Veracruz - Argentina.
Daniel, Gabriel, Ramon e Vitria, delegao brasileira na 44th IChO (USA) e na 17a. OIAQ (Argentina)
Destaques
>> Olimpada Brasileira de Qumica - 2012
159
Destaques
Daniel, medalha de ouro na 17a. OIAQ e medalha de
Prata na 44th IChO
Ramon, medalha de prata na 17a. OIAQ e medalha
de Bronze na 44th IChO
Ramon, medalha de prata na 17a. OIAQ e medalha
de Bronze na 44th IChO
Gabriel, medalha de prata na 17a. OIAQ e medalha
de Bronze na 44th IChO
Programa Nacional Olimpadas de Qumica <<
160
Delegao brasileira da 44th IChO. Prof Arimatia Lopes, Vitria, Daniel, Gabriel, Ramon e Prof. Srgio Melo
Estudantes brasileiros no ptio da University of Maryland posam ao lado do renomado autor Prof. John C. Kotz.
Destaques
>> Olimpada Brasileira de Qumica - 2012
161
Polticas pblicas para
a qumica no Brasil
A minha participao nas olimpadas de qumica me enriqueceu muito como
estudante e como pessoa, e por isso eu sou muito grata de poder compartilhar
minha experincia. Tudo comeou quando eu participei da primeira edio da OBQ
Jr., em 2008. Embora a qumica parecesse um pouco sombria para mim nesse mo-
mento, recebi uma medalha de bronze e fui convidada para participar de treina-
mentos no meu colgio, o que fez com que eu me interessasse cada vez mais pela
qumica.
Embora eu no estivesse muito segura das minhas capacidades em qumica,
e j desconfasse que eu teria medo de orgnica pelo resto da minha vida, me di-
vertia estudando, e gostava muito dos meus professores e colegas de olimpada.
Graas minha participao na OBQ Jr., pude participar da OBQ de 2009 e, com
cada fase que ia passando, minha vontade de estudar qumica e minha admira-
o por todas as pessoas envolvidas com a olimpada, s iam aumentando. E assim
eu fui selecionada, entre os 15 fnalistas da OBQ de 2009, para fazer um curso na
Unicamp, preparatrio para a ltima seletiva para a IChO. As duas semanas que
passei na Unicamp me deram ainda mais incentivos para estudar: conheci outros
estudantes, que se tornaram meus amigos, apaixonados por qumica e professores
universitrios que estavam fazendo pesquisas maravilhosas na rea.Embora eu no
tenha sido selecionada para a IChO de 2010, estava to apaixonada pela olimpada
e por qumica, que decidi dar o meu melhor para ir para a IChO de 2011.
Enquanto eu participava de todas as fases da OBQ, outras olimpadas tambm
foram muito importantes para o meu aprendizado e me colocaram em contato
com muitas pessoas que passei a admirar. Minha participao na OQSP (Olmpia-
da de Qumica de So Paulo) e no TVQ (Torneio Virtual de Qumica), e o apoio de
professores e amigos, me permitiram chegar novamente fase fnal da OBQ e ser
selecionada para estar no time brasileiro. A 43rd IChO foi realizada na Turquia e foi
uma experincia que eu nunca esquecerei. Conhecer pessoas de mais 70 pases e
compartilhar experincias em um pas to histrico e lindo foi simplesmente de
tirar o flego. Aps essa que foi uma das viagens mais lindas da minha vida, fui
selecionada para participar da OIAQ (Olmpiada Ibero-Americana de Qumica), rea-
lizada em Teresina. Embora apenas 14 pases participassem e a olmpiada fosse no
meu pas, essa foi a melhor viagem da minha vida. Todos os estudantes se aproxi-
maram bastante, construram amizades que duram at hoje, e disseram ter vivido
as melhores experincias de suas vidas. E assim eu encerrei, de uma maneira muito
especial, minha carreira nas olmpiada de qumica.
Depoimento
Programa Nacional Olimpadas de Qumica <<
162
Depoimento
As olimpadas de qumica tiveram um grande impacto na minha vida.
Atualmente, eu sou uma estudante em Harvard, estudando Astrofsica e Cincias
Polticas. Embora eu no esteja cursando qumica, a minha paixo por cincias que
comeou com essa olimpada, fez com que eu descobrisse a astronomia. E a de-
terminao e amor que eu vi nos professores e organizadores envolvidos com as
olimpadas de qumica certamente me inspiram muito para seguir o meu sonho,
que trabalhar com poltica da educao no Brasil. Por isso, sou muita grata por ter
tido a oportunidade de participar de algo to incrvel.
Tabata Cludia Amaral de Pontes
OQSP (Olimpada de Qumica de So Paulo): 2010-ouro e 2011-ouro
OBQ Jr. (Olimpada Brasileira de Qumica Jr.): 2008-bronze
OBQ (Olimpada Brasileira de Qumica): 2009-prata, 2010-prata e 2011-meno honrosa
TVQ (Torneio Virtual de Qumica): 2010-ouro (individual) e -prata (equipe) e 2011-ouro (individual)
IChO (International Chemistry Olympiad): 2011-bronze
OIAQ (Olmpiada Ibero-Americana de Qumica): 2011-ouro
IOAA (International Olympiad on Astronomy and Astrophysics): 2010-bronze e 2011-meno honrosa
OLAA (Olimpada Latino-Americana de Astronomia e Astronutica): 2011-ouro

Aliando qumica medicina
Comecei a me envolver com as Olimpadas de Qumica no primeiro ano do
ensino mdio. O tema da redao da OQSP na poca era Qumica Forense. Decidi-
do a participar, e sabendo que minhas habilidades como escritor no eram boas,
decidi-me por estudar para a prova da FUVEST e tentar uma vaga de treineiro.
Com sucesso, continuei meus estudos em qumica, fcando em primeiro lugar na
OQSP-2009. Em seguida, stimo na OBQ. A olimpada me proporcionou esse am-
biente de estudos, incentivando cada vez mais conhecer a qumica. Cheguei at a
prova de vdeo, onde conheci um pouco mais da parte experimental. Sendo bem
sucedido, freqentei a UNICAMP por duas semanas, para ter aulas tericas no Curso
de Aprofundamento e Excelncia. A experincia foi maravilhosa: estive presente
num ambiente universitrio aumentando a vontade de estudar.
Fui selecionado, com a melhor nota, para representar o Brasil na 42 Olimpa-
da Internacional de Qumica, que ocorreu no Japo. A experincia foi novamente
maravilhosa: pude viajar at o Japo, e l fcar por 11 dias e conhecer muito de sua
cultura e costumes, alm de fazer diversos amigos de vrios pases. Ganhei meda-
>> Olimpada Brasileira de Qumica - 2012
163
Depoimento
lha de bronze. No fnal de outubro, participei da XV Olimpada Ibero-Americana
de Qumica no Mxico. Novamente, 12 dias conhecendo pessoas de outros pases,
culturas diferentes.
Semanas depois prestei vestibular para Medicina. Afrmo, com toda certeza, de
que no seria bem sucedido caso no tivesse me dedicado s olimpadas: estudan-
do qumica, tive enorme facilidade com a prova (inclusive a parte de fsica e a parte
qumica da biologia).
Hoje, estudo na Universidade de So Paulo e mesmo cursando Medicina, entro
em contato com a Qumica todos os dias, e muito do que aprendi nas Olimpa-
das eu uso no dia-a-dia com pacientes, ainda mais com os avanos da medicina
molecular, farmacologia, e exames diagnsticos. Ainda me dedico s Olimpadas
de Qumica como no s para agradecer, mas para ajudar outros jovens a terem
experincias maravilhosas tambm.
Idealizei e organizei com a ajuda do IQ-USP, amigos olmpicos e alguns docen-
tes (os quais destaco os nomes dos Professores Ivano Gutz, Omar El Seoud, Peter
Tiedemann e Srgio Melo), a Escola Olmpica de Qumica, que forneceu preparao
a jovens olmpicos do Estado de So Paulo premiados na OQSP para OBQ. Esses
jovens fcaram encantados com a EOQ por permitirem o contato com um ambiente
universitrio, incentivarem e mostrarem a qumica em outra perspectiva que no a
mesma do ensino mdio (talvez at mesmo uma qumica mais prxima da realida-
de), e a criao de laos de amizades.
Desse modo, sou muito grato a Olimpada de Qumica porque ela pode ajudar
diversos jovens a: encontrarem suas carreiras (seja na Qumica, Farmcia, Medicina,
ou at mesmo nas reas mais distantes, como Economia, para aqueles que perce-
berem que Qumica no lhes servem); experimentarem um estudo mais aprofun-
dado e universitrio, provocando um desenvolvimento intelectual, pois os incenti-
va a estudarem por conta prpria temas que muitas vezes no so usuais ao Ensino
Mdio; formarem novos laos de amizade e vivenciarem momentos nicos com
as atividades exclusivas das olimpadas; e representarem o pas numa competio
internacional, uma atividade de enorme honra.
Andr Silva Franco
Andr Silva Franco
Graduando em Medicina pela Faculdade de Medicina da USP
Coordenador da Escola Olmpica de Qumica
Medalha de Bronze na 42nd IChO - Tokyo
Medalha de Ouro na VX OIAQ - Cidade do Mxico
Medalhas de Ouro (2010) e Prata (2009) na OBQ
Medalha de Ouro (2010 e 2009) na OQSP
Programa Nacional Olimpadas de Qumica <<
164
Consideraes Finais
Formando docentes para a qumica
Teria eu dotes artsticos? No sei, nunca fui estimulado a exercitar o mais sim-
ples dos instrumentos musicais. necessrio por em prova, ser submetido a situa-
es desafadoras para concluir que capaz de resolv-las com certa facilidade.
nesse caminho que as aptides e os talentos se revelam e a vocao se afrma. As
olimpadas cientfcas desempenham esse papel, revelando talentos, estimulando-
-os prosseguir estudos nos nveis mais elevados. As olimpadas de qumica, abriga-
das no Programa Nacional Olimpadas de Qumica, no fogem regra, todas elas
cumprem esse objetivo. Organizadas por coordenadores estaduais e regionais es-
ses certames acontecem em diferentes recantos deste pas, em todos os estados
brasileiros, capilarizando-se por vilas e aldeias em escolas pblicas ou privadas e
alcanando um contingente que, neste ano, ultrapassou 169 mil estudantes. Para o
prximo ano, mais duas olimpadas regionais iro ampliar esse conjunto, foram in-
corporadas a Olimpada de Qumica do Centro-Oeste Potiguar (OQCOP), que deve-
r impulsionar o projeto no serto do Rio Grande do Norte com polos nas cidades
de Macau, Ipanguau, Caic, Santa Cruz, Currais Novos, Mossor, Apodi e Pau dos
Ferros e, tambm, a Olimpada de Qumica de Timon e regio, com abrangncia
nas cidades de Caxias e Cod, no Maranho. Contudo, a medida do comprovado
sucesso deste projeto reside na quantidade sempre crescente de ex-participantes
que decidem abraar a qumica, pelas portas do exame vestibular, ENEM e outros
processos seletivos e ingressam nos cursos de qumica e suas reas afns em univer-
sidades brasileiras e no exterior, neste ano no foi diferente.
Em 2012, um dos mais concorridos certames nacionais foi a Olimpada Brasi-
leira de Qumica Jnior que em sua fase fnal (Fase II) teve a participao de 4.296
estudantes do ensino fundamental, quantidade essa que representou os 15% dos
estudantes com melhor desempenho na etapa anterior (Fase I). A agradvel sur-
presa fcou por conta do resultado apresentado pelo grupo, uma distribuio mais
equitativa das medalhas que representam os escores mais elevados do certame,
dentre estes, dois dos participantes obtiveram 100% de acerto nas questes. A
OBQjr cumpre signifcativo papel no estmulo ao estudo da qumica, constatamos
que parte dos estudantes que alcanam a Fase VI da Olimpada Brasileira de Qumi-
ca iniciaram trajetria na OBQ Jnior e, se destacam dentre os demais. Neste ano,
dos quatro estudantes que compuseram a delegao que representou o brasil nas
olimpadas internacionais, trs foram oriundos da OBQjr.
A parceria da CAPES com Programa Nacional Olimpadas de Qumica permi-
tiu fortalecer uma cultura cientfca nas escolas envolvidas ao provocar mudanas
de atitudes, estimular a socializao e experincias didticas entre licenciandos
de qumica, e desmistifcar essa cincia com aulas ldicas, estimulantes da criati-
>> Olimpada Brasileira de Qumica - 2012
165
Consideraes Finais
vidade. O projeto denominado Aes Construtivas do Conhecimento Qumico nas
Escolas Pblicas, patrocinado por essa agncia, atuou em escolas pblicas de de
ensino mdio em treze estados brasileiros. A ao foi desenvolvida no contratur-
no escolar durante oito meses e teve por objetivo incentivar o hbito de estudar
qumica, aprimorar o rendimento dos alunos em sala de aula e prepar-los para a
participao nas olimpadas de qumica.
No que pesem as difculdades enfrentadas decorrentes dos longos perodos
de greve tanto nas IES quanto nas escolas pblicas, contabilizamos ganhos para os
alunos e para os licenciandos em qumica que atuaram como bolsistas no projeto.
Estes, em sua maioria, tiveram a sua primeira experincia na prtica docente no
exerccio deste projeto, um aspecto altamente positivo pelo apoio oferecido na for-
mao acadmica destes que sero futuros professores de qumica. O compartilha-
mento de experincias entre os bolsistas foi determinante para o enfrentamento
das defcincias dos alunos em relao a contedos e a tentativa de alfabetiz-los
cientifcamente.
Com relao aos alunos das escolas pblicas, a tnica foi motiv-los para o es-
tudo e estabelecer conexes com o cotidiano de cada um deles. Ao fnal, eviden-
ciou-se grande satisfao por parte dos prprio alunos em termos de domnio do
contedo e da autoconfana na participao em exames.
Agradecimentos queles que no se furtaram em contribur para o cresci-
mento da qualidade do Programa, particularmente os coordenadores-estaduais,
e registrar nosso regozijo pela nomeao do colega Dr. Arimatia Lopes, nosso
vice-coordenador nacional para o mais elevado cargo na Universidade Federal do
Piau. Avaliando o legado que construiu no Programa Nacional Olimpadas de Qu-
mica estou certo que deixar na UFPI a marca do seu efciente trabalho como reitor.
Srgio Melo
Programa Nacional Olimpadas de Qumica <<
166
Endereos
Endereos Coordenadorias
NCLEO COORDENADOR ENDEREO PROFISSIONAL
COORDENADORIA GERAL Prof. Srgio Maia Melo
melo@ufc.br
obq@ufc.br
Programa Nacional Olimpadas de
Qumica
Rua Antnio Lima, 191
60.115-270 Fortaleza - CE
COORDENADORIA REGIES SUL/SUDESTE Prof. lvaro Chrispino
alvaro.chrispino@gmail.com
CEFET - RJ
(21) 642.6644 (Telefax)
Vice-COORDENADORIA Prof. Jos Arimatia Dantas Lopes
arilopes@gmail.com
arilopes@ufpi.br
Universidade Federal do Piau
Centro de Cincias da Natureza
64.049-550 Teresina - PI
(86) 3215.5840 3215.5692 (Telefax)
ESTADO COORDENADOR ENDERECO PROFISSIONAL
ACRE Prof. Dlcio Dias Marques
delciomarques@globo.com
Prof. Rogrio A. Sartori
rogerio_sartori@yahoo.com.br
Universidade Federal do Acre - CCN
Depto. de Ciencias da Natureza BR 364 Km 4
63.915-900 Rio Branco - AC
(68) 3901.2591
ALAGOAS Prof. Joacy Vicente Ferreira
joacyferreira@ifal.edu.br
IFAL - Campus Macei
Instituto Federal de Alagoas
Rua Mizael Domingues, 75 Poo
57.020-600 Maceio - AL
Fone: (82) 2126-7000 / 7024 2126 .7050 (fax
AMAP
www.unifap.br/olimpiadas
Prof. Roberto Messias Bezerra
messias@unifap.br
Universidade Federal do Amap
Rod. Juscelino Kubitscheck, Km 02
68.902-280 Macap - AP
Fone: (96)3312-1700
AMAZONAS
www.oaq.ufam.edu.br
olimpadasdequimica@
hotmail.com
Profa. Tereza Cristina Souza de Olivei-
raterezacristina@ufam.edu.br
Prof . Paulo Rogrio da Costa Couceiro
couceiro@ufam.edu.br
Universidade Federal do Amazonas
Instituto de Cincias Exatas - Departamento de
Qumica - Bloco 10 - Departamento de Qumica/
ICE/UFAM Setor Norte do Campus Universitrio
Sen. Arthur Virglio Filho
Av . Gal . Rodrigo Otvio Jordo Ramos, 6.200
69.077-000 Manaus - AM Coroado
(92) 3305-2874 (telefone)
>> Olimpada Brasileira de Qumica - 2012
167
Endereos
BAHIA
www.obaq.ufba.br
Prof . Lafaiete Almeida Cardoso
lafaiete@ufba.br
Universidade Federal da Bahia
Instituto de Qumica - Depto . Qui . Orgnica
Rua Baro de Geremoabo, s/n (Ondina) 40 .170-
115 Salvador - BA
(71) 3283 .6813 3237.4117 (Fax)
CEAR
http://www.necim.ufc.br
Prof. Leonilde Maria Cmara Jatahy
necim@bol.com.br
leojatahy@ig.com.br
Prof. Cludia Christina B. S. Carneiro
Universidade Federal do Cear
NECIM - Ncleo de Ensino de Cincias e Matemtica
Av. da Universidade, 2470
60020-180 Fortaleza - Cear (85) 3366.7796
DISTRITO FEDERAL
http://www.petiq.unb.br/
Profa. Elaine Rose Maia
emaia@unb.br elaine.rose.maia@
gmail.com petqui@unb.br
Universidade de Braslia - Instituto de Qumica
Campus Universitrio Darcy Ribeiro -
70910-970 ICC Sul - Asa Norte Cx.Postal: 04478
(61) (61) 3107-3895 / 3893 / 3806
ESPRITO SANTO
www.cce.ufes.br/dqui/
ocq-es
Prof. Carlos Vital Paixo de Melo
cvpaixao@npd.ufes.br
Universidade Federal do Esprito Santo
Departamento de Qumica - CCE
Av. Fernando Ferrari, 514 Goiabeiras
29.075-910 Vitria ES
(27) 3335.2486 3335.2826
GOIS
http://www.jatai.cefetgo.
br/licenciatura/OBQ.html
www.obqgoias.com.br
Renato Cndido da Silva
obqgoias@yahoo.com.br
Prof. Carlos Czar da Silva
cefetjatai@yahoo.com.br
Prof. Hernane de Toledo Barcelos
Universidade Federal de Gois, Instituto de Qumica
Campos II Samambaia Bloco I
74001-970 Goiania, GO
Telefone: 62 3521.1167
Centro Federal de Educao Tecnolgica de Gois
UNED JATA
Rua Riachuelo, 2090 Setor Samuel Graham
75.800-000 Jata - GO
(64) 3632.8600
Centro Federal de Educao Tecnolgica de Gois
Campus Goinia
Rua 75, n 46, Centro.
74055-110. Goinia - GO
(62) 3227-2700
MARANHO Prof. Jean Carlo Antunes Catapreta
jcac889@hotmail.com
Universidade Federal do Maranho
Departamento de Qumica
Av. dos Portugueses, s/n Campus da Bacanga
65.080-040 So Luis - MA
Fone: (98) 3301.8280
MATO GROSSO Prof. Luiz Both
bothluiz@ibest.com.br ou luiz_both@
hotmail.com
IFMT
Rua 28, Quadra 38, Casa 14, Jardim Universitrio
78.075-592 Cuiab - MT
Fone: 65 3653.9206 (IFMT) 3663.1374
Programa Nacional Olimpadas de Qumica <<
168
MATO GROSSO DO SUL Prof. Onofre Salgado Siqueira
olimpiada.quimica.ms@gmail.com
Universidade Federal do Mato Grosso do Sul
Centro de Cincias Exatas e Tecnologia - Departa-
mento de Qumica (Cidade universitria)
79.070-900 Campo Grande - MS
(67) 345.3556 345.3552 (FAX)
MINAS GERAIS
http://www.qui.ufmg.br/
omq/
Profa. Ana Luiza de Quadros
omq.ufmg@gmail.com
Universidade Federal de Minas Gerais
Departamento de Qumica
Instituto de Cincias Exatas (ICEx)
Av. Pres. Antnio Carlos, 6627 Pampulha
31.270-901 Belo Horizonte - MG
(31) 3409.7558 Fax: (31) 3499.5700
PAR Prof. Mrcio de Souza Farias
toraqk@yahoo.com.br
Profa. Patrcia da Luz
pdaluz@yahoo.com
Instituto Federal do Par - IFPA
Departamento de Qumica
Avenida Almirante Barroso, 1155
Bairro do Marco
66093-020 Belm - PA
PARABA Prof. Vimario Simes Silva
vimario@deq.ufcg.edu.br
Francisco Ferreira Dantas Filho
Prof. Jos Estrela dos Santos
santosje@gmail.com
Universidade Federal de Campina Grande - Unidade
Acadmica de Engenharia Qumica
58.109-900 Campina Grande (83) 2101.1115
Universidade Estadual da Paraba UEPB -Campus
VIII - Centro de Cincias Tecnologia e Sade
Rua Coronel Pedro Targino SN, Centro - CEP:
5823300 - Araruna -PB - Telefone: (83) 3373-1040
- (83) 9919 -7772
Universidade Federal de Campina Grande
Rua Srgio Moreira de Figueiredo, S/N
Bairro: Casas Populares - 58.900-000 Cajazeiras - PB
(83) 3532-2040 3532.2063 3531.3940 (FAX)
PARAN
http://www.oprq.daqbi.
ct.utfpr.edu.br/
Prof. Jos Carlos Colombo
colombo@utfpr.edu.br
Universidade Tecnolgica Federal do Paran
Campus Curitiba - Sede Ecoville
Rua Dep. Heitor Alencar Furtado, 4900
81280-340 - Curitiba - PR - Brasil
Fone: (41) 3279.4575 ou 3279.4576
PERNAMBUCO
www.espacociencia.pe.gov.
br/index.php/atividade/
quimica/
Prof. Antnio Carlos Pavo
pavao@ufpe.br
Vice-coordenadora:
Profa. Lindomar Silva
lindomar.avelino88@gmail.com
Espao Cincia - Memorial Arcoverde, Complexo de
Salgadinho Olinda -PE
Fone: 81-3183.5525 / 3183.5528
Universidade Federal de Pernambuco, CCEN, Depto.
de Qumica Fundamental, Cidade Universitria, 50
740-521 Recife - PE
Fone: 81-2126.7415 / 2126 .8442 (fax)
Endereos
>> Olimpada Brasileira de Qumica - 2012
169
Endereos
PIAU
http://www.ufpi.br/
quimica/opq/
Prof. Jos Milton Elias de Matos
jmematos@gmail.com
Universidade Federal do Piau
Depto de Qumica - SG2 CCN - Campus da Ininga
64049-550 - Teresina - PI
Fone: (86) 3215.5620 telefax
RIO DE JANEIRO
http://sites.google.com/
site/olimpiadadequimicarj/
Prof. Paulo Chagas
paulo.chagas@ifrj.edu.br
Prof. Luis Carlos de Abreu Gomes
luisquimica@globo.com
IFRJ - Instituto Federal de Cincia e Tecnologia do
Rio de Janeiro - Rua Senador Furtado, 121 Praa da
Bandeira - 20.270-021 Rio de Janeiro - RJ
(21) 3978. 5918 3567.0283 (Fax)
CIEP 436 Neusa Brizola - 24.425 - 004 Neves - So
Gonalo - RJ - Fone: 21 9795 0176
Rua Dr. Jos Augusto Pereira dos Santos s/n
Fone: 21 8897-4492
RIO GRANDE DO NORTE
oqrn.quimica.ufrn.br
Prof. Fabiano do Esprito Santo Gomes
feibi_natal@yahoo.com.br
Profa. Maria de Ftima Vitria de
Moura
mfvmoura@quimica.ufrn.br
Universidade Federal do Rio Grande do Norte
Centro de Cincias Exatas e da Terra
Depto de Qumica
Av. Senador Salgado Filho, 3000 Lagoa Nova
Campus Universitrio
59.072-970 Natal - RN
Fone: (84) 3215.3828 R: 222 3215.9224 (Fax
RIO GRANDE DO SUL
http://gaia.liberato.com.br/
olimpiada/index.php
Twitter: http://twitter.com/
oqdors
Coordenao Colegiada:
Prof. Daniel Jacobus
Prof. Fvio Roberto Becker Dillio
Prof. Nair Cristina Muller
oqdors@gmail.com
quimica@liberato.com.br
Fundao Escola Tcnica Liberato Salzano V. da
Cunha
Rua Inconfdentes, 395 Primavera
93.340-140 - Novo Hamburgo - RS
(51) 3584.2027
RONDNIA Prof. Jlio Sancho Linhares Teixeira
Milito mailto:militao@unir.br
Prof. Jandi: costajandi@gmail.com
Fundao Universidade Federal de Rondnia- UNIR
Laboratrio de Qumica Analtica de Solos
Departamento de Qumica BR 364 km 9
78.000-000 Porto Velho - RO
Fone (69) 2182.2193
RORAIMA Profa. Maria Lcia Taveira
taveiraml@ig.com.br
Universidade Federal de Roraima
Departamento de Qumica - Campus do Paricarana
69.301-270 Boa Vista - RR
Fone: (95) 621.3140 621.3137 623.1581 224.7302
(Fax)
SANTA CATARINA Prof. Gilson Rocha Reynaldo
gilson@unisul.br
Jos Maximiliano Muller Netto
max@crq.org.br
Universidade do Sul de Santa Catarina - UNISUL
Av. Jos Accio Moreira, 787 - Caixa postal 370
88.704-900 Tubaro - SC Bairro Dehon Fone: (48)
621.3371 - Fax (48) 621 3000
Programa Nacional Olimpadas de Qumica <<
170
Endereos
SO PAULO
http://allchemy.iq.usp.br
Prof. Ivano G. R. Gutz
abqsp@iq.usp.br
gutz@iq.usp.br
ABQ Regional So Paulo
Instituto de Qumica da USP
Av. Prof. Lineu Prestes, 748 sala 1274
05.508-000 So Paulo - SP
(11) 3091.2159 (Mirian, 8 -12h) 3091.2150
SERGIPE
www.osequim.com.br
Prof. Juvenal Carolino da Silva Filho
jcarolino@hotmail.com
Profa. Eliana Midori Sussuchi
esmidori@gmail.com
Universidade Federal de Sergipe - Depto. de
Qumica
Campus Prof. Alberto Carvalho
Av. Vereador Olimpio Grande, s/n Centro
49.500.000 Itabaiana - SE
Fone: (79) 3431.8216 3432.8200
TOCANTINS
http://olimpiadatocanti-
nensedequimica.blogspot.
com.br/
Prof. Jos Expedito Cavalcante da Silva
mailto:eliete@unitins.br
jecs@mail.uft.edu.br
Universidade Federal do Tocantins
Coordenao de Qumica,
Rua Paraguai, s/n (esquina com Urixamas) - Setor
Cimba
77.838-824, Araguana-TO
Fone: 63 81112869, 63 21122201
PATROCINADORES
ABICLOR - Associao Brasileira da Indstria de lcalis e Cloro
Derivados
www.abiclor.com.br
Rua Chedid Jafet, 222 Bloco C 4 andar Vila Olmpia
04.551-065 So Paulo SP
(11) 2148.4780 FAX 2148.4788
ABIQUIM - Associao Brasileira da Indstria Qumica
www.abiquim.org.br
Rua Chedid Jafet, 222 Bloco C 4 andar Vila Olmpia
04.551-065 So Paulo SP
(11) 2148.4700 FAX 2148.4760
Banco do Nordeste do Brasil SA
www.bnb.gov.br
Av. Paranjana, 5700 Castelo
60.180-420 Fortaleza - CE
(85) 4005.3300
PROMOTORES
Universidade Federal do Cear
Pr-Reitoria de Extenso
Av. da Universidade, 2932 Campus do Benfca
60.020 Fortaleza - CE
Fone: (85) 3366.7300
Universidade Estadual do Cear
Pr-Reitoria de Extenso
Av. Dede Brasil, 1700 Paranjana
60.740-000 Fortaleza- CE
Fone: (85) 3299.2555
Universidade Federal do Piau
Pr-Reitoria de Extenso
Campus da Ininga
64.049-550 Teresina - PI
Fone: (86) 3215.5692 Fax: (86) 215.5570
>> Olimpada Brasileira de Qumica - 2012
171
Endereos
APOIO
CNPq - Conselho Nacional de Desenvolvimento Cientfco e
Tecnolgico
SHIS QI 1 Conjunto B - Bloco D, 2 andar
Edifcio Santos Dumont, Lago Sul
71605-190 Braslia - DF
Fone: (61) 3211-9408
CAPES - Coordenao de Aperfeioamento de Pessoal de
Nvel Superior
Ministrio da Educao - MEC
Setor Bancrio Norte, Quadra 02, Bloco L, Lote 6, 4. Andar
70.040-020 - Braslia/DF
Associao Brasileira de Engenharia Qumica
abeq@abeq.org.br
Rua Lbero Bardar, 152 - 11 andar - Centro
01008-903 So Paulo - SP
Fone: (11) 3107-8747 ou Telefax: (11) 3104-4649
Associao Norte-Nordeste de Qumica
www.annq.com
Caixa Postal 167
69.301-970 Boa Vista RR
Fone: (95) 3224.7128
Espao Cincia
www.espacociencia.pe.gov.br
Complexo de Salgadinho s/n Parque 2
53.111-970 Olinda PE
Fone: (81) 3301-6139
Fundao Estudar
www.estudar.org.br
Rua Tabapu, 145 Itaim Bibi
So Paulo - SP
REALIZADOR
Associao Brasileira de Qumica
www.abq.org.br
Av. Presidente Vargas, 633 sala 2208
20071-004 Rio de Janeiro RJ
Telefone: 21 2224-4480 Fax: 21 2224-6881
E-mail: abqrj@alternex.com.br
Programa Nacional Olimpadas de Qumica <<
172

Potrebbero piacerti anche